中考数学一轮复习教案完整版

申明敬告: 本站不保证该用户上传的文档完整性,不预览、不比对内容而直接下载产生的反悔问题本站不予受理。

文档介绍

中考数学一轮复习教案完整版

第一课时 实数的有关概念 知识点:有理数、无理数、实数、非负数、相反数、倒数、数的绝对值 大纲要求: 1. 使学生复习巩固有理数、实数的有关概念. 2. 了解有理数、无理数以及实数的有关概念;理解数轴、相反数、绝对值等概念,了解数 的绝对值的几何意义。 3. 会求一个数的相反数和绝对值,会比较实数的大小 4. 画数轴,了解实数与数轴上的点一一对应,能用数轴上的点表示实数,会利用数轴比较 大小。 考查重点: 1. 有理数、无理数、实数、非负数概念; 2.相反数、倒数、数的绝对值概念; 3.在已知中,以非负数 a2、|a|、 a (a≥0)之和为零作为条件,解决有关问题。 实数的有关概念 (1)实数的组成                    正整数 整数 零 负整数有理数 有尽小数或无尽循环小数 正分数实数 分数 负分数 正无理数无理数 无尽不循环小数 负无理数 (2)数轴:规定了原点、正方向和单位长度的直线叫做数轴(画数轴时,要注童上述规定的 三要素缺一个不可), 实数与数轴上的点是一一对应的。 数轴上任一点对应的数总大于这个点左边的点对应的数, (3)相反数 实数的相反数是一对数(只有符号不同的两个数,叫做互为相反数,零的相反效是零). 从数轴上看,互为相反数的两个数所对应的点关于原点对称. (4)绝对值        )0( )0(0 )0( || aa a aa a 从数轴上看,一个数的绝对值就是表示这个数的点与原点的距离 (5)倒数 实数 a(a≠0)的倒数是 a 1 (乘积为 1 的两个数,叫做互为倒数);零没有倒数. 考查题型: 以填空和选择题为主。如 一、考查题型: 1. -1 的相反数的倒数是 2. 已知|a+3|+ b+1 =0,则实数(a+b)的相反数 3. 数-3.14 与-Л的大小关系是 4. 和数轴上的点成一一对应关系的是 5. 和数轴上表示数-3 的点 A 距离等于 2.5 的 B 所表示的数是 6. 在实数中Л,-2 5 ,0, 3 ,-3.14, 4 无理数有( ) (A)1 个 (B)2 个 (C)3 个 (D)4 个 7.一个数的绝对值等于这个数的相反数,这样的数是( ) (A)非负数 (B)非正数 (C)负数 (D)正数 8.若 x<-3,则|x+3|等于( ) (A)x+3 (B)-x-3 (C)-x+3 (D)x-3 9.下列说法正确是( ) (A) 有理数都是实数 (B)实数都是有理数 (B) 带根号的数都是无理数 (D)无理数都是开方开不尽的数 10.实数在数轴上的对应点的位置如图,比较下列每组数的大小: (1) c-b 和 d-a (2) bc 和 ad 二、考点训练: 1.判断题: (1)如果 a 为实数,那么-a 一定是负数;( ) (2)对于任何实数 a 与 b,|a-b|=|b-a|恒成立;( ) (3)两个无理数之和一定是无理数;( ) (4)两个无理数之积不一定是无理数;( ) (5)任何有理数都有倒数;( ) (6)最小的负数是-1;( ) (7)a 的相反数的绝对值是它本身;( ) (8)若|a|=2,|b|=3 且 ab>0,则 a-b=-1;( ) 2.把下列各数分别填入相应的集合里 -|-3|,21.3,-1.234,-22 7 ,0,sin60°º,- 9 ,- 3 -1 8 , -Л 2 , 8 , ( 2 - 3 )0,3-2,ctg45°,1.2121121112......中 无理数集合{ } 负分数集合{ } 整数集合 { } 非负数集合{ } 3.已知 10,且 y<|x|,用"<"连结 x,-x,-|y|,y。 10.最大负整数、最小的正整数、最小的自然数、绝对值最小的实数各是什么? 11.绝对值、相反数、倒数、平方数、算术平方根、立方根是它本身的数各是什么? 12.把下列语句译成式子: (1)a 是负数 ;(2)a、b 两数异号 ;(3)a、b 互为相反数 ; (4) a、b 互为倒数 ;(5)x 与 y 的平方和是非负数 ; (6)c、d 两数中至少有一个为零 ;(7)a、b 两数均不为 0 。 13.数轴上作出表示 2 , 3 ,- 5 的点。 四.独立训练: 1.0 的相反数是 ,3-л的相反数是 , 3 -8 的相反数是 ;-л的绝对值 是 ,0 的绝对值是 , 2 - 3 的倒数是 2.数轴上表示-3.2 的点它离开原点的距离是 。 A 表示的数是-1 2 ,且 AB=1 3 ,则点 B 表示的数是 。 3 - 3 3 ,л,(1- 2 )º,-22 7 ,0.1313…,2cos60º, -3-1 ,1.101001000… (两 1 之间依次多一个 0),中无理数有 ,整数有 ,负数有 。 4. 若 a 的相反数是 27,则|a|= ;5.若|a|= 2 ,则 a= 5.若实数 x,y 满足等式(x+3)2+|4-y|=0,则 x+y 的值是 6.实数可分为( ) (A)正数和零(B)有理数和无理数(C)负数和零 (D)正数和负数 7.若 2a 与 1-a 互为相反数,则 a 等于( ) (A)1 (B)-1 (C)1 2 (D)1 3 8.当 a 为实数时, a2 =-a 在数轴上对应的点在( ) (C) 原点右侧(B)原点左侧(C)原点或原点的右侧(D)原点或原点左侧 *9.代数式 a |a| + b |b| + ab |ab| 的所有可能的值有( ) (A)2 个 (B)3 个 (C)4 个 (D)无数个 10.已知实数 a、b 在数轴上对应点的位置如图 (1)比较 a-b 与 a+b 的大小 (2)化简|b-a|+|a+b| 11 . 实 数 a 、 b 、 c 在 数 轴 上 的 对 应 点 如 图 所 示 , 其 中 | a | = | c | 试化简:|b-c|-|b-a|+|a-c-2b|-|c-a| 12.已知等腰三角形一边长为a,一边长b,且(2a-b)2+|9-a2|=0 。求它的周 长。 *13.若 3,m,5 为三角形三边,化简: (2-m)2 - (m-8)2 第二课 实数的运算 知识点:有理数的运算种类、各种运算法则、运算律、运算顺序、科学计数法、近似数与有 效数字、计算器功能鍵及应用。 大纲要求: 1.了解有理数的加、减、乘、除的意义,理解乘方、幂的有关概念、掌握有理数运算法则、 运算委和运算顺序,能熟练地进行有理数加、减、乘、除、乘方和简单的混合运算。 2.了解有理数的运算率和运算法则在实数运算中同样适用,复习巩固有理数的运算法则, 灵活运用运算律简化运算能正确进行实数的加、减、乘、除、乘方运算。 3.了解近似数和准确数的概念,会根据指定的正确度或有效数字的个数,用四舍五入法求 有理数的近似值(在解决某些实际问题时也能用进一法和去尾法取近似值),会按所要求的 精确度运用近似的有限小数代替无理数进行实数的近似运算。 4 了解电子计算器使用基本过程。会用电子计算器进行四则运算。 考查重点: 1.考查近似数、有效数字、科学计算法; 2.考查实数的运算; 3.计算器的使用。 实数的运算 (1)加法 同号两数相加,取原来的符号,并把绝对值相加; 异号两数相加。取绝对值较大的数的符号,并用较大的绝对值减去较小的绝对值; 任何数与零相加等于原数。 (2)减法 a-b=a+(-b) (3)乘法 两数相乘,同号得正,异号得负,并把绝对值相乘;零乘以任何数都得零.即       )(0 ),(|||| ),(|||| 为零或 异号 同号 ba baba baba ab (4)除法 )0(1  bbab a (5)乘方  个n n aaaa  (6)开方 如果 x2=a 且 x≥0,那么 a =x; 如果 x3=a,那么 xa 3 在同一个式于里,先乘方、开方,然后乘、除,最后加、减.有括号时,先算括号里面. 3.实数的运算律 (1)加法交换律 a+b=b+a (2)加法结合律 (a+b)+c=a+(b+c) (3)乘法交换律 ab=ba. (4)乘法结合律 (ab)c=a(bc) (5)分配律 a(b+c)=ab+ac 其中 a、b、c 表示任意实数.运用运算律有时可使运算简便. 典型题型与习题 一、填空题: 1.我国数学家刘徽,是第一个找到计算圆周率π方法的人,他求出π的近似值是 3.1416, 如果取 3.142 是精确到 位,它有 个有效数字,分别是 。 1.5972 精确到百分位的近似数是 ;我国的国土面积约为 9600000 平方干米,用 科学计数法表示为 平方干米。 2.按鍵顺序-1·2÷4=,结果是 。 3.我国 1990 年的人口出生数为 23784659 人。保留三个有效数字的近似值是 人。 4.由四舍五入法得到的近似数 3.10×104,它精确到 位。这个近似值的有效数字 是 。 5.2 的相反数与倒数的和的绝对值等于 。 6.若 n 为自然数时(-1)2n+1+(-1)2n= . 7.查表得 2.132=4.5,4.1053=69.18,则-21.32= 。(-0.0213)2= ,0.41053 = ,-(-410.5)3= 。若 8.3202=69.32,x2=6.932×105,则 x= . 4.44 =2.107 44.4 =6.663 0.00444 = . 8.已知 2a-b=4, 2(b-2a)2-3(b-2a)+1= 9.已知:|x|=4,y2= 1 49 且 x>0,y<0,则 x-y= 。 二、选择题 1. 下列命题中:(1)几个有理数相乘,如果负因数个数是奇数,则积必为负; (2)两数之积为 1,那么这两数都是 1 或都是-1;(3)两个实数之和为正数,积为负数, 则两数异号,且正数的绝对值大;(4)一个实数的偶次幂是正数,那么这个实数一定不等于 零,其中错误的命题的个数是( ) (A)1 个 (B)2 个 (C)3 个 (D)4 个 2.近似数 1.30 所表示的准确数 A 的范围是( ) (A)1.25≤A<1.35 (B)1.20<A<1.30 (C)1.295≤A<1.305 (D)1.300≤A<1.305 3.设 a 为实数,则|a+|a||运算的结果( ) (A) 可能是负数(B)不可能是负数(C)一定是负数(D)可能是正数。 4.已知|a|=8,|b|=2,|a-b|=b-a,则 a+b 的值是( ) (A) 10 (B)-6 (C)-6 或-10 (D)-10 5.绝对值小于 8 的所有整数的和是( ) (A)0 (B)28 (C)-28 (D)以上都不是 6.由四舍五入法得到的近似数 4.9 万精确到( ) (A)万位 (B)千位 (C)十分位 (D)千分位 7. 计算下列各题: (1) 32÷(-3)2+|- 1 6 |×(- 6)+ 49 ; (2) {21 3 (-1 2 )-2 3 × 3 -8 ÷1 6 }×(-6); (3)-0.252÷(-1 2 )4+(11 2 +23 8 -3.75)×24; (4){-3(2 3 )2-22 ×0.125-(-1)3÷3 4 }÷{2×(-1 2 )2-1}。 (5){1 2 ×(-2)2-(1 2 )2+ 1 1-1 3 }÷| 21996·(-1 2 )1995| . (6) (-2)3×(-1)4- (-12)2 ÷{-(1 2 )2} 0.25×4+{1-32×(-2)} (7)0.3-1-(- 1 6 )-2+43-3-1+(π-3)0+tg2300 (8)(2 3 )-1-(2001+ctg300)0+(-2)2··· 1 16 + 1 2-1 第 3 课 整式 知识点 代数式、代数式的值、整式、同类项、合并同类项、去括号与去括号法则、幂的运算法则、 整式的加减乘除乘方运算法则、乘法公式、正整数指数幂、零指数幂、负整数指数幂。 大纲要求 1、 了解代数式的概念,会列简单的代数式。理解代数式的值的概念,能正确地求出代数式 的值; 2、 理解整式、单项式、多项式的概念,会把多项式按字母的降幂(或升幂)排列,理解同 类项的概念,会合并同类项; 3、 掌握同底数幂的乘法和除法、幂的乘方和积的乘方运算法则,并能熟练地进行数字指数 幂的运算; 4、 能熟练地运用乘法公式(平方差公式,完全平方公式及(x+a)(x+b)=x2+(a+b)x+ab)进 行运算; 5、 掌握整式的加减乘除乘方运算,会进行整式的加减乘除乘方的简单混合运算。 考查重点 1.代数式的有关概念. (1)代数式:代数式是由运算符号(加、减、乘、除、乘方、开方)把数或表示数的字母 连结而成的式子.单独的一个数或者一个字母也是代数式. (2)代数式的值;用数值代替代数式里的字母,计算后所得的结果 p 叫做代数式的值. 求代数式的值可以直接代入、计算.如果给出的代数式可以化简,要先化简再求值. (3)代数式的分类 2.整式的有关概念 (1)单项式:只含有数与字母的积的代数式叫做单项式. 对于给出的单项式,要注意分析它的系数是什么,含有哪些字母,各个字母的指数分别 是什么。 (2)多项式:几个单项式的和,叫做多项式 对于给出的多项式,要注意分析它是几次几项式,各项是什么,对各项再像分析单项 式那样来分析 (3)多项式的降幂排列与升幂排列 把一个多项式技某一个字母的指数从大列小的顺序排列起来,叫做把这个多项式按这个 字母降幂排列 把—个多项式按某一个字母的指数从小到大的顺斤排列起来,叫做把这个多项式技这个 字母升幂排列, 给出一个多项式,要会根据要求对它进行降幂排列或升幂排列. (4)同类项 所含字母相同,并且相同字母的指数也分别相同的项,叫做同类顷. 要 会 判 断 给 出 的 项 是 否 同 类 项 , 知 道 同 类 项 可 以 合 并 . 即 xbabxax )(  其中的 X 可以代表单项式中的字母部分,代表其他式子。 3.整式的运算 (1)整式的加减:几个整式相加减,通常用括号把每一个整式括起来,再用加减号连接.整 式加减的一般步骤是: (i)如果遇到括号.按去括号法则先去括号:括号前是“十”号,把括号和它前面的“+” 号去掉。括号里各项都不变符号,括号前是“一”号,把括号和它前面的“一”号去掉.括 号里各项都改变符号. (ii)合并同类项: 同类项的系数相加,所得的结果作为系数.字母和字母的指数不变. (2)整式的乘除:单项式相乘(除),把它们的系数、相同字母分别相乘(除),对于只在 一个单项式(被除式)里含有的字母,则连同它的指数作为积(商)的一个因式相同字母相乘 (除)要用到同底数幂的运算性质: ),,0( ),( 是整数 是整数 nmaaaa nmaaa nmnm nmnm     多项式乘(除)以单项式,先把这个多项式的每一项乘(除)以这个单项式,再把所得的积 (商)相加. 多项式与多项式相乘,先用一个多项式的每一项乘以另一个多项式的每一项,再把所得 的积相加. 遇到特殊形式的多项式乘法,还可以直接算: .))(( ,2)( ,))(( ,)())(( 3322 22 22 2 babababa bababa bababa abxbaxbxax      (3)整式的乘方 单项式乘方,把系数乘方,作为结果的系数,再把乘方的次数与字母的指数分别相乘所 得的幂作为结果的因式。 单项式的乘方要用到幂的乘方性质与积的乘方性质: )()( ),,()( 是整数 是整数 nbaab nmaa nnn mnnm   多项式的乘方只涉及 .222)( ,2)( 2222 222 cabcabcbacba bababa   考查重点与常见题型 1、 考查列代数式的能力。题型多为选择题,如: 下列各题中,所列代数错误的是( ) (A) 表示“比 a 与 b 的积的 2 倍小 5 的数”的代数式是 2ab-5 (B) 表示“a 与 b 的平方差的倒数”的代数式是 1 a-b2 (C) 表示“被 5 除商是 a,余数是 2 的数”的代数式是 5a+2 (D) 表示“数的一半与数的 3 倍的差”的代数式是a 2 -3b 2、 考查整数指数幂的运算、零指数。题型多为选择题,在实数运算中也有出现,如: 下列各式中,正确的是( ) (A)a3+a3=a6 (B)(3a3)2=6a6 (C)a3•a3=a6 (D)(a3)2=a6 整式的运算,题型多样,常见的填空、选择、化简等都有。 考查题型: 1.下列各题中,所列代数错误的是( ) (E) 表示“比 a 与 b 的积的 2 倍小 5 的数”的代数式是 2ab-5 (F) 表示“a 与 b 的平方差的倒数”的代数式是 1 a-b2 (G) 表示“被 5 除商是 a,余数是 2 的数”的代数式是 5a+2 (H) 表示“数a的一半与数b的 3 倍的差”的代数式是a 2 -3b 2.下列各式中,正确的是( ) (A)a3+a3=a6 (B)(3a3)2=6a6 (C)a3•a3=a6 (D)(a3)2=a6 3.用代数式表示:(1)a 的绝对值的相反数与 b 的和的倒数; (2)x 平方与 y 的和的平方减去 x 平方与 y 的立方的差; 4.- лa2b3 12 的系数是 ,是 次单项式; 5.多项式 3x2-1-6x5-4x3 是 次 项式,其中最高次项是 ,常数项是 ,三次项 系数是 ,按 x 的降幂排列 ; 6.如果 3m7xny+7 和-4m2-4yn2x 是同类项,则 x= ,y= ;这两个单项式的积是__。 7.下列运算结果正确的是( ) ①2x3-x2=x ②x3•(x5)2=x13 ③(-x)6÷(-x)3=x3 ④(0.1)-2•10-1=10 (A)①② (B)②④ (C)②③ (D)②③④ 考查训练: 1、代数式 a2-1,0, 1 3a ,x+1 y ,-xy2 4 ,m,x+y 2 , 2 –3b 中单项式是 ,多项 式是 ,分式是 。 2、-x2yz3 3 是 次单项式,它的系数是 。 3、多项式 3yx2-1-6y2x5-4yx3 是 次 项式,其中最高次项是 ,常数项是 ,三次 项系数是 ,按 x 的降幂排列为 。 4、已知梯形的上底为 4a-3b,下底为 2a+b,高为 3a+b。试用含 a,b 的代数式表示出梯形的 面积,并求出当 a=5,b=3 时梯形的面积。 5、下列计算中错误的是( ) (A)(-a3b)2·(-ab2)3=-a9b8 (B) (-a2b3)3÷(-ab2)3=a3b3 (C)(-a3)2·(-b2)3=a6b6 (D)[(-a3)2·(-b2)3]3=-a18b18 6、计算:3xy3·(-1 2 x3y4)÷(-1 6 x2y3)2 7.已知代数式 3y2-2y+6 的值为 8,求代数式3 2 y2-y+1 的值 8.设a-b=-2,求a2+b2 2 -ab的值。 7、利用公式计算: (1) (1 3 a2-1 4 b)( -1 4 b-1 3 a2) (2) (a-1 2 )2 (a2+1 4 )2(a+1 2 )2 (3)(x+y-z)(x-y+z)-(x+y+z)(x-y-z) (4)[(x2+6x+9) ÷(x+3)](x2-3x+9) (5)(a2-4)(a2-2a+4)(a2+2a+4) (6)101×99 解题指导: 1、代数式 15 -2x2 3 是( ) (A)整式 (B)分式 (C)单项式 (D)无理式 2、如果 3x7-myn+3 和-4x1-4my2n 是同类项,那么 m,n 的值是( ) (A)m=-3,n=2 (B) m=2,n=-3 (C) m=-2,n=3 (D) m=3,n=-2 3、正确叙述代数式1 3 (2a-b2)的是( ) (A) a与 2 的积减去b平方与 3 的商 (B)a与 2 的积减去b的平方的差除以 3 (C)a与 2 倍减去b平方的差的1 3 (D)a的 2 倍减去b平方1 3 4、用乘法公式计算: (1) (-2a-3b)2 (2) (a-3b+2c)2 (3) (2y-z)2[2y(z+2y)+z2]2 5、计算: (1)(c-2b+3a)(2b+c-3a) (2)(a-b)(a+b)2-2ab(a2-b2) 6、用竖式计算: (5-4x3+5x2+2x4)÷(3+x2-2x) 7、已知 6x3-9x2+mx+n 能被 6x2-x+4 整除,求 m,n 的值,并写出被除式。 8、已知x+y=4,xy=3,求:3x2+3y2;(x-y)2 巩固提高 1、 若一个多项式加上 2x2-x3-5-3x4 得 3x4-5x3-3,则这个多项式是 ; 2、 若 3xn-(m-1)x+1 为三次二项式,则 m-n2 的值为 ; 3、 用代数式表示,m,n 两数的和除这两数的平方的差 ; 用语言叙述代数式x3-3 6 ; 4.若除式=x+2,商式=2x+1,余式=-5,则被除式= ; 5、当 x=-2 时,ax3+bx-7=5,则 x=2 时,ax3+bx-7= ; a-b=-2,a-c=-3,则(b-c)2-3(b-c)+1= 6、如果(a+b-x)2 的结果中不含的 x 一次项,那么 a,b 必满足( ) (A) a=b (B)a=0,b=0 (C)a=-b (D)以上都不对 7、-[a-(b-c)]去括号正确的是( ) (A) -a-b+c (B)-a+b-c (C)-a-b-c (D)-a+b+c 8、设 P 是关于 x 的五次多项式,Q 是关于 x 的三次多项式,则( ) (A)P+Q 是关于的八次多项式 (B)P-Q 是关于的二次多项式 (C)P·Q 是关于的八次多项式 (D)Q P 是关于的二次多项式 9.下列计算中正确的是( ) (A)xn+2÷xn+1=x2 (B)(xy)5÷xy3=(xy)2 (C)x10÷(x4÷x2)=x8 (D)(x4n÷x2n) ·x3n=x3n+2 10.若(am+1bn+2)(a2n-1b2m)=a5b3,则m+n的值为( ) (A)1 (B)2 (C)3 (D)-3 11、计算: (1) (-2ax)2·(-2 5 x4y3z3) ÷(-1 2 a5xy2) (2) (1 3 an+2+2an+1) ÷(-1 3 an-1) (3) 5(m+n)(m-n)-2(m+n)2-3(m-n)2 (4)(a-b+c-d)(-a-b-c-d) (5)(-x-y)2(x2-xy+y2)2 (6)15+2a-{9a-[a-9-(3-6a)]} (7)(a2c-bc2)-(a-b+c)(a+b-c) *(8)(a-b)(a+b)2-(a+b)(a-b)2+2b(a2+b2) 第 4 课 因式分解 〖知识点〗 因式分解定义,提取公因式、应用公式法、分组分解法、二次三项式的因式(十字相乘 法、求根)、因式分解一般步骤。 〖大纲要求〗 理解因式分解的概念,掌握提取公因式法、公式法、分组分解法等因式分解方法,掌握 利用二次方程求根公式分解二次二项式的方法,能把简单多项式分解因式。 〖考查重点与常见题型〗 考查因式分解能力,在中考试题中,因式分解出现的频率很高。重点考查的分式提取公 因式、应用公式法、分组分解法及它们的综合运用。习题类型以填空题为多,也有选择题和 解答题。 因式分解知识点 多项式的因式分解,就是把一个多项式化为几个整式的积.分解因式要进行到每一个因 式都不能再分解为止.分解因式的常用方法有: (1)提公因式法 如多项式 ),( cbamcmbmam  其中 m 叫做这个多项式各项的公因式, m 既可以是一个单项式,也可以是一个多项式. (2)运用公式法,即用 ))(( ,)(2 ),)(( 2233 222 22 babababa bababa bababa     写出结果. (3)十字相乘法 对于二次项系数为 l 的二次三项式 ,2 qpxx  寻找满足 ab=q,a+b=p 的 a,b,如有, 则 );)((2 bxaxqpxx  对于一般的二次三项式 ),0(2  acbxax 寻找满足 a1a2=a,c1c2=c,a1c2+a2c1=b 的 a1,a2,c1,c2,如有,则 ).)(( 2211 2 cxacxacbxax  (4)分组分解法:把各项适当分组,先使分解因式能分组进行,再使分解因式在各组之 间进行. 分组时要用到添括号:括号前面是“+”号,括到括号里的各项都不变符号;括号前面 是“-”号,括到括号里的各项都改变符号. (5)求根公式法:如果 ),0(02  acbxax 有两个根 X1,X2,那么 ).)(( 21 2 xxxxacbxax  考查题型: 1.下列因式分解中,正确的是( ) (A) 1- 1 4 x2= 1 4 (x + 2) (x- 2) (B)4x –2 x2 – 2 = - 2(x- 1)2 (C) ( x- y )3 –(y- x) = (x – y) (x – y + 1) ( x –y – 1) (D) x2 –y2 – x + y = ( x + y) (x – y – 1) 2.下列各等式(1) a2- b2 = (a + b) (a–b ),(2) x2–3x +2 = x(x–3) + 2 (3 ) 1 x2 –y2 - 1 ( x + y) (x – y ) ,(4 )x2 + 1 x2 -2-( x -1 x )2 从左到是因式分解的个数为( ) (A) 1 个 (B) 2 个 (C) 3 个 (D) 4 个 3.若 x2+mx+25 是一个完全平方式,则 m 的值是( ) (A) 20 (B) 10 (C) ± 20 (D) ±10 4.若 x2+mx+n 能分解成( x+2 ) (x – 5),则 m= ,n= ; 5.若二次三项式 2x2+x+5m 在实数范围内能因式分解,则 m= ; 6.若 x2+kx-6 有一个因式是(x-2),则 k 的值是 ; 7.把下列因式因式分解: (1)a3-a2-2a (2)4m2-9n2-4m+1 (3)3a2+bc-3ac-ab (4)9-x2+2xy-y2 8.在实数范围内因式分解: (1)2x2-3x-1 (2)-2x2+5xy+2y2 考点训练: 1.分解下列因式: (1).10a(x-y)2-5b(y-x) (2).an+1-4an+4an-1 (3).x3(2x-y)-2x+y (4).x(6x-1)-1 (5).2ax-10ay+5by+6x (6).1-a2-ab-1 4 b2 *(7).a4+4 (8).(x2+x)(x2+x-3)+2 (9).x5y-9xy5 (10).-4x2+3xy+2y2 (11).4a-a5 (12).2x2-4x+1 (13).4y2+4y-5 (14)3X2-7X+2 解题指导: 1.下列运算:(1) (a-3)2=a2-6a+9 (2) x-4=( x +2)( x -2) (3) ax2+a2xy+a=a(x2+ax) (4) 1 16 x2-1 4 x+1 4 =x2-4x+4=(x-2)2 其中是因式分解, 且运算正确的个数是( ) (A)1 (B)2 (C)3 (D)4 2.不论a为何值,代数式-a2+4a-5 值( ) (A)大于或等于 0 (B)0 (C)大于 0 (D)小于 0 3.若 x2+2(m-3)x+16 是一个完全平方式,则 m 的值是( ) (A)-5 (B)7 (C)-1 (D)7 或-1 4.(x2+y2)(x2-1+y2)-12=0,则 x2+y2 的值是 ; 5.分解下列因式: (1).8xy(x-y)-2(y-x)3 *(2).x6-y6 (3).x3+2xy-x-xy2 *(4).(x+y)(x+y-1)-12 (5).4ab-(1-a2)(1-b2) (6).-3m2-2m+4 *4。已知 a+b=1,求 a3+3ab+b3 的值 5.a、b、c为⊿ABC 三边,利用因式分解说明b2-a2+2ac-c2 的符号 6.0<a≤5,a为整数,若 2x2+3x+a能用十字相乘法分解因式,求符合条件的a 独立训练: 1.多项式 x2-y2, x2-2xy+y2, x3-y3 的公因式是 。 2.填上适当的数或式,使左边可分解为右边的结果: (1)9x2-( )2=(3x+ )( -1 5 y), (2).5x2+6xy-8y2=(x )( -4y). 3.矩形的面积为 6x2+13x+5 (x>0),其中一边长为 2x+1,则另为 。 4.把 a2-a-6 分解因式,正确的是( ) (A)a(a-1)-6 (B)(a-2)(a+3) (C)(a+2)(a-3) (D)(a-1)(a+6) 5.多项式 a2+4ab+2b2,a2-4ab+16b2,a2+a+1 4 ,9a2-12ab+4b2 中,能用完全平方公式分 解因式的有( ) (A) 1 个 (B) 2 个 (C) 3 个 (D) 4 个 6.设(x+y)(x+2+y)-15=0,则 x+y 的值是( ) (A)-5 或 3 (B) -3 或 5 (C)3 (D)5 7.关于的二次三项式 x2-4x+c 能分解成两个整系数的一次的积式,那么 c 可取下面四个 值中的( ) (A) -8 (B) -7 (C) -6 (D) -5 8.若 x2-mx+n=(x-4)(x+3) 则 m,n 的值为( ) (A) m=-1, n=-12 (B)m=-1,n=12 (C) m=1,n=-12 (D) m=1,n=12. 9.代数式 y2+my+25 4 是一个完全平方式,则 m 的值是 。 10.已知 2x2-3xy+y2=0(x,y 均不为零),则 x y + y x 的值为 。 11.分解因式: (1).x2(y-z)+81(z-y) (2).9m2-6m+2n-n2 *(3).ab(c2+d2)+cd(a2+b2) (4).a4-3a2-4 *(5).x4+4y4 *(6).a2+2ab+b2-2a-2b+1 12.实数范围内因式分解 (1)x2-2x-4 (2)4x2+8x-1 (3)2x2+4xy+y2 第 5 课 分式 知识点: 分式,分式的基本性质,最简分式,分式的运算,零指数,负整数,整数,整数指数幂 的运算 大纲要求: 了解分式的概念,会确定使分式有意义的分式中字母的取值范围。掌握分式的基本性质, 会约分,通分。会进行简单的分式的加减乘除乘方的运算。掌握指数指数幂的运算。 考查重点与常见题型: 1.考查整数指数幂的运算,零运算,有关习题经常出现在选择题中,如:下列运算正确的 是( ) (A)-40 =1 (B) (-2)-1= 1 2 (C) (-3m-n)2=9m-n (D)(a+b)-1=a-1+b-1 2.考查分式的化简求值。在中考题中,经常出现分式的计算就或化简求值,有关习题多为中 档的解答题。注意解答有关习题时,要按照试题的要求,先化简后求值,化简要认真仔细, 如: 化简并求值: x (x-y)2 . x3-y3 x2+xy+y2 +(2x+2 x-y –2),其中 x=cos30°,y=sin90° 知识要点 1.分式的有关概念 设 A、B 表示两个整式.如果 B 中含有字母,式子 B A 就叫做分式.注意分母 B 的值不能 为零,否则分式没有意义 分子与分母没有公因式的分式叫做最简分式.如果分子分母有公因式,要进行约分化简 2、分式的基本性质 ,MB MA B A   MB MA B A   (M 为不等于零的整式) 3.分式的运算 (分式的运算法则与分数的运算法则类似). bd bcad d c b a  (异分母相加,先通分); ; ; bc ad c d b a d c b a bd ac d c b a   .)( n n n b a b a  4.零指数 )0(10  aa 5.负整数指数 ).,0(1 为正整数paaa p p  注意正整数幂的运算性质 nnn mnnm nmnm nmnm baab aa aaaa aaa       )( ,)( ),0( , 可以推广到整数指数幂,也就是上述等式中的 m、 n 可以是 O 或负整数. 考查题型: 1. 下列运算正确的是( ) (A)-40 =1 (B) (-2)-1= 1 2 (C) (-3m-n)2=9m-n (D)(a+b)-1=a-1+b-1 2.化简并求值: x (x-y)2 . x3-y3 x2+xy+y2 +(2x+2 x-y –2),其中 x=cos30°,y=sin90° 3.a 3 、x-4 x 、x-y 2 、1 a 、 p Л+1 、3 2 a+b、3ab2c3 5 中分式有___ 4.当 x=-----------时, 分式 |x|-1 (x-3)(x+1) 的值为零; 5.当 x 取---------------值时,分式 x2-1 x2+2x-3 有意义; 6.已知 4 x2-1 = A x-1 + B x+1 是恒等式,则 A=___,B=___。 7.化简( x+2 x2-2x – x-1 x2-4x+4 )÷x-4 x 8.先化简后再求值:x-3 x2-1 ÷x2-2x-3 x2+2x+1 + 1 x+1 ,其中 x= 1 2 -1 9.已知 a a-b =2,求a3-4a2b-5ab2 a3-6a2b+5ab2 的值 考点训练: 1,分式 -3 x-2 当 x=----------- 时有意义,当 x=-----------时值为正。 2,分式 1 1- 1 1-x2 中的取值范围是( ) (A)x≠1 (B)x≠-1 (C)x≠0 (D)x≠±1 且 x≠0 3,当 x=-------------------时,分式 |x|-3 x2+4x+12 的值为零? 4,化简 (1)1- 1 x+1 + 2 1-x2 (2) a2+7a+10 a2-a+1 • a3+1 a2+4a+4 ÷a+1 a+2 (3) [a+(a- 1 1-a )• 2-a-a2 a2-a+1 ]÷(a-2)(a+1) (4)。已知 b(b-1)-a(2b-a)=-b+6,求a2+b2 2 –ab 的值 *(5).[(1+ 4 x-2 )(x-4+4 x )–3]÷ (4 x –1) *(6). 已知 x+1 x = 5 ,求 2x2 x4-x2+1 的值 *(7)若a+b=1,求证: a b3-1 - b a3-1 =2(b-a) a2b2+3 解题指导, 1.当 a=----- -时,分式 a2-1 a2-2a-3 无意义,当 a-=----- -时,这个分式的值为零. 2.写出下列各式中未知的分子或分母, (1) x-y 5y = (y-x)2 ( ) (2) -2x 1-2x =( ) 2x2-x 3.不改变分式的值,把分式 4 3 b+2 1 2 -2b2 的分子,分母各项的系数化为整数,且最高次项的系数均 为正整数,得-------------------------,分式 a2-1 -a2-a+2 约分的结果为____。 4.把分式 3x x+y 中的 x,y 都扩大两倍,那么分式的值( ) (A)扩大两倍 (B) 不变 (C) 缩小 (D) 缩小两倍 5.分式- 1 2x2 , 5x-1 4(m-n) , 2 n-m 的最简公分母为( ) (A) 4(m-n)(n-m)x2 (B) 1 4x2(m-n) (C)4x2(m-n)2 (D)4(m-n)x2 6.下列各式的变号中,正确的是 (A)x-y y-x = - y-x x-y ( B)x-y y-x2 =y-x y-x2 (C)-x-1 -y+1 =x-1 y+1 (D)-x-y y-x =- x+y y-x 7.若 x >y>0,则x+1 y+1 - y x 的结果是( ) (A) 0 (B)正数 (C) 负数 (D) 以上情况都有可能 8.化简下列各式: (1) 1 a-3 +a+1 6+2a - 6 a2-9 (2) (xy+y2)÷x2+2xy+y2 xy ·x+y y2 *(3) [1-(a- 1 1-a )2÷ a2-a+1 a2-2a+1 ]· 1 1-a (4) 若( 2 –1)a=1,求 a 1+1 a - 1 1+a +1 的值 (5) 已知 x2-5xy+6y2=0 求 x2+3xy 2y2 的值 独立训练 1.化简6-5x+x2 x2-16 ÷ x-3 4-x · x2+5x+4 4-x2 *2.当 a= 3 时,求分式(a2+6 a2-1 - a+1 a-1 +1) ÷ a3+8 a4+3a3+2a2 的值 *3.化简 a a+1 +1 1+ 3a2 a2-1 4。已知 1 a +1 b = 1 a+b 值,求a b +b a 的值 5.已知 m2-5m+1=o 求(1) m3+1 m3 (2)m-1 m 的值 *6。当 x=1998,y=1999 时, 求分式 x4-y4 x3+x2y+xy2+y3 的值 7.已知a+2b 5 =3b-c 3 =2c-a 7 ,求 c-2b 3a+2b 的值 * 8.化简a3-a2-a+1 1-2|a|+a2 *(9) x x2+x+1 =1 4 求 x2 x4+x2+1 的值。 *(10)设1 a +1 b + 1 c = 1 a+b+c ,求证:a、b、c三个数中必有两个数之和为零。 第 6 课 数的开方与二次根式 〖知识点〗 平方根、立方根、算术平方根、二次根式、二次根式性质、最简二次根式、 同类二次根式、二次根式运算、分母有理化 〖大纲要求〗 1.理解平方根、立方根、算术平方根的概念,会用根号表示数的平方根、立方根和算术 平方根。会求实数的平方根、算术平方根和立方根(包括利用计算器及查表); 2.了解二次根式、最简二次根式、同类二次根式的概念,会辨别最简二次根式和同类二 次根式。掌握二次根式的性质,会化简简单的二次根式,能根据指定字母的取值范围将二次 根式化简; 3.掌握二次根式的运算法则,能进行二次根式的加减乘除四则运算,会进行简单的分母 有理化。 内容分析 1.二次根式的有关概念 (1)二次根式 式子 )0( aa 叫做二次根式.注意被开方数只能是正数或 O. (2)最简二次根式 被开方数所含因数是整数,因式是整式,不含能开得尽方的因数或因式的二次根式,叫 做最简二次根式. (3)同类二次根式 化成最简二次根式后,被开方数相同的二次根式,叫做同类二次根式. 2.二次根式的性质 ).0;0( );0;0( );0( ),0(|| );0()( 2 2         ba b a b a babaab aa aaaa aaa 3.二次根式的运算 (1)二次根式的加减 二次根式相加减,先把各个二次根式化成最简二次根式,再把同类三次根式分别合并. (2)三次根式的乘法 二次根式相乘,等于各个因式的被开方数的积的算术平方根,即 ).0,0(  baabba 二次根式的和相乘,可参照多项式的乘法进行. 两个含有二次根式的代数式相乘,如果它们的积不含有二次根式,那么这两个三次根式 互为有理化因式. (3)二次根式的除法 二次根式相除,通常先写成分式的形式,然后分子、分母都乘以分母的有理化因式,把 分母的根号化去(或分子、分母约分).把分母的根号化去,叫做分母有理化. 〖考查重点与常见题型〗 1.考查平方根、算术平方根、立方根的概念。有关试题在试题中出现的频率很高,习题 类型多为选择题或填空题。 2.考查最简二次根式、同类二次根式概念。有关习题经常出现在选择题中。 3.考查二次根式的计算或化简求值,有关问题在中考题中出现的频率非常高,在选择题 和中档解答题中出现的较多。 考查题型 1.下列命题中,假命题是( ) (A)9 的算术平方根是 3 (B) 16的平方根是±2 (C)27 的立方根是±3 (D)立方根等于-1 的实数是-1 2.在二次根式 45, 2x3, 11, 5 4 , x 4 中,最简二次根式个数是( ) (A) 1 个 (B)2 个 (C)3 个 (D)4 个 (2)下列各组二次根式中,同类二次根式是( ) (A)1 3 6,3 2 (B)3 5, 15 (C)1 2 12, 1 3 (D) 8, 2 3 3. 化简并求值,a+ ab ab+b + ab-b a- ab ,其中 a=2+ 3,b=2- 3 4. 2+1 的倒数与 2- 3的相反数的和列式为 ,计算结果为 5.(-1 4 )2 的算术平方根是 ,27 的立方根是 , 4 9 的算术平 方根是 , 49 81 的平方根是 . 考点训练: 1.如果 x2=a,已知 x 求 a 的运算叫做 ,其中 a 叫做 x 的 ;已知 a 求 x 的运算 叫做 ,其中 x 叫做 a 的 。 2.(- 2 )2 的平方根是 ,9 的算术平方根是 , 是-64 的立方根。 3.当 a<0 时,化简∣a∣+ a2 + 3 a3 = 。 4.若 5.062 =2.249, 50.62 =7.114, x =0.2249,则 x 等于( ) (A)5.062 (B)0.5062 (C)0.005062 (D)0.05062 5.设 x 是实数,则(2x+3)(2x-5)+16 的算术平方根是( ) (A)2x-1 (B)1-2x (C)∣2x-1∣ (D)∣2x+1∣ 6.x 为实数,当 x 取何值时,下列各根式才有意义: (1) -3x-2 ( )(2) x2+5 ( )(3) 1 x2 ( ) (4) 1 3 1-x ( )(5) 1 1- x+2 ( )(6) x + -x ( ) 7.等式 3-x x+2 = 3-x x+2 成立的条件是( ) (A)-2-2 (D)x≤3 8.计算及化简: (1)(-7 2 7 )2 (2) ab2(c+1)2 (3) 0.01×64 0.36×324 (4)2a2 3b b3 a4-b2 a4 (b>1) (5) x x-3y x2y-6xy2+9y3 x (x>3y) (6)( 48 -6 0.5 )(4 3 + 18 )-(2 3 -3 2 )2 (7)已知方程 4x2-2ax+2a-3=0 无实数根, 化简 4a2-12a+9 +|a-6| 解题指导 1.下列命题:(1)任何数的平方根都有两个(2)如果一个数有立方根,那么它一定有平方 根(3)算术平方根一定是正数(4)非负数的立方根不一定是非负数,错误的个数为( ) (A)1 (B)2 (C)3 (D)4 2.已知 3 0.5 =0.794, 3 5 =1.710, 3 50 =3.684,则 3 5000 等于( ) (A)7.94 (B)17.10 (C)36.84 (D)79.4 3.当 1a) 7.计算:( 32 + 0.5 -2 1 3 )-( 1 8 -1 5 75 ) 8.已知 a= 3- 2 3+ 2 ,b= 3+ 2 3- 2 ,求 a2-5ab+b2 的值。 9.计算:9 45 ÷3 1 5 ×3 2 22 3 10.化简: 6 3 2-2 3 11.设 5+1 5-1 的整数部分为a,小数部分为b,求a2+1 2 ab+b2 的值。 独立训练 1. 2 - 3 的倒数是 ; 2 - 3 的绝对值是 。 2. 8 的有理化因式是 , x-y 的有理化因式是 。 3. 1 x- x-1 与 1 x-1+ x 的关系是 。 4.三角形三边 a=7 50 ,b=4 72 ,c=2 98 ,则周长是 。 5.直接写出答案: (1) 3 · 2 ÷ 30 = ,(2)4xy 2x = ,(3)( 3 -2)8( 3 +2)8= 。 6.如果 a - b 的相反数与 a + b 互为倒数,那么( ) (A)a、b 中必有一个为 0 (B)∣a∣=∣b∣(C)a=b+1 (D)b=a+1 7.如果 (2-x)2 + (x-3)2 =(x-2)+(3-x),那么 x 的取值范围是( ) (A)x≥3 (B)x≤2 (C)x>3 (D)2≤x≤3 8.把(a-b) - 1 a-b 化成最简二次根式,正确的结果是( ) (A) b-a (B) a-b (C)- b-a (D)- a-b 9.化简-3x x - 1 x + 4x3 的结果必为( ) (A)正数 (B)负数 (C)零 (D)不能确定 10.计算及化简: (1)(5 8 27 · 11 3 ·3 54 ) (2) 18 + 2 2-1 -4 1 2 -2( 2 +1)0 (3)(3x 2 x y -2 5 3 xy +1 3 xy 2 )÷x 2 x y (4) a a-b a2-ab a3-2a2b+ab2 (a>b) 11.已知x+3 x+2 = 1 3+ 2+1 ,求 x-3 2x-4 ÷( 5 x-2 -的值 x-2)。 12.先化简,再求值:( x+2 xy +y x + y + 1 x - y )+ x- y+1 x 其中 x=2 - 3 ,y=2 + 3 13.设 11-6 2 的整数部分为 m,小数部分为 n,求代数式 m+n+2 n 的值。 14.试求函数t=2- -3x2+12x-9 的最大值和最小值。 15.如果a+b+| c-1 -1|=4 a-2 +2 b+1 -4,那么a+2b-3c的值 第 7 课 整式方程 〖知识点〗 等式及基本性质、方程、方程的解、解方程、一元一次方程、一元二次方程、简单的高 次方程 〖大纲要求〗 1. 理解方程和一元一次方程、一元二次方程概念; 2. 理解等式的基本性质,能利用等式的基本性质进行方程的变形,掌握解一元一次方程的 一般步骤,能熟练地解一元一次方程; 3. 会推导一元二次方程的求根公式,理解公式法与用直接开平方法、配方法解一元二次方 程的关系,会选用适当的方法熟练地解一元二次方程; 4. 了解高次方程的概念,会用因式分解法或换元法解可化为一元一次方程和一元二次方程 的简单的高次方程; 5. 体验“未知”与“已知”的对立统一关系。 内容分析 1.方程的有关概念 含有未知数的等式叫做方程.使方程左右两边的值相等的未知数的值叫做方程的解(只含 有—个未知数的方程的解,也叫做根). 2.一次方程(组)的解法和应用 只含有一个未知数,并且未知数的次数是 1,系数不为零的方程,叫做一元一次方程. 解一元一次方程的一般步骤是去分母、去括号、移项、合并同类项和系数化成 1. 3.一元二次方程的解法 (!)直接开平方法 形如(mx+n)2=r(r≥o)的方程,两边开平方,即可转化为两个一元一次方程来解,这种 方法叫做直接开平方法. (2)把一元二次方程通过配方化成 (mx+n)2=r(r≥o) 的形式,再用直接开平方法解,这种方法叫做配方法. (3)公式法 通过配方法可以求得一元二次方程 ax2+bx+c=0(a≠0) 的求根公式: a acbbx 2 42  用求根公式解一元二次方程的方法叫做公式法. (4)因式分解法 如果一元二次方程 ax2+bx+c=0(a≠0)的左边可以分解为两个一次因式的积,那么根据两 个因式的积等于 O,这两个因式至少有一个为 O,原方程可转化为两个一元一次方程来解, 这种方法叫做因式分解法. 〖考查重点与常见题型〗 考查一元一次方程、一元二次方程及高次方程的解法,有关习题常出现在填空题和选择 题中。 考查题型 1.方程 x2 = x +1 的根是( ) (A)x = x+1 ( B) x = 1± 5 2 (C) x = ± x+1 (D) x = -1± 5 2 2.方程 2 x2 + x = 0 的解为( ) (A) x1 = 0 x 2=1 2 (B) x1 = 0 x 2= - 2 (C) x = - 1 2 (D) x1 = 0 x 2 = -1 2 3. p x2 – 3x + p2 – p= 0 是关于 x 的一元二次方程,则( ) (A) p=1 (B) p>0 (C)p≠0 (D) p 为任何实数 4.下列方程中,解为 x = 2 的是( ) (A)3x = x+3 (B)- x + 3 = 0 (C) 2 x = 6 (D) 5 x –2 = 8 5. 关于 x 的方程 x2- 3 m x + m2 – m = 0 的一个根为-1,那么 m 的值是( ) 6. 已知 2 x – 3 和 1 + 4x 互为相反数,则 x = 。 7.解下列方程: (1) X - 1 3 [ x- 1 3 (x – 9)] = 1 9 (x–9) (2) x2 – 12 x = 3 (配方法) (3)y3– 2 y2 = 5 y – 10 (4)3x2 – 5 x – 2 = 0 (5) x2 – 6x + 1=0 考点训练: 1. 关于 x 的一元二次方程(2-m)x2=m(3-x)-1 的二次项系数是 ,一 次项系数是 ,常数项是 ,对的限制是 。 2. 当 x = ______ 时, x - 1-x2 3 的值等于 1。 3. 方程 a x2 + b x + c = 0, 当 a ≠ 0, b2 – 4 a c ≥ 0 时,其实根 x = 4. X 的 20 % 减去 15 的差的一半等于 2 ,用方程表示_______________ 5. 将方程(2 X +1) (3 X – 2 ) = 3 (X2 – 2 ) 化成一元二次方程的一般形式得 _____________ 6.若方程 a - (7 – 5 x ) = 5 - x 的解是 x = - 1 2 ,则 a = 7.代数式 2k-1 3 与代数式 1 4 k +3 的值相等时,k 的值为( ) (A) 7 (B) 8 (C) 9 (D) 10 8.若 1 3 m + 1 与 2m-7 3 互为相反数,则 m 的值为( ) (A) 3 4 (B) 4 3 (C)- 3 4 (D)- 4 3 9.方程 a x2 + b x = 0 ( a ≠ 0 ) 的二根是( ) (A) X1 = X2 = 0(B)X1 = 0 ,X2 = -b a (C) X1 = 0, X2 = b a (D) X1 = a b , X2 = b a 10.解下列方程: (1) 2x-1 3 - x+0.1 0.6 = 2x+1 4 – 1 (2) 14.5 - 2(t-3) 5 = 15t 10 - 4t-28 6 (3) 2 x(5x – 2 )= x(7–5 x)+14 (4) 2 t2 –4 = 7 t (5) 3(2x – 1)2 = 75 (6) x3 + 8 x2 + 15 x = 0 (7) (x2 – x )2 – 4 (2 x2 – 2 x – 3 ) = 0 解题指导 1.k = 时,2 是关于 x 的方程 3│k│- 2 x = 6 x + 4 的解 2.方程 4 x2 – 9 = 0 的根是 ,方程 (x – a )2 = b (b > 0 ) 的根是 3.若 x2 + 3 x + 1 = 0 则 x + 1 x = 4.已知三角形的两边长分别是 1 和 2,第三边的数值是方程 2 x2 – 5 x +3 = 0 的根,则 三角形的周长为 . 5.k 为 时, 方程 (k2 – 3 k + 2 ) x2 + (k2 + 6 k – 7 ) x + 2 k + 1 = 0, 是关 于 X 的一元 二次方程; k 为 时, 这个方程是关于 X 的一元一次方程. 6.方程2-x 3 - x-1 4 = 5 的解是( ) (A) 5 (B) - 5 (C) 7 (D)- 7 7.若关于 x 的方程 2x – 4= 3m 和 x+2=m 有相同的根,则 m 的值是( ) (A) 10 (B) – 8 (C) – 10 (D) 8 8.把下列各式配方 (1) X2 - 1 2 X+ =(X - )2 (2) X2 - X+25=(x - )2 9 若 2x2 – 3xy – 20y2=0 y≠0 求x y = . 10. 解下列方程: (1) (x – 1 ) ( x + 3 ) – 2 ( x + 3 )2 + 3 ( x + 3 ) ( x – 3 ) = 0 (2) x3–2x2 +1=0 (3)(3 x2 –2x +1)( 3x2 –2 x –7) +12 = 0 独立训练 1.已知实数 a.b.c 满足 a2-3a+2 +│b+1│+(c + 3)2 = 0 求方程 ax2+bx+c=0 的根 2.已知关于 x 的一元二次方程 (a x + 1 ) ( x – a ) = a – 2 的各项系数之和等于 3, 求这时方程的解 3.解方程 (1) (2x – 3)2 = (3x – 2)2 (2) 4x+14 5 -x-5 2 = 2 3 x+2 (3) (1+ 2 )x2 –(3 + 2 )x+ =0 (4) 5m2 – 17m + 14=0 (5) (x2 +x+1)(x2 +x + 12)=42 (6) 2x2 + (3a-b)x –2a2+3ab- b2 =0 4.解关于 x 的方程 x2+x – 2+k(x2+2x)=0 (对 k 要讨论) 第 8 课 分式方程与二次根式方程 〖知识点〗 分式方程、二次根式的概念、解法思路、解法、增根 〖大纲要求〗 了解分式方程、二次根式方程的概念。掌握把简单的分式方程、二次根式方程转化为一 元一次方程、一元二次方程的一般方法,会用换元法解方程,会检验。 内容分析 1.分式方程的解法 (1)去分母法 用去分母法解分式方程的一般步骤是: (i)在方程的两边都乘以最简公分母,约去分母,化成整式方程; (ii)解这个整式方程; (iii)把整式方程的根代入最简公分母,看结果是不是零,使最简公分母不为零的根是 原方程的根,使最简公分母为零的根是增根,必须舍去. 在上述步骤中,去分母是关键,验根只需代入员简公分母. (2)换元法 用换元法解分式方程,也就是把适当的分式换成新的未知数,求出新的未知数后求出原 来的未知数. 2.二次根式方程的解法 (1)两边平方法 用两边平方法解无理方程的—般步骤是: (i)方程两边都平方,去掉根号,化成有理方程; (ii)解这个有理方程; (iii)把有理方程的根代入原方程进行检验,如果适合,就是原方程的根,如果不适合, 就是增根,必须舍去. 在上述步骤中,两边平方是关键,验根必须代入原方程进行. (2)换元法 用换元法解无理方程,就是把适当的根号下台有未知数的式子换成新的未知数,求出新 的未知数后再求原来的未知数. 〖考查重点与常见题型〗 考查换元法解分式方程和二次根式方程,有一部分只考查换元的能力,常出现 在选择 题中另一部分习题考查完整的解题能力,习题出现在中档解答题中。 考题类型 1.(1)用换元法解分式方程 3x x2-1 +x2-1 3x =3 时,设 3x x2-1 =y,原方程变形为( ) (A)y2-3y+1=0(B)y2+3y+1=0(C)y2+3y-1=0(D)y2-y+3=0 2.用换元法解方程 x2+8x+ x2+8x-11 =23,若设 y= x2+8x-11 ,则原方程可化为 ( ) (A)y2+y+12=0(B)y2+y-23=0(C)y2+y-12=0(D)y2+y-34=0 3.若解分式方程 2x x-1 -m+1 x2+x =x+1 x 产生增根,则 m 的值是( ) (A)-1 或-2 (B)-1 或 2 (C)1 或 2 (D)1 或-2 4.解方程4 x - 1 x-1 =1 时,需将方程两边都乘以同一个整式(各分母的最简公分母),约 去分母,所乘的这个整式为( ) (A)x-1 (B)x(x-1) (C)x (D)x+1 5.先阅读下面解方程 x+ x-2 =2 的过程,然后填空. 解:(第一步)将方程整理为 x-2+ x-2 =0;(第二步)设 y= x-2 ,原方程可化为 y2+y=0;(第三步)解这个方程的 y1=0,y2=-1(第四步)当 y=0 时, x-2 =0;解 得 x=2,当 y=-1 时, x-2 =-1,方程无解;(第五步)所以 x=2 是原方程的根以上 解题过程中,第二步用的方法是___,第四步中,能够判定方程 x-2 =-1 无解原根据 是__。上述解题过程不完整,缺少的一步是___。 考点训练: 1. 给出下列六个方程:1)x2-2x+2=0 2) x-2 =1-x 3) x-3 + x-2 =0 4) x+1 +2=0 5)1 x + 1 x-1 =0 6) 1 x-1 +1= x x-1 具中有实数解的方程有( ) (A)0 个 (B)1 个 (C)2 个 (D)多于 2 个 2. 方程 2x x2-4 -1= 1 x+2 的解是( ) (A)-1 (B)2 或-1 (C)-2 或 3 (D)3 3. 当分母解 x 的方程x-3 x-1 = m x-1 时产生增根,则 m 的值等于( ) (A)-2 (B)-1 (C)1. (D)2 4. 方程 2x-3 - x+1 =0 的解是_________。 5. 能使(x-5) x-7 =0 成立的 x 是______。 6. 关于 x 的方程 m(m-1)x+3 =2x-15 是根式方程,则 m 的取值范围是_____。 7. 解下列方程: (1) 12x+1 2x2-7x+5 - 3 1-x = 4 2x-5 (2) 3x x2-1 +x2-1 3x = 5 2 (3)x2+ 1 x2 -7 2 (x-1 x )+1=0 解题指导: 1.解下列方程: (1) x+2 =x (2) 2 x2-9 + x-2 x(x-3) = 1 x2+3x (3)x2+2x+2= 6 (x+1)2 (4) 3x+2 - x-8 =3 2 独立训练 1.方程 x(x2+1) =0 的解是_______. 方程 2x+3 =-x 的解是_______,方程 1 x-1 = 4 x+2 的解是___________ . 2.设 y= ____时,分式方程( x x-1 )2+5( x x-1 )+6=0 可转化为__________. 3.用换元法解方程 2x-3x2+4 3x2-2x+5 +1=0 可设 y =_________.从而把方程化为 _____________. 4.下列方程有实数解的是( ) (A) x+2 +5=4 (B) 3-x + x-3 =0 (C)x2-2x+4=0 (D) 2 x+1 + 3 x-1 = 6 x2-1 5.解下列方程. (1) 1 x-2 = x+2 x2-4 (2) x+4 x2+2x - 1 x+2 = 1 x +1 (3) a-x b+x =5-4(b+x) a-x (a+b≠0) (4) 2-x + 5-4x =2 (5) 2x2-4x-3 x2-2x-4 =10 (6)4(x2+ 1 x2 )-5(x-1 x )-14=0 (7)3x2+15x+2 3x2+15x+1 =2 (8) x+2 x-1 + x-1 x+2 = 5 2 6.若关于 x 的方程 x x-2 - m+1 x2+2 = x+1 x +1 产生增根,求 m 的值。 m 为何值时,关于 x 的方程 2 x-2 - mx x2-4 = 3 x+2 会产生增根。 7. 当 a 为何值时,方程x-1 x - 8x+a 2x(x-1) + x x-1 =0 只有一个实数根。 方程 x x+1 + x+1 x = - 4x+a x(x+1) 只有一个实数根,求 a 的值 8.当 m 为何值时,方程3 x + 6 x-1 - x+m x(x-1) = 0 有解 第 9 课 方程组 〖知识点〗 方程组、方程组的解、解方程组、二元一次方程(组)、三元一次方程(组)、二元二次方程 (组)、解方程组的基本思想、解方程组的常见方法。 〖大纲要求〗 了解方程组和它的解、解方程组等概念,灵活运用代入法、加减法解二元一次方程组,并会 解简单的三元一次方程组。掌握由一个二元一次方程和一个二元二次方程组成的方程组的解 法,掌握由一个二元二次方程和一个可以分解为两个二元一次方程的二元二次方程组成的方 程组的解法。 内容分析 1. 方程组的有关概念 含有两个未知数并且未知项的次数是 1 的方程叫做二元一次方程.两个二元—次方程合在一 起就组成了一个—。元一次方程组.二元一次方程组可化为      rnymx cbyax , (a,b,m、n 不全为零)的形式. 使方程组中的各个方程的左、右两边都相等的未知数的值,叫做方程组的解. 2.一次方程组的解法和应用 解二元(三元)一次方程组的一般方法是代入消元法和加减消元法. 3. 简单的二元二次方程组的解法 (1)可用代入法解一个二元二次方程和一个二元一次方程组成的方程组. (2)对于两个二元三次方程组成的方程组,如果其中一个可以分解因式,那么原方程组 可以转化为两个由一个二元二次方程和一个二元一次方程组成的方程组来解. 〖考查重点与常见题型〗 考查二元一次方程组、二元二次方程组的能力,有关试题多为解答题,也出现在选择题、填 空题中,近年的中考试题中出现了有关的阅读理解题。 考题类型 1.方程组     2 2 2 6x -5xy+y =0 1 y=x +6x+4 2  的解的个数( ) A.4 B.3 C.2 D.1 2.方程组 ax+by=4 bx+ay=5    的解是 x=2 y=1    ,则 a+b= 3.若方程组    2 y=mx+2 1 y +4x+1=2y 2  没有实数解,则实数 m 的取值范围是( ) A.m>1 B.m<-1 C.m<1 且 m≠0 D.m>-1 且 m≠0 4.阅读:解方程组     2 2 2 2 x -3xy+2y =0 1 x +y =10 2  解:由①的(x-y)(x-2y)=0 则 x-y=0 或 x-2y=0 (第一步) 因此,原方程组化为两个方程组 2 2 x-y=0 x +y =10    2 2 x-2y=0 x +y =10    分别解这两个方程组,得原方程组的解为 1 1 x = 5 y = 5   2 2 x =- 5 y =- 5   3 3 x =2 2 y = 2   4 4 x =-2 2 y =- 2   (第二步) 填空:第一步中,运用_______法将方程①化为两个二元一次方程,达到了_________的目的。 由第一步到第二步,将原方程组化为两个由一个二元一次方程和一个二元二次方程组成的方 程组,体现了_________的数学思想,第二步中,两个方程组都运用了_______法达到了 ________的目的,从而使方程组得以求解。 5.已知方程组     2x - (2k+1)y - 4=0 1 y=x - 2 2  (1) 求证不论 k 为何值时此方程总一定有实数解。 (2) 设等腰△ABC 的三边长分别为 a、b、c,其中 c=4,且 x=a y=a-2    , x=b y=b-2    是该方程的 两个解,求△ABC 的周长 6.解方程组 1+ 1=5 x+y=13 x y   解题指导 1.若 2 y=-1 x    是关于 x,y 的二元一次方程组 3x-by=7a+4 ax+by=2-b    的解, 求 4a+b2+(-a)2001 的值。 2.已知(3x-y-4)2+ 4x+y-3 =0 求 xy 的值。 3.若 2 5 x5m+2n+2y3 与 -3 4 x6y3m-2n-1 的和是单项式,求 m,n 的值。 4.在公式 s=v0t + 1 2 at2 中,当 t=1 时 s=13;当 t=2 时 s=42,求 t=3 时 s 的值。 5.解下列方程组 (1) 32x+y+z= - 2 4 x+2y+z = -2 x+y+2z = 3        (2) 2 2 2 2 x +y = 5 2x - 3xy-2y = 0    考点训练 1. 若 1 y=2 x    是方程组 ax+by=12 ay-bx= - 1    的解,求 a,b 的值。 2.已知方程 2m -1 n -8(m-2)x +(n+3)y =5 是二元一次方程,求 m,n 的值。 若 x = 1 2 时,求相应的 y 的值。 3.解方程组 (1) 1- = 7 6 2 5(x+y) - 2(x-y) - 1 = 0 x y x y   (2) 2 2 x+y=4 x - y = 8    4.方程组 2 2 kx-y-4=0 4x +9y +18y-18=0    中,k 为何值时此方程组只有一个实数解? 独立训练 1.如果方程组 2y =2x y=x+b    有两个相等的实数解,那么 b=___,这时方程组解为_______. 2. 方程组 (x+y)(x-y)=0 (x+2y-1)(x-2y+1)=0    的解是______________________. 3.方程组 1+ 2=5 x+y=14 x y   的解是_____________________ 4 . 当 m_______ 时 , 方 程 组 2 2 5x+my=1 mx+(m -1)y = - 4    是 关 于 x,y 的 二 元 二 次 方 程 组 , 当 m=0 时,这个方程组的解是________________。 5.已知方程 4x+5y=8,用含 x 的代数式表示 y 为____________________. 6.方程 x+2y=5 在自然数范围内的解是____________________. 7.已知关于 x,y 的方程组 x+y=5m x-y=9m    的解满足 2x-3y=9,则 m 的值是_________. 8.解下列方程组: (1) 2 2x -4y +x+3y-1=0 2x-y-1=0    (2) 2v+t 3 =3v-2t 8 =3 (3) x:y=3:2 y:z=5:4 x+y+z=66    (4) 2x-y=5 2 - =1y x x y   (5) 2x +y(y-2x)=9 (x+y)(x+y-3)=10    第 10 课 判别式与韦达定理 〖知识点〗 一元二次方程根的判别式、判别式与根的个数关系、判别式与根、韦达定理及其逆定理 〖大纲要求〗 1.掌握一元二次方程根的判别式,会判断常数系数一元二次方程根的情况。对含有字母 系数的由一元二次方程,会根据字母的取值范围判断根的情况,也会根据根的情况确定字母 的取值范围; 2.掌握韦达定理及其简单的应用; 3.会在实数范围内把二次三项式分解因式; 4.会应用一元二次方程的根的判别式和韦达定理分析解决一些简单的综合性问题。 内容分析 1.一元二次方程的根的判别式 一元二次方程 ax2+bx+c=0(a≠0)的根的判别式△=b2-4ac 当△>0 时,方程有两个不相等的实数根; 当△=0 时,方程有两个相等的实数根, 当△<0 时,方程没有实数根. 2.一元二次方程的根与系数的关系 (1)如果一元二次方程 ax2+bx+c=0(a≠0)的两个根是 x1,x2,那么 a bxx  21 , a cxx 21 (2)如果方程 x2+px+q=0 的两个根是 x1,x2,那么 x1+x2=-P, x1x2=q (3)以 x1,x2 为根的一元二次方程(二次项系数为 1)是 x2-(x1+x2)x+x1x2=0. 3.二次三项式的因式分解(公式法) 在分解二次三项式 ax2+bx+c 的因式时,如果可用公式求出方程 ax2+bx+c=0 的两个根是 x1,x2,那么 ax2+bx+c=a(x-x1)(x-x2). 〖考查重点与常见题型〗 1.利用根的判别式判别一元二次方程根的情况,有关试题出现在选择题或填空题中,如: 关于 x 的方程 ax2-2x+1=0 中,如果 a<0,那么梗的情况是( ) (A)有两个相等的实数根 (B)有两个不相等的实数根 (C)没有实数根 (D)不能确定 2.利用一元二次方程的根与系数的关系求有关两根的代数式的值,有关问题在中考试题 中出现的频率非常高,多为选择题或填空题,如: 设 x1,x2 是方程 2x2-6x+3=0 的两根,则 x1 2+x2 2 的值是( ) (A)15 (B)12 (C)6 (D)3 3.在中考试题中常出现有关根的判别式、根与系数关系的综合解答题。在近三年试题 中又出现了有关的开放探索型试题,考查了考生分析问题、解决问题的能力。 考查题型 1.关于 x 的方程 ax2-2x+1=0 中,如果 a<0,那么根的情况是( ) (A)有两个相等的实数根 (B)有两个不相等的实数根 (C)没有实数根 (D)不能确定 2.设 x1,x2 是方程 2x2-6x+3=0 的两根,则 x1 2+x2 2 的值是( ) (A)15 (B)12 (C)6 (D)3 3.下列方程中,有两个相等的实数根的是( ) (A) 2y2+5=6y(B)x2+5=2 5 x(C) 3 x2- 2 x+2=0(D)3x2-2 6 x+1=0 4.以方程 x2+2x-3=0 的两个根的和与积为两根的一元二次方程是( ) (A) y2+5y-6=0 (B)y2+5y+6=0 (C)y2-5y+6=0 (D)y2-5y-6=0 5.如果 x1,x2 是两个不相等实数,且满足 x1 2-2x1=1,x2 2-2x2=1, 那么 x1·x2 等于( ) (A)2 (B)-2 (C)1 (D)-1 6.如果一元二次方程 x2+4x+k2=0 有两个相等的实数根,那么 k= 7.如果关于 x 的方程 2x2-(4k+1)x+2 k2-1=0 有两个不相等的实数根,那么 k 的取值范 围是 8.已知 x1,x2 是方程 2x2-7x+4=0 的两根,则 x1+x2= ,x1·x2= ,(x1-x2) 2= 9.若关于 x 的方程(m2-2)x2-(m-2)x+1=0 的两个根互为倒数,则 m= 二、考点训练: 1、 不解方程,判别下列方程根的情况: (1)x2-x=5 (2)9x2-6 2 +2=0 (3)x2-x+2=0 2、 当 m= 时,方程 x2+mx+4=0 有两个相等的实数根; 当 m= 时,方程 mx2+4x+1=0 有两个不相等的实数根; 3、 已知关于 x 的方程 10x2-(m+3)x+m-7=0,若有一个根为 0,则 m= ,这时方程的 另一个根是 ;若两根之和为-3 5 ,则 m= ,这时方程的两个根为 . 4、 已知 3- 2 是方程 x2+mx+7=0 的一个根,求另一个根及 m 的值。 5、 求证:方程(m2+1)x2-2mx+(m2+4)=0 没有实数根。 6、 求作一个一元二次方程使它的两根分别是 1- 5 和 1+ 5 。 7、 设 x1,x2 是方程 2x2+4x-3=0 的两根,利用根与系数关系求下列各式的值: (1) (x1+1)(x2+1) (2)x2 x1 + x1 x2 (3)x1 2+ x1x2+2 x1 解题指导 1、 如果 x2-2(m+1)x+m2+5 是一个完全平方式,则 m= ; 2、 方程 2x(mx-4)=x2-6 没有实数根,则最小的整数 m= ; 3、 已知方程 2(x-1)(x-3m)=x(m-4)两根的和与两根的积相等,则 m= ; 4、 设关于 x 的方程 x2-6x+k=0 的两根是 m 和 n,且 3m+2n=20,则 k 值为 ; 5、 设方程 4x2-7x+3=0 的两根为 x1,x2,不解方程,求下列各式的值: (1) x1 2+x2 2 (2)x1-x2 (3) x1 + x2 *(4)x1x2 2+1 2 x1 *6.实数s、t分别满足方程 19s 2 +99s+1=0 和且 19+99t+t 2 =0 求代数式 st+4s+1 t 的值。 7.已知 a 是实数,且方程 x2+2ax+1=0 有两个不相等的实根,试判别方程 x2+2ax+1-1 2 (a2x2 -a2-1)=0 有无实根? 8.求证:不论 k 为何实数,关于 x 的式子(x-1)(x-2)-k2 都可以分解成两个一次因式的积。 9.实数 K 在什么范围取值时,方程kx2+2(k-1)x-(K-1)=0 有实数正根? 独立训练(一) 1、 不解方程,请判别下列方程根的情况; (1)2t2+3t-4=0, ; (2)16x2+9=24x, ; (3)5(u2+1)-7u=0, ; 2、 若方程 x2-(2m-1)x+m2+1=0 有实数根,则 m 的取值范围是 ; 3、 一元二次方程 x2+px+q=0 两个根分别是 2+ 3 和 2- 3 ,则 p= ,q= ; 4、 已知方程 3x2-19x+m=0 的一个根是 1,那么它的另一个根是 ,m= ; 5、 若方程 x2+mx-1=0 的两个实数根互为相反数,那么 m 的值是 ; 6、 m,n 是关于 x 的方程 x2-(2m-1)x+m2+1=0 的两个实数根,则代数式 mn= 。 7、 已知关于 x 的方程 x2-(k+1)x+k+2=0 的两根的平方和等于 6,求 k 的值; 8、 如果α和β是方程 2x2+3x-1=0 的两个根,利用根与系数关系,求作一个一元二次方程, 使它的两个根分别等于α+ 1 β 和β+ 1 α ; 9、 已知 a,b,c 是三角形的三边长,且方程(a2+b2+c2)x2+2(a+b+c)x+3=0 有两个相等的实数根, 求证:这个三角形是正三角形 10.取什么实数时,二次三项式 2x2-(4k+1)x+2k2-1 可因式分解. 11.已知关于 X 的一元二次方程m2x2+2(3-m)x+1=0 的两实数根为α,β,若s= 1 α + 1 β ,求s的取值范围。 独立训练(二) 1、 已知方程 x2-3x+1=0 的两个根为α,β,则α+β= , αβ= ; 2、 如果关于 x 的方程 x2-4x+m=0 与 x2-x-2m=0 有一个根相同,则 m 的值为 ; 3、 已知方程 2x2-3x+k=0 的两根之差为 21 2 ,则 k= ; 4、 若方程 x2+(a2-2)x-3=0 的两根是 1 和-3,则 a= ; 5、 方程 4x2-2(a-b)x-ab=0 的根的判别式的值是 ; 6、 若关于 x 的方程 x2+2(m-1)x+4m2=0 有两个实数根,且这两个根互为倒数,那么 m 的值 为 ; 7、 已知 p<0,q<0,则一元二次方程 x2+px+q=0 的根的情况是 ; 8、 以方程 x2-3x-1=0 的两个根的平方为根的一元二次方程是 ; 9、 设 x1,x2 是方程 2x2-6x+3=0 的两个根,求下列各式的值: (1)x1 2x2+x1x2 2 (2) 1 x1 -1 x2 10.m 取什么值时,方程 2x2-(4m+1)x+2m2-1=0 (1) 有两个不相等的实数根,(2)有两个相等的实数根,(3)没有实数根; 11.设方程 x2+px+q=0 两根之比为 1:2,根的判别式Δ=1,求 p,q 的值。 12.是否存在实数k,使关于x的方程 9x2-(4k-7)x-6k2=0 的两个实根 x1,x2,满足| x1 x2 |=3 2 ,如果存在,试求出所有满足条件的k的值,如果不存在,请说明理由。 第 11 课 应用题 〖知识点〗 列方程(组)解应用题的一般步骤、列方程(组)解应用题的核心、应用问题的主要类型 〖大纲要求〗能够列方程(组)解应用题 内容分析 列出方程(组)解应用题的一般步骤是: (i)弄清题意和题目中的已知数、未知数,用字母表示题目中的一个(或几个)未知数; (ii)找出能够表示应用题全部含义的一个(或几个)相等关系; (iii)根据找出的相等关系列出需要的代数式,从而列出方程(或方程组); (iv)解这个方程(或方程组),求出未知数的值; (v)写出答案(包括单位名称). 〖考查重点与常见题型〗 考查列方程(组)解应用题的能力,其中重点是列一元二次方程或列分式方程解应用题, 习题以工程问题、行程问题为主,近几年出现了一些经济问题,应引起注意 一、填空题 1.某商品标价为 165 元,若降价以九折出售(即优惠 10%),仍可获利 10%(相对于进货价), 则该商品的进货价是 2.甲、乙二人投资合办一个企业,并协议按照投资额的比例分配所得利润,已知甲与乙投资 额的比例为 3:4,首年的利润为 38500 元,则甲、乙二人可获得利润分别为 元 和 元 3.某公司 1996 年出口创收 135 万美元,1997 年、1998 年每年都比上一年增加 a%,那么, 1998 年这个公司出口创汇 万美元 4.某城市现有 42 万人口,计划一年后城镇人口增加 0.8%,农村人口增加 1.1%,这样全市 人口将增加 1%,求这个城市现有的城镇人口数与农村人口数,若设城镇现有人口数为 x 万, 农村现有人口 y 万,则所列方程组为 5.在农业生产上,需要用含盐 16%的盐水来选种,现有含盐 24%的盐水 200 千克,需要加 水多少千克? 解:设需要加水 x 千克根据题意,列方程为 ,解这个方程,得 答: . 6.某电视机厂 1994 年向国家上缴利税 400 万元,1996 年增加到 484 万元,则该厂两年上缴 的利税平均每年增长的百分率 7.某种商品的进货价每件为 x 元,零售价为每件 900 元,为了适应市场竞争,商店按零售价 的九折降价并让利 40 元销售,仍可获利 10%(相对于进价),则 x= 元 8.一个批发与零售兼营的文具店规定,凡是一次购买铅笔 301 支以上(包括 301 支),可以 按批发价付款;购买 300 支以下(包括 300 支)只能按零售价付款,现有学生小王来购买铅 笔,如果给学校初三年级学生每人买 1 支,则只能按零售价付款,需用(m2-1)元(m 为正整 数,且 m2-1>100);如果多买 60 支,则可以按批发价付款,同样需用(m2-1)元. (1)设这个学校初三年级共有 x 名学生,则(a)x 的取值范围应为 (b)铅笔的零售价每支应为 元,批发价每支应为 元 (用含 x,m 的代数式表示) (2)若按批发价每购 15 支比按零售价每购 15 少付款 1 元,试求这个学校初三年级共有多少 名学生,并确定 m 的值。 二.列方程解应用题 1. 某商店运进 120 台空调准备销售,由于开展了促销活动,每天比原计划多售出 4 台,结 果提前 5 天完成销售任务,原计划每天销售多少台? 2. 我省 1995 年初中毕业会考(中考)六科成绩合格的人数为 8 万人,1997 年上升到 9 万 人,求则两年平均增长的百分率(取 2 =1.41) 3. 甲、乙两队完成某项工作,甲单独完成比乙单独完成快 15 天,如果甲单独先工作 10 天, 再由乙单独工作 15 天,就可完成这项工作的2 3 ,求甲、乙两人单独完成这项工作各需多少 天? 4. 某校校长暑期将带领该校市级“三好学生”去北京旅游,甲旅行社说:“如果校长买全 票一张,则其余学生可享受半价优待”,乙旅行社说:“包括校长在内全部按全票价的 6 折优 惠(即按全票价的 60%收费),若全票为 240 元 (1)设学生数为 x,甲旅行社收费为 y 甲,乙旅行社收费为 y 乙,分别计算两家旅行社的收费 (建立表达式) (2)当学生数为多少时,两家旅行社的收费一样? (3)就学生数 x 讨论哪家旅行社更优惠? 5. 现有含盐 15%的盐水内 400 克,张老师要求将盐水质量分数变为 12%。某同学由于计 算失误,加进了 110 克的水,请你通过列方程计算说明这位同学加多了,并指出多加了多少 克的水? 6. 甲步行上午 6 时从 A 地出发于下午 5 时到达 B 地,乙骑自行车上午 10 时从 A 地出发, 于下午 3 时到达 B 地,问乙在什么时间追上甲的? 7. 中华中学为迎接香港回归,从 1994 年到 1997 年内师生共植树 1997 棵,已知该校 1994 年植树 342 棵,1995 年植树 500 棵,如果 1996 年和 1997 年植树棵数的年增长率相同,那 么该校 1997 年植树多少棵? 8. 要建一个面积为 150m2 的长方形养鸡场,为了节约材料,鸡场的一边靠着原有的一条墙, 墙长为 am,另三边用竹篱笆围成,如图,如果篱笆的长为 35m,(1)求鸡场的长与宽各为多 少?(2)题中墙的长度 a 对题目的解起着怎样的作用? 9. 永盛电子有限公司向工商银行申请了甲乙两种款,共 计 68 万元,每年需付出利息 8.42 万元,甲种贷款每年的 利率是 12%,乙种贷款每年的利率是 13%,求这两种贷 款的数额各是多少? 10.小明将勤工俭学挣得的 100 元钱按一年期存入少儿银 行,到期后取出 50 元用来购买学习用品,剩下的 50 元和应得的利息又全部按一年期存入。 若存款的年利率保持不变,这样到期后可得本金和利息共 66 元,求这种存款的年利率。 11.某公司向银行贷款 40 万元,用来生产某种新产品,已知该贷款的年利率为 15%(不计 复利,即还贷前每年息不重复计息),每个新产品的成本是 2.3 元,售价是 4 元,应纳税款 为销售额的 10%。如果每年生产该种产品 20 万个,并把所得利润(利润=销售额-成本- 应纳税款)用来归还贷款,问需几年后能一次还清? 12.某车间在规定时间内加工 130 个零件,加工了 40 个零件后,由于改进操作技术,每天比 原来计划多加工 10 个零件,结果总共用 5 天完成任务。求原计划每天加工多少个零件? 13.东西两车站相距 600 千米,甲车从西站、乙车从东站同时同速相向而行,相遇后,甲车 以原速,乙车以每小时比原速快 10 千米的速度继续行驶,结果,当乙车到达西站 1 小时后, 甲车也到达东站,求甲、乙两车相遇后的速度? 14.一个水池有甲、乙两个进水管,单独开放甲管注满水池比单独开放乙管少用 10 小时。如 果单独开放甲管 10 小时后,加入乙管,需要 6 小时可把水池注满。问单独开放一个水管, 各需多少小时才能把水池注满? 15.某商店 1995 年实现利税 40 万元(利税=销售金额-成本),1996 年由于在销售管理上 进行了一系列改革,销售金额增加到 154 万元,成本却下降到 90 万元,(1)这个商店利税 1996 年比 1995 年增长百分之几? (2)若这个商店 1996 年比 1995 年销售金额增长的百分数和成本下降的百分数相同,求这 个商店销售金额 1996 年比 1995 年增长百分之几? 16.甲、乙两辆汽车同时从 A 地出发,经 C 地去 B 地,已知 C 地离 B 地 180 千米,出发时甲 车每小时比乙车多行驶 5 千米。因此,乙车经过 C 地比甲车晚半小时,为赶上甲车,乙车从 C 地起将车速每小时增加 10 千米,结果两从同时到达 B 地,求(1)甲、乙两从出发时的速 度;(2)A、B 两地间的距离. 17.某项工程,甲、乙两人合作,8 天可以完成,需费用 3520 元;若甲单独做 6 天后,剩下 A B C D E F 的工程由乙独做,乙还需 12 天才能完成,这样需要费用 3480 元,问:(1)甲、乙两人单独 完成此项工程,各需多少天? (2)甲、乙两人单独完成此项工程,各需费用多少元? 18.某河的水流速度为每小时 2 千米,A、B 两地相距 36 千米,一动力橡皮船从 A 地出发, 逆流而上去 B 地,出航后 1 小时,机器发生故障,橡皮船随水向下漂移,30 分钟后机器修 复,继续向 B 地开去,但船速比修复前每小时慢了 1 千米,到达 B 地比预定时间迟了 54 分 钟,求橡皮船在静水中起初的速度. 第 12 课 不等式 〖知识点〗 不等式概念,不等式基本性质,不等式的解集,解不等式,不等式组,不等式组的解集,解 不等式组,一元一次不等式,一元一次不等式组。 大纲要求 1.理解不等式,不等式的解等概念,会在数轴上表示不等式的解; 2.理解不等式的基本性质,会应用不等式的基本性质进行简单的不等式变形,会解一元 一次不等式; 3.理解一元一次不等式组和它的解的概念,会解一元一次不等式组; 4.能应用一元一次不等式(组)的知识分析和解决简单的数学问题和实际问题。 内容分析 一元一次不等式、一元一次不等式组的解法 (1)只含有一个未知数,并且未知数的次数是 1,系数不为零的不等式,叫做一元一次 不等式. 解一元一次不等式的一般步骤是去分母、去括号、移项、合并同类项和系数化成 1.要 特别注意,不等式的两边都乘以(或除以)同一个负数,要改变不等号的方向. (2)解一元一次不等式组的一般步骤是: (i)先求出这个不等式组中各个一元一次不等式的解集; (ii)再利用数轴确定各个解集的公共部分,即求出了这个一元一次不等式组的解集. 考查重点与常见题型 考查解一元一次不等式(组)的能力,有关试题多为解答题,也出现在选择题,填空题 中。 考查题型 1.下列式子中是一元一次不等式的是( ) (A)-2>-5 (B)x2>4 (C)xy>0 (D)x 2 –x< -1 2.下列说法正确的是( ) (A) 不等式两边都乘以同一个数,不等号的方向不变; (B) 不等式两边都乘以同一个不为零的数,不等号的方向不变; (C) 不等式两边都乘以同一个非负数,不等号的方向不变; (D) 不等式两边都乘以(或除以)同一个正数,不等号的方向不变; 3.对不等式的两边进行变形,使不等号方向改变,可采取的变形方法是( ) (A)加上同一个负数 (B)乘以同一个小于零的数 (C)除以同一个不为零的数 (D) 乘以同一个非正数 4.在数轴上表示不等式组 x>-2 x 1    的解,其中正确的是( ) 5.下列不等式组中,无解的是( ) (A) 2x+3<0 3x+2>0    (B) 3x+2<0 2x+3>0    (C) 3x+2>0 2x+3>0    (D) 2x+3<0 3x+2<0    6.若 a0 (B)a+b<0 (C)ac -b 7.解下列不等式(组) (1)x-x-3 8 <2 + 3(x+1) 2 (2) 2x-1b 用”>”或”<”连接下列各式; (1)a-3 ---- b-3, (2)2a ----- 2b, (3)- a 3 ----- -b 3 (4)4a-3 ---- 4b-3 (5)a-b --- 0 2.判断题: (1) 若 a>b 则1 a < 1 b ( ) (2) 若 a>b 则|a|>|b| ( ) (3)若 ac >bc 则 a>b ( ) (4)若a c2 >b c2 则 a>b ( ) 3.a,b 是已知数,当 a>0 时,不等式 ax+b<0 的解集为------------, 当 a<0 不等式 ax+b<0 的解集 为---------------- 4.已知正整数 x 满足x-2 3 <0 ,则代数式(x-2)1999 - 7 x 的值是----------------. 5.解不等式 x-3x-2 4 ≥2(1+x) 3 -1,将解集在数轴上表示出来,且写出它的正整数解 6.解不等式组 x+1 x+21- >2 - 2 3 x(x-1) <(x+3)(x-3)   7. x 为何值时,代数式x 2 -3(x+4)的值是:(1)非负数(2)不大于零 8.已知三角形三边长分别为 3,(1-2a),8,试求a的取值范围。 解题指导: 1.解不等式 1-7x-1 8 >3x-2 4 ,并说明每一步的理由。 2.比较 x2-4x-1 与 x2-6x+3 的大小。 3.已知不等式 5(x-2)+8 < 6(x-1)+7 的最小整数解为方程 2x-ax=3 的解,求代数式 4a-14 a 的值。 4.求不等式组 3x-10 < 0 16 -1< 6x3   的整数解 5.已 知 方 程 组 x+y=3a+9 x-y=5a+1    的 解 为 正 数 , 求 ( 1 ) a 的 取 值 范 围 。 (2)化简|4a+5|-|a-4| *6.a、b为任意实数。解关于x的不等式a(x+b2)>b(x+a2) 独立训练: 1.用不等式表示:x 的2 3 与 5 的差小于 1 为________ 2. 不 等 式 5x - 17 ≤ 0 的 正 整 数 解 是 -------------_; 不 等 式 组 -4-1    的解集是-------------- 3.代数式 1-x-2 2 的值不大于1+3x 3 的值,那么的取值范围是_____________. 4.不等式组 2x+3>5 3x-2<4    的解集在数轴上的表示是( ) 5.如果 00 (B)a<0 (C)a<2 (D)a>2 7.已知不等式组 6x-1 3x-3 2 1 1<3 2 x x     的整数解满足方程 3(x+a)-5a= -2,求代数式 633(a2+2 a ) 的值。 8.解不等式-1≤2x+1 3 < 4 9.不等式 2x-3a<7b 6b-3x<5a    组的解 5 5 2 第 13 课 坐标系与函数 〖知识点〗 平面直角坐标系、常量与变量、函数与自变量、函数表示方法 〖大纲要求〗 1.了解平面直角坐标系的有关概念,会画直角坐标系,能由点的坐标系确定点的位置, 由点的位置确定点的坐标; 2.理解常量和变量的意义,了解函数的一般概念,会用解析法表示简单函数; 3.理解自变量的取值范围和函数值的意义,会用描点法画出函数的图像。 内容分析 1.平面直角坐标系的初步知识 在平面内画两条互相垂直的数轴,就组成平面直角坐标系,水平的数轴叫做 x 轴或横轴 (正方向向右),铅直的数轴叫做 y 轴或纵轴(正方向向上),两轴交点 O 是原点.这个平面叫 做坐标平面. x 轴和 y 把坐标平面分成四个象限(每个象限都不包括坐标轴上的点),要注意象限的编 号顺序及各象限内点的坐标的符号: 由坐标平面内一点向 x 轴作垂线,垂足在 x 轴上的坐标叫做这个点的横坐标,由这个点 向 y 轴作垂线,垂足在 y 轴上的坐标叫做这个点的纵坐标,这个点的横坐标、纵坐标合在一 起叫做这个点的坐标(横坐标在前,纵坐标在后).一个点的坐标是一对有序实数,对于坐 标平面内任意一点,都有唯一一对有序实数和它对应,对于任意一对有序实数,在坐标平面 都有一点和它对应,也就是说,坐标平面内的点与有序实数对是一一对应的. 2.函数 设在一个变化过程中有两个变量 x 与 y,如果对于 x 的每一个值, y 都有唯一的值与它 对应,那么就说 x 是自变量, y 是 x 的函数. 用数学式子表示函数的方法叫做解析法.在用解析式表示函数时,要考虑自变量的取值 范围必须使解析式有意义.遇到实际问题,还必须使实际问题有意义. 当自变量在取值范围内取一个值时,函数的对应值叫做自变量取这个值时的函数值. 3.函数的图象 把自变量的一个值和自变量取这个值时的函数值分别作为点的横坐标和纵坐标,可以在 坐标平面内描出一个点,所有这些点组成的图形,就是这个函数的图象.也就是说函数图象 上的点的坐标都满足函数的解析式,以满足函数解析式的自变量值和与它对应的函数值为坐 标的点都在函数图象上. 知道函数的解析式,一般用描点法按下列步骤画出函数的图象: (i)列表.在自变量的取值范围内取一些值,算出对应的函数值,列成表. (ii)描点.把表中自变量的值和与它相应的函数值分别作为横坐标与纵坐标,在坐标平 面内描出相应的点. (iii)连线.按照自变量由小到大的顺序、用平滑的曲线把所描各点连结起来. 〖考查重点与常见题型〗 1.考查各象限内点的符号,有关试题常出选择题,如: 若点 P(a,b)在第四象限,则点 M(b-a,a-b)在( ) (A) 第一象限 (B)第二象限 (C)第三象限 (D)第四象限 2.考查对称点的坐标,有关试题在中考试卷中经常出现,习题类型多为填空题或选择题, 如:点 P(-1,-3)关于 y 轴对称的点的坐标是( ) (A)(-1,3) (B)(1,3) (C)(3,-1) (D)(1,-3) 3.考查自变量的取值范围,有关试题出现的频率很高,重点考查的是含有二次根式的函数式 中自变量的取值范围,题型多为填空题,如:函数 y= 2x-3的自变量 x 的取值范围是 4.函数自变量的取值范围: (1)函数 y= 1 x-1 中自变量 x 的取值范围是 (2)函数 y= x+2+ 5-x中自变量 x 的取值范围是 (3)函数 y= x-2 (2-x)2-1 中自变量 x 的取值范围是 5.已知点 P(a,b),a·b>0,a+b<0,则点 P 在( ) (A) 第一象限(B)第二象限 (C)第三象限 (D)第四象限 6.在直角坐标系中,点 P(-1,-1 2 )关于 x 轴对称的点的坐标是( ) (A)(-1,-1 2 )(B)(1,-1 2 )(C)(1,1 2 )(D)(-1,1 2 ) 7.已知点 P(x,y)的坐标满足方程|x+1|+ y-2 =0,则点 P 在( ) (A) 第一象限(B)第二象限 (C)第三象限 (D)第四象限 考点训练: 1.点A(x,y)是平面直角坐标系中的一点,若xy<0,则点A在 象限;若x=0则点A在 ; 若 x<0,y≠0 则点 A 在 ; 若 xy>0,且 x=y, 则点 A 在 2.已知点 A(a,b), B(a,-b), 那么点 A,B 关于 对称,直线 AB 平行于 轴 3.点 P(-4,-7)到 x 轴的距离为 ,到 y 轴的距离为 ,到原点距离为 4.已知 P 是第二象限内坐标轴夹角平分线上一点,点 P 到原点距离为 4,那么点 P 坐标为 5、某音乐厅有 20 排座位,第一排有 18 个座位,后面每排比前一排多一个座位,每排座位 数 m 与这排的排数 n 的函数关系是 ,自变量 n 的取值范围是 6、求下列函数中自变量的取值范围: (1)y=-x2+1 2 x+5 ( ) (2)y=x2--4 x+2 ( ) (3)y= 1 3 2x+1 ( ) (4)y=-- 3x--1 ∣x∣--2 ( ) 7、下列函数中(1)y=x∣,(2)y=( x )2,(3)y=x2 x ,(4)y= x2 ,(5)y= 3 x3 中与函数 y=x 相同的 函数个数是( ) (A)0 个 (B)1 个 (C)2 个 (D)3 个 8、如图,四边形 EFGH 是△ABC 的内接正方形,BC=a, 试写出正方形的边长 y 的与△ABC 高 AD=x 的函数关系式。 9、正方形 ABCD,边长 AB=4,顶点 A 与原点重合,点 B 在第一象限且 OB 与x轴正方向成 30°,点 D 在第二象限,求正方形的四个顶点坐标。 解题指导 1.点 P(x,y)在第二象限,且│x│= 2 , │y│= 3 ,则点 P 的坐标 是 ,点 P 到原点 O 的距离 OP= . 2.已知点 P(x,4), Q(--3,y)。若 P,Q 关于 y 轴对称,则 x= , y= ; 若 P,Q 关于 x 轴对称,则 x= , y= ;若 P,Q 关于原点 O 对称, 则 x= , y= 。 3.以 A(0,2), -4,0), C(3,0)为三个顶点画三角形,则 S△ABC= 。 4.依此连结 A(-6,-1), B(-3,-4), C(2,1), D(-1,4)四点,则四边形 ABCD 是 形。 5.当 x=- 2 时,函数 y=2x--1 x+1 的值是 ;函数 y=x2+3x+4 的值为 2,则自变量 x= 6.函数 y= --x x--1 中自变量 x 的取值范围是 . 7.用含 x 的代数式表示 y, (x+1)(2y-1)=1, 8.函数 y= 4x+1 与 y=2x-1 的值相等,则 x= . 9.等腰三角形的底角的度数为 x,顶角的度数为 y,写出以 x 表示 y 的函数关系 式 ,并指出自变量 x 的取值范围 。 10.多边形的内角和 a 与边数 n(n≥3)的关系式是 ;多边形的对角线条数 m 与 边数 n(n≥3)的关系式是 11.某公司的职员按工资的高低交纳公积金,办法如下: 年工资 公积金 3000 元以下 不交纳 3000~5000 元 交纳超过 3000 元部分的 10% 5000~7000 元 3000~5000 元部分交纳 10%,超过 5000 元部分交纳 20% 该公司每年职工工资为 x 千元,交纳公积金后实得数为 y 千元,根据此表列出 y 与 x 之间的 函数关系式。 独立训练 1.已知 A(- 3 , 2 )与点 B 关于 y 轴对称,则点 B 的坐标是 ,与点 B 关于原点 对称的点 C 的坐标是 ,这时点 A 与点 C 关于 对称。 2.在函数 y= x x2--1 中,自变量 x 的取值范围是 . 3.在有序实数对(3,-20), (-4,1), (2 3 ,3), (5, 5 2 )中,在函数 y=1 2 x+3 的图象上的点 有 个 4.若点 M(a,b)在第二象限,则点 N(a-1,b)在第 象限. 5.所有横坐标为零的点都在 上,所有纵坐标为零的点都 上 6.若点 P(a,--3)在第三象限内两条坐标轴夹角的平分线上,则 a= 7.菱形边长为 6,一个内角为 120°,它的对角线与两坐标轴重合,则菱形四个顶点的坐标 分别是 8.写出下列函数关系式:(1)某城市共有绿化面积 108m2,这个城市人均占有绿化面积 y(m2) 与人数 a 的函数关系式 ,其中自变量是 ; (2)地面气温是 25℃,如果每升高 1 千米,气温下降 5℃,则气温 t℃与高度 h 千米的函数关系式是 ,其中自变量是 9.若 A(a,b), B(b,a)表示同一点,则这一点在 10.求下列函数自变量 x 的取值范围: (1)y=3x2-5x+1 ( ) (2)y= 3 x-1 x-2 ( ) (3)y= 3 2+ x-1 ( )(4)y= 2x-3 + 9-3x ( ) 11.某市体育馆原有长 100m, 宽 60m 的矩形游泳池,准备扩建成周长为 600m 的较大矩形游 泳池。假设长增加 x 米,宽增加 y 米,扩建后面积为 S 平方米。(1)将 y 表示成 x 的函数; (2)将 S 表示成 x 的函数。 12.如图,在平行四边形 ABCD 中,AB=8, AD=6, E 是 AB 边上一动点,记 AE=x,DE 的延长 线 CB 的延长线于 F。设 CF=y,求 y 与 x 的函数关系式。 第 14 课 正比例、反比例、一次函数 〖知识点〗 正比例函数及其图像、一次函数及其图像、反比例函数及其图像 〖大纲要求〗 1.理解正比例函数、一次函数、反比例函数的概念; 2.理解正比例函数、一次函数、反比例函数的性质; 3.会画出它们的图像; 4.会用待定系数法求正比例、反比例函数、一次函数的解析式 内容分析 1、一次函数 (1)一次函数及其图象 如果 y=kx+b(K,b 是常数,K≠0),那么,Y 叫做 X 的一次函数。 特别地,如果 y=kx(k 是常数,K≠0),那么,y 叫做 x 的正比例函数 一次函数的图象是直线,画一次函数的图象,只要先描出两点,再连成直线 (2)一次函数的性质 当 k>0 时 y 随 x 的增大而增大,当 k<0 时,y 随 x 的增大而减小。 2、反比例函数 (1) 反比例函数及其图象 如果 )0,(  kkx ky 是常数 ,那么,y 是 x 的反比例函数。 反比例函数的图象是双曲线,它有两个分支,可用描点法画出反比例函数的图象 (2)反比例函数的性质 当 K>0 时,图象的两个分支分别在一、二、三象限内,在每个象限内, y 随 x 的增大 而减小; 当 K<0 时,图象的两个分支分别在二、四象限内,在每个象限内,y 随 x 的增大而增大。 3.待定系数法 先设出式子中的未知数,再根据条件求出未知系数,从而写出这个式子的方法叫做待定 系数法可用待定系数法求一次函数、二次函数和反比例函数的解析式 〖考查重点与常见题型〗 1. 考查正比例函数、反比例函数、一次函数的定义、性质,有关试题常出现在选择题 中 2. 综合考查正比例、反比例、一次函数的图像,习题的特点是在同一直角坐标系内考 查两个函数的图像,试题类型为选择题 3. 考查用待定系数法求正比例、反比例、一次函数的解析式,有关习题出现的频率很 高,习题类型有中档解答题和选拔性的综合题 4. 利用函数解决实际问题,并求最值,这是近三年中考应用题的新特点。 考查题型 1.若函数 y=(m+1)xm 2 +3m+1 是反比例函数,则 m 的值是( ) (A) m=-1 (B)m=-2(C)m=2 或 m=1 (D)m=-2 或 m=-1 2.已知一次函数 y=(m+2)x+(1-m),若 y 随 x 的增大而减小,且该函数的图像与 x 轴的交点在原点的右侧,则 m 的取值范围是( ) (A)m>-2 (B)m<1 (C)-20 时,y 随 x 的增大而 7.如果直线 y=2x+m 不经过第二象限,那么实数 m 的取值范围是 8.若双曲线 y=(m-1)x-1 在第二、四象限,则 m 的取值范围是 9.已知直线 y=3 4 x+b被两坐标轴截取的线段长为 5,求此直线函数解析式。 10.已知一次函数 y=kx+2b+3 的图象经过点(-1,-3),k是方程 m2-3m=10 的一个根,且 Y 随x的增大而增大,求这个一次函数解析式。 考点训练: 1. y= x 的图象是一条过原点及点(-3,3 2 )的直线 2.一次函数 y=kx+b 的图象经过 P(1,0) 和 Q(0,1)两点,则 k= ,b= . 3.正比例函数的图象与直线 y= -2 3 x+4 平行,则该正比例函数的解析式为 , 该正比例函数 y 随 x 的增大而 . 4.已知 y-2 与 x 成正比例,且 x=2 时,y=4,则 y 与 x 之间的函数关系是 ,若点 (m,2m+7), 在这个函数的图象上,则 m = 5. 函数 y=(m-4)xm2-5m-5 的图象是过一、三象限的一条直线,则 m = 6.函数 y=k x (k≠0)的图象经过点( 2 ,3),则 k= ,当 x>0 时,y 随着 x 的增大而 7.如果一次函数 y=kx+b 和反比例函数 y=k x 的图象都经过(-2,1)点,则 b 的值是 8.已知一次函数 y=kx+b 的 y 随 x 的增大而减小,那么它的图象必经过 象限。 9.已知函数 y= -2x-6。(1)求当 x= -4 时,y 的值,当 y= -2 时,x 的值。 (2)画出函数图象; (3)求出函数图象与坐标轴的两个交点之间的距离; (4)如果 y 的取值范围-4≤y≤2,求 x 的取值范围. 10.已知 z 与 y- 3 成正比例,x 与 6 z 成反比例,(1)证明:y 是 x 的一次函数;(2)如果 这个一次函数的图象经过点(-2,3 3 ),并且与 x、y 轴分别交于 A、B 两点。求两 点的坐 标。 *11.已知函数 y=k x 的图象上有一点 P(m,n),且m,n关于t的方程t2-4at+4 a2-6a-8=0 的两个实数根,其中a是使方程有实数根的最小整数,求函数 y=k x 的解析 式, 解题指导 1.函数 y= - 3 2 x 的图象是一条过原点(0,0)及点(2, )的直线,这条直线经过第 象限,y 随的增大而 2.已知一次函数 y= - 1 2 x+2,当 x= 时,y=0;当 x 时 y>0; 当 x 时 y<0. 3.若一次函数 y1=kx-b 图象经过第一、三、四象限,则一次函数 y2=bx+k 的图象经过第 象 限。 4.直线 y1=k1x+b1 和直线 y2=k2x+b2 相交于 y 轴上同一点的条件是 ;这两直线平行的 条件是 5.过点(0,2)且与直线 y= - x 平行的直线是 。 6.y 与 3x+2 成正比例,比例系数是 4,则 y 与 x 的函数关系式是 。 7.等腰三角形的周长为 30cm,它的腰长为 ycm 与底长 xcm 的函数关系式是 。 8.y= x -1 的图象是一条过点(4 5 ,- 3 4 )的双曲线,在它的图象所在的每一个象限内, y 随 x 的增大而 。 9.把直线 y= -3 2 x -2 向上平移 2 个单位,得到直线 , 把直线 y= - 3 2 x -2 向 平移 个单位,得到直线 y= - 3 2 (x+4) 10.写出满足下表的一个一次函数的关系式 x - 1 2 5 y 7.5 6 4.5 11.直线 y=kx+b 经过点(0,3),且与两坐标轴构成的直角三角形的面积是 6,求其解析式。 12.已知反比例函数 y=k x (k>0)的图象上的一点 P,它到原点 O 的距离 OP=2 5 ,PQ 垂直于 y 轴,垂足为 Q.若△OPQ 的面积为 4 平方单位,求:(1)点 P 的坐标;(2)这个反比例函数的 解析式. 独立训练(一): 1.函数 y= - 2 x 是 函数,这个函数的图象位于第 象限。 2.对函数 y= - 5 3x 当 x>0 时,y 随 x 的增大而 。 3.反比例函数 y=k x 的图象上有一点 P,它的横坐标 m 与纵坐标 n 是方程 t2-4t-2=0 的两个 根,则 k= 4.如图,P 为反比例函数 y=k x 的图象上的点,过 P 分别向 x 轴和 y 轴引垂线,它们与两条坐标轴围成的矩形面积为 2, 这个反比例函数解析式为 。 5.反比例函数 y=(a-3)x 2a -2a-4 的函数值是 4 时,它的自变量 x 的值是 。 6.一次函数 y=kx+b 与反比例函数 y=2 x 的图象的两个交点的横坐标为1 2 和 -1,则一次函数 y= 7.一次函数 y=kx+b 过点(-2,5),且它的图象与 y 轴的交点和直线 y=-1 2 x+3 与 y 轴的 交点关于 x 轴对称,那么一次函数的解析式是 8.如图,在矩形 ABCD 中,已知 AB=2 3 ,BD=6,对角线 AC 和 BD 相交于 O,以 O 为原点分别以平行于 AB 和 AD 的直线为 轴和轴建立平面直角坐标系,则对角线 AC 和 BD 的函 数表达式分别为 。 9.求直线 y=3x+10,y= -2x-5 与 y 轴所围成的三角形的面积。 10.如图,一次函数 y=k1x+b 的图象过一、三、四象限,且 与双曲线 y=k2 x 的图象交于 A、B 两点,与 y 轴交于 C 点,且 A(x1,y1)是∠XOA 终边上一点。 (1) tg∠XOA=1 5 ,原点到 A 点的距离为 26 ,求 A 点的坐标; (2)在(1)的条件下,若 S△AOC=b2-6,求一次函数的解析式。 独立训练(二): 1. 如图,A、B 是函数 y=1 x 的图象上关于原点 O 对称的任意两点,AC 平行于 y 轴,BC 平行于 x 轴,△ABC 的面积 S,则( ) (A)S=1 (B) 12 2.函数 y=k1x+b(k1b<0)与 y=k2 x (k2<0)在同一坐标系中的图象大致是( ) 3.在边长为 2 的正方形 ABCD 的边 BC 上,有一点 P 从 B 点运动到 C 点,设 PB=x,图形 APCD 的面积为 y, 写出 y 与自变量 x 的函数关系式,并且在直角坐标系 中画出它的图象 4.已知 y=y1+y2,y1 与 x2 成正比例,y2 与 x 成反比例,并且当 x=1 时,y=1,当 x=3 时,y=-17, 求 x=-1 时,y 的值 5.如图,在 y= 8 x (x>0)反比例函数的图象上有不重合的两点 A、B,且 A 点的纵坐标是 2,B 点的横坐标为 2,BB1 和 AA1 都垂直于轴,垂足分别为 B1 和 A1,(1)求 A 点横坐标; (2)求 S△ 1OBB (3)当 OB=2 5 时,求 S△OBA 6.如图已知 AB 是⊙O 的直径,P 是 BA 延长线上一点, PC 切⊙O 于 C,PA=6,PEF 是⊙O 的割线,设 PE=x, PF=y,弦 CM⊥AB 于 D,且 AD:DB=1:2, 求y与x之间的函数关系式, 并求出自变量x取值范围。 第 15 课 二次函数 〖知识点〗二次函数、抛物线的顶点、对称轴和开口方向 〖大纲要求〗 1. 理解二次函数的概念; 2. 会把二次函数的一般式化为顶点式,确定图象的顶点坐标、对称轴和开口方向,会 用描点法画二次函数的图象; 3. 会平移二次函数 y=ax2(a≠0)的图象得到二次函数 y=a(ax+m)2+k 的图象,了解 特殊与一般相互联系和转化的思想; 4. 会用待定系数法求二次函数的解析式; A OP B C M F D E 5. 利用二次函数的图象,了解二次函数的增减性,会求二次函数的图象与 x 轴的交点 坐标和函数的最大值、最小值,了解二次函数与一元二次方程和不等式之间的联系。 内容 (1)二次函数及其图象 如果 y=ax2+bx+c(a,b,c 是常数,a≠0),那么,y 叫做 x 的二次函数。 二次函数的图象是抛物线,可用描点法画出二次函数的图象。 (2)抛物线的顶点、对称轴和开口方向 抛物线 y=ax2+bx+c(a≠0)的顶点是 )4 4,2( 2 a bac a b  ,对称轴是 a bx 2  ,当 a>0 时, 抛物线开口向上,当 a<0 时,抛物线开口向下。 抛物线 y=a(x+h)2+k(a≠0)的顶点是(-h,k),对称轴是 x=-h. 〖考查重点与常见题型〗 1. 考查二次函数的定义、性质,有关试题常出现在选择题中,如: 已知以 x 为自变量的二次函数 y=(m-2)x2+m2-m-2 额图像经过原点, 则 m 的值是 2. 综合考查正比例、反比例、一次函数、二次函数的图像,习题的特点是在同一直角 坐标系内考查两个函数的图像,试题类型为选择题,如: 如图,如果函数 y=kx+b 的图像在第一、二、三象限内,那么函数 y=kx2+bx-1 的图像大致是( ) y y y y 1 1 0 x o-1 x 0 x 0 -1 x A B C D 3. 考查用待定系数法求二次函数的解析式,有关习题出现的频率很高,习题类型有中 档解答题和选拔性的综合题,如: 已知一条抛物线经过(0,3),(4,6)两点,对称轴为 x=5 3 ,求这条抛物线的解析式。 4. 考查用配方法求抛物线的顶点坐标、对称轴、二次函数的极值,有关试题为解答题, 如: 已知抛物线 y=ax2+bx+c(a≠0)与 x 轴的两个交点的横坐标是-1、3,与 y 轴交点的纵 坐标是-3 2 (1)确定抛物线的解析式;(2)用配方法确定抛物线的开口方向、对称轴和顶 点坐标. 5.考查代数与几何的综合能力,常见的作为专项压轴题。 习题 1: 一、填空题:(每小题 3 分,共 30 分) 1、已知A(3,6)在第一象限,则点B(3,-6)在第 象限 2、对于y=-1 x ,当x>0时,y随x的增大而 3、二次函数y=x2+x-5取最小值是,自变量x的值是 4、抛物线y=(x-1)2-7的对称轴是直线x= 5、直线y=-5x-8在y轴上的截距是 6、函数y= 1 2-4x 中,自变量x的取值范围是 7、若函数y=(m+1)xm2+3m+1是反比例函数,则 m 的值为 8、在公式1-a 2+a =b中,如果b是已知数,则a= 9、已知关于x的一次函数y=(m-1)x+7,如果y随x的增大而减小,则m的取值 范围是 10、 某乡粮食总产值为m吨,那么该乡每人平均拥有粮食y(吨),与该乡人口数x的 函数关系式是 二、选择题:(每题 3 分,共 30 分) 11、函数y= x-5 中,自变量x的取值范围 ( ) (A)x>5 (B)x<5 (C)x≤5 (D)x≥5 12、抛物线y=(x+3)2-2的顶点在 ( ) (A)第一象限 (B) 第二象限 (C) 第三象限 (D) 第四象限 13、抛物线y=(x-1)(x-2)与坐标轴交点的个数为 ( ) (A)0 (B)1 (C)2 (D)3 14、下列各图中能表示函数和在同一坐标系中的图象大致是( ) -3 -2 -1 1 2 3 3 2 1 -1 -2 -3 4 5 -5 -4 4 5 -4 -5 x y 0 -3 -2 -1 1 2 3 3 2 1 -1 -2 -3 4 5 -5 -4 4 5 -4 -5 x y 0 -3 -2 -1 1 2 3 3 2 1 -1 -2 -3 4 5 -5 -4 4 5 -4 -5 x y 0 -3 -2 -1 1 2 3 3 2 1 -1 -2 -3 4 5 -5 -4 4 5 -4 -5 x y 0 (A) (B) (C) (D) 15.平面三角坐标系内与点(3,-5)关于y轴对称点的坐标为( ) (A)(-3,5) (B)(3,5) (C)(-3,-5) (D)(3,-5) 16.下列抛物线,对称轴是直线x=1 2 的是( ) (A) y=1 2 x2(B)y=x2+2x(C)y=x2+x+2(D)y=x2-x-2 17.函数y= 3x 1-2x 中,x的取值范围是( ) (A)x≠0 (B)x>1 2 (C)x≠1 2 (D)x<1 2 18.已知 A(0,0),B(3,2)两点,则经过 A、B 两点的直线是( ) (A)y=2 3 x (B)y=3 2 x (C)y=3x (D)y=1 3 x+1 19.不论m为何实数,直线y=x+2m与y=-x+4 的交点不可能在( ) (A)第一象限 (B)第二象限 (C)第三象限 (D)第四象限 20.某幢建筑物,从 10 米高的窗口 A 用水管和向外喷水,喷的水流呈抛物线(抛 物线所在平面与墙面垂直,(如图)如果抛物线的最高点 M 离墙 1 米,离地面40 3 米, 则水流下落点 B 离墙距离 OB 是( ) (A)2 米 (B)3 米 (C)4 米 (D)5 米 三.解答下列各题(21 题 6 分,22----25 每题 4 分,26-----28 每题 6 分,共 40 分) 21.已知:直线y=1 2 x+k过点 A(4,-3)。(1)求k的值;(2)判断点 B(-2,-6) 是否在这条直线上;(3)指出这条直线不过哪个象限。 22.已知抛物线经过 A(0,3),B(4,6)两点,对称轴为x=5 3 , (1) 求这条抛物线的解析式; (2) 试证明这条抛物线与 X 轴的两个交点中,必有一点 C,使得对于x轴上任意一点 D 都有 AC+BC≤AD+BD。 23.已知:金属棒的长 1 是温度t的一次函数,现有一根金属棒,在 O℃时长度为 200cm, 温度提高 1℃,它就伸长 0.002cm。 (1) 求这根金属棒长度l与温度t的函数关系式; (2) 当温度为 100℃时,求这根金属棒的长度; (3) 当这根金属棒加热后长度伸长到 201.6cm时,求这时金属棒的温度。 24.已知x1,x2,是关于x的方程x2-3x+m=0 的两个不同的实数根,设s=x1 2+x 2 2 (1) 求 S 关于m的解析式;并求m的取值范围; (2) 当函数值s=7 时,求x1 3+8x2 的值; 25.已知抛物线y=x2-(a+2)x+9 顶点在坐标轴上,求a的值。 26、如图,在直角梯形ABCD中,∠A=∠D=Rt∠,截取AE=BF=DG=x, 已知AB=6,CD=3,AD=4,求: (1) 四边形CGEF的面积S关于x的函数表达式和X的取值范围; (2) 当x为何值时,S的数值是x的4倍。 27、国家对某种产品的税收标准原定每销售100元需缴税8元(即税率为8%),台洲 经济开发区某工厂计划销售这种产品m吨,每吨2000元。国家为了减轻工人负担,将税 收调整为每100元缴税(8-x)元(即税率为(8-x)%),这样工厂扩大了生产, 实际销售比原计划增加2x%。 (1) 写出调整后税款y(元)与x的函数关系式,指出x的取值范围; (2) 要使调整后税款等于原计划税款(销售m吨,税率为8%)的78%,求x的值. 28、已知抛物线y=x2+(2-m)x-2m(m≠2)与y轴的交点为A,与x轴的 交点为B,C(B点在C点左边) (1) 写出A,B,C三点的坐标; (2) 设m=a2-2a+4试问是否存在实数a,使△ABC为Rt△?若存在,求出a 的值,若不存在,请说明理由; (3) 设m=a2-2a+4,当∠BAC最大时,求实数a的值。 习题 2: 一.填空(20 分) 1.二次函数=2(x - 3 2 )2 +1 图象的对称轴是 。 2.函数 y= 1 2 1 x x   的自变量的取值范围是 。 3.若一次函数 y=(m-3)x+m+1 的图象过一、二、四象限,则的取值范围是 。 4.已知关于的二次函数图象顶点(1,-1),且图象过点(0,-3),则这个二次函数解析式 为 。 5.若 y 与 x2 成反比例,位于第四象限的一点 P(a,b)在这个函数图象上,且 a,b 是方程 x2-x -12=0 的两根,则这个函数的关系式 。 6.已知点 P(1,a)在反比例函数 y= k x (k≠0)的图象上,其中 a=m2+2m+3(m 为实数), 则这个函数图象在第 象限。 7. x,y 满足等式 x= 3 2 2 1 y y   ,把 y 写成 x 的函数 ,其中自变量 x 的取值范 围是 。 8.二次函数 y=ax2+bx+c+(a  0)的图象如图,则点 P(2a-3,b+2) 在坐标系中位于第 象限 9.二次函数 y=(x-1)2+(x-3)2,当 x= 时,达到最小值 。 10.抛物线 y=x2-(2m-1)x- 6m 与 x 轴交于(x1,0)和(x2,0)两点,已知 x1x2=x1+x2+49, 要使抛物线经过原点,应将它向右平移 个单位。 二.选择题(30 分) 11.抛物线 y=x2+6x+8 与 y 轴交点坐标( ) (A)(0,8) (B)(0,-8) (C)(0,6) (D)(-2,0)(-4,0) 12.抛物线 y= - 1 2 (x+1)2+3 的顶点坐标( ) (A)(1,3) (B)(1,-3) (C)(-1,-3) (D)(-1,3) 13.如图,如果函数 y=kx+b 的图象在第一、二、三象限,那么函数 y=kx2+bx-1 的图象大致 是( ) 14.函数 y= 2 1 x x   的自变量 x 的取值范围是( ) (A)x  2 (B)x<2 (C)x> - 2 且 x  1 (D)x  2 且 x  –1 15.把抛物线y=3x2先向上平移2 个单位,再向右平移3 个单位,所得抛物线的解析式是( ) (A)=3(x+3)2 -2 (B)=3(x+2)2+2 (C)=3(x-3)2 -2 (D)=3(x-3)2+2 16.已知抛物线=x2+2mx+m -7 与 x 轴的两个交点在点(1,0)两旁,则关于 x 的方程 1 4 x2+ (m+1)x+m2+5=0 的根的情况是( ) (A)有两个正根 (B)有两个负数根 (C)有一正根和一个负根 (D)无实根 17.函数 y= - x 的图象与图象 y=x+1 的交点在( ) (A) 第一象限 (B)第二象限 (C)第三象限 (D)第四象限 y xO x y o-2 -2 x y o x y o x y o x y o 1 1 -1 -1 A B C D 18.如果以 y 轴为对称轴的抛物线 y=ax2+bx+c 的图象,如图, 则代数式 b+c-a 与 0 的关系( ) (A)b+c-a=0 (B)b+c-a>0 (C)b+c-a<0 (D)不能确定 19.已知:二直线 y= - 3 5 x +6 和 y=x - 2,它们与 y 轴所围成的三角形的面积为( ) (A)6 (B)10 (C)20 (D)12 20.某学生从家里去学校,开始时匀速跑步前进,跑累了后,再匀速步行余下的路程。下图 所示图中,横轴表示该生从家里出发的时间 t,纵轴表示离学校的路程 s,则路程 s 与时间 t 之间的函数关系的图象大致是( ) 三.解答题(21~23 每题 5 分,24~28 每题 7 分,共 50 分) 21.已知抛物线 y=ax2+bx+c(a  0)与 x 轴的两交点的横坐标分别是-1 和 3,与 y 轴交点 的纵坐标是- 3 2 ; (1)确定抛物线的解析式; (2)用配方法确定抛物线的开口方向,对称轴和顶点坐标。 22、如图抛物线与直线 )4(  xky 都经过坐标轴的正半轴上 A,B 两点,该抛物线的对 称轴 x=—1,与 x 轴交于点 C,且∠ABC=90°求: (1)直线 AB 的解析式; (2)抛物线的解析式。 23、某商场销售一批名脾衬衫,平均每天可售出 20 件,每件盈利 40 元,为了扩大销售,增 加盈利,尽快减少库存,商场决定采取适当的降价措施.经调查发现每件衬衫降价 1 元, 商 场平均每天可多售出 2 件: (1)若商场平均每天要盈利 1200 元,每件衬衫要降价多少元, (2)每件衬衫降价多少元时,商场平均每天盈利最多? 24、已知:二次函数 1222  baxxy 和 1)3( 22  bxaxy 的图象都经过 x 轴上两个不同的点 M、N,求 a、b 的值。 25、如图,已知⊿ABC 是边长为 4 的正三角形,AB 在 x 轴上,点 C 在第一象限,AC 与 y 轴 交于点 D,点 A 的坐标为{—1,0),求 (1)B,C,D 三点的坐标; (2)抛物线 cbxaxy  2 经过 B,C,D 三点,求它的解析式; (3)过点 D 作 DE∥AB 交过 B,C,D 三点的抛物线于 E,求 DE 的长。 s to s to s to s to A B C D Y X B C O A Y XB C OA D E 26 某市电力公司为了鼓励居民用电,采用分段计费的方法计算电费:每月用电不超 100 度 时,按每度 0.57 元计费:每月用电超过 100 度时.其中的 100 度仍按原标准收费,超过部 分按每度 0.50 元计费。 (1)设月用电 x 度时,应交电费 y 元,当 x≤100 和 x>100 时,分别写出 y 关于 x 的函数 关系式; (2)小王家第一季度交纳电费情况如下: 月 份 一月份 二月份 三月份 合 计 交费金额 76 元 63 元 45 元 6 角 184 元 6 角 问小王家第一季度共用电多少度? 27、巳知:抛物线 62)5( 222  mxmxy (1)求证;不论 m 取何值,抛物线与 x 轴必有两个交点,并且有一个交点是 A(2,0); (2)设抛物线与 x 轴的另一个交点为 B,AB 的长为 d,求 d 与 m 之间的函数关系式; (3)设 d=10,P(a,b)为抛物线上一点: ①当⊿ABP是直角三角形时,求 b 的值; ②当⊿ABP是锐角三角形,钝角三角形时,分别写出 b 的取值范围(第 2 题不要求写 出过程) 28、已知二次函数的图象 )9 24(2)2 54( 222  mmxmmxy 与 x 轴的交点为 A, B(点B在点 A 的右边),与 y 轴的交点为 C; (1)若⊿ABC 为 Rt⊿,求 m 的值; (1)在⊿ABC 中,若 AC=BC,求 sin∠ACB 的值; (3)设⊿ABC 的面积为 S,求当 m 为何值时,s 有最小值.并求这个最小值。 第 16 课 统计初步 〖知识点〗 总体、个体、样本、样本容量、平均数、方差、标准差、方差的简化公式、 频率分布、频率分布直方图 〖大纲要求〗 1. 了解总体、个体、样本、样本容量等概念; 2. 了解样本方差、总体方差、样本标准差的意义,理解加权平均数的概念,掌握它的 计算公式,会计算样本方差和样本标准差,理解频数、频率的概念,掌握整理数据 的步骤和方法,会列出样本频率分布表,画出频率分布直方图。 〖考查重点与常见题型〗 1. 通过具体问题考查总体、个体、样本、样本容量的概念,有关试题常出现在选择题 中,如: 为了了解某地区初一年级 7000 名学生的体重情况,从中抽取了 500 名学生的体重,就 这个问题来说,下面说法中正确的是( ) (A)7000 名学生是总体 (B)每个学生是个体 (C)500 名学生是所抽取的一个样本 (D)样本容量是 500 2. 考查平均数的求法,有关习题常出现在填空题或选择题中,如: (1) 已知一组数据为 3,12,4,x,9,5,6,7,8 的平均数为 7,则 x= (2)某校篮球代表队中,5 名队员的身高如下(单位:厘米):185,178, 184,183,180,则这些队员的平均身高为( ) (A)183 (B)182 (C)181 (D)180 3. 考查样本方差、标准差的计算,有关试题常出现在选择题或填空题中,如: (1)数据 90,91,92,93 的标准差是( ) (A) 2 (B)5 4 (C) 5 4 (D) 5 2 (2)甲、乙两人各射靶 5 次,已知甲所中环数是 8、7、9、7、9,乙所中的环数的平 均数 x2=8,方差 S2 乙=0.4,那么,对甲、乙的射击成绩的正确判断是( ) (A)甲的射击成绩较稳定 (B)乙的射击成绩较稳定 (C)甲、乙的射击成绩同样稳定 (D)甲、乙的射击成绩无法比较 4. 考查频率、频数的求法,有关试题常出现在选择题中,如: 第十中学教研组有 25 名教师,将他的年龄分成 3 组,在 38~45 岁组内有 8 名教师, 那么这个小组的频数是( ) (A)0.12 (B)0.38 (C)0.32 (D)3.12 〖预习练习〗 1. 一各样本中,数据 15 和 13 各有 4 个,数据 14 有 2 个,求这个样本的 平均数、方差和标准差(标准差保留两个有效数据) 一.考点训练 1.某市今年有 9068 名初中毕业生参加升学考试,从中抽出 300 名考生的成绩进行分析。 在 这 个 问 题 中 , 总 体 是 __________________________ ; 个 体 是 ___________ ; 样 本 是 _______________________;样本容量是__________. 2.在一个班级 50 名学生中,30 名男生的平均身高是 1.60 米,20 名女生的平均身高是 1.50 米,那么这个班学生的平均身高是________米. 3.已知一个样本为 8,14,12,18,那么样本的方差是_______;标准差是_________. 4.甲乙两个学生参加夏令营的射击比赛,每人射击 5 次,甲的环数分别是 5,9,8,10, 8;乙的环数是 6,10,5,10,9;问:(1)甲乙两人谁的命中率高些?(2)谁的射击水平 发挥得较稳定? 5.某班 45 名同学在一次数学测试中成绩如下(单位:分) 83,70,82,95,91,100,98,89,91,94,68,75,85,90,97,83, 92,56,70,89,100,90,72,63,60,79,85,86,78,65,92,80, 75,74,81,80,97,90,74,85,96,87,82,75,70, 选择恰当的组距,画出频率分布直方图。 解:(1)计算最大值与最小值的差_________. (2)决定组距与组数:取组距 10,组数____,分为______组。 (3)决定分点 50.5~60.5,______________________________,90.5~100.5 (4)列频率分布表: (5)绘制频率分布直方图: 分 组 频 数 累 计 频 数 频 率 ~ ~ ~ ~ ~ 频率 组距 1 0.5 分数 二.解题指导: 1.某班有 45 人,平均体重为 48 千克,其中有 20 人是女生,平均体重为 43 千克,问: 男生平均体重是________千克。 2.一个班的学生中,14 岁的有 16 人,15 岁的有 14 人,16 岁的有 8 人,17 岁的有 4 人。这个班学生的平均年龄是______岁. 3.从同一家工厂生产的 20 瓦日光灯中抽出 6 支,40 瓦日光灯中抽出 8 支进行使用寿 命(单位:小时)测试,结果如下: 20 瓦 457 443 459 451 464 438 40 瓦 466 452 438 467 455 459 464 439 哪种日光灯的寿命长?哪种日光灯的质量比较稳定? 4.一个样本中所有的数据各不相同,若将其分为 5 组,已知第一、二、三组的累计频 数是 84,第三、四、五组的累计频数是 72,并且前后两个三组中有 47 个相同的数据,求这 个 样本的容量。 5.某校初三年级的一次自然测验中,样本数据落在 79.5~84.5 之间的频数是 0.35, 全年级共有学生 240 人,则估计全年级这次自然测验成绩在 79.5~84.5 分之间的同学大约 有多少人? 6.某样本数据分为五组,第一组的频率是 0.3,第二、三组的频率相等,第四、五组 的频率之和为 0.2,则第三组的频率是多少? 三.独立训练 1.为了考察一个养鸡场的鸡的生长情况,从中抓了 5 只,秤得它们的重量(单位:千 克)是:3.0,3.4,3.1,3.2,3.3,在这个问题中样本是指______,样本容量是_________, 样本平均数____________(千克)。 2.有一个样本,各个数据的和为 505,如果这个样本的平均数为 5,则它的样本容量为 _____________. 3.一组数据同时减去 70,算得新的一组数据的平均数为 0.3,则原数据的平均数为 ______________. 4.若 2,7,6 和 x 四个数的平均数是 5,18,1,6,x 与 y 五个数的平均数是 10,则 y=_______. 5.将一批数据分成五组,列出频数分布表,第一组频率为 0.2,第四组与第二组的频 率之和为 0.5,那么第三、五组频率之和为_________. 6.对 150 名男生的身高进行测量,数据最大的是 181 厘米,最小的是 164 厘米,为了 列频率分布表取组距为 2 厘米,则应将数据分成___________组。 7.已知数据 x1,x2,x3 的平均数是 m,那么数据 3x1+7,3x2+7,3x3+7 的平均数等于 _________. 8.计算样本 1,2,2,-3,3 的方差为____________. 9.在统计中,样本的方差可以近似地反映一组数据的_________. 10.如果数据 x1,x2,x3,…xn 的的平均数是 x,求:(x1 - x)+(x2 - x)+…+(xn -x)的值。 11.已知在 n 个数据中,x1 出现 f1 次,x2 出现 f2 次,……xk 出现 fk 次(f1+f2+f3+…+fk=n), x 是这 n 个数据的平均数, 求证:f1(x1 – x)+f2(x2 – x)+…+fk(xk – x)=0 12.甲乙两种棉苗各抽 10 株,测得它们的株高分别如下:(单位:厘米) 甲:25,41,40,37,22,14,19,21,42,39 乙:27,16,44,27,44,16,40,40,16,40 哪一种棉苗长得高?哪一种棉花长得齐? 第 17 课 概率 〖知识点〗 必然事件、不可能事件、随机事件、概率、等可能性事件、树图、生命表 意义、期望值 〖大纲要求〗 了解学习概率的意义,理解随机事件、不可能事件、必然事件,理解并学 会概率的定义及其统计算法和等可能性事件的概率及其计算方法,了解并 初步学会概率的简单应用。 〖考查重点与常见题型〗 考查必然事件、不可能事件的概率,等可能性事件的概率及其计算,概率 的简单应用(生命表、中奖率、期望值),如: (1) 有左、右两个抽屉,左边抽屉有 2 个红球,右边抽屉有 1 个红球和 2 个白球, 从中任取一球是红球的概率是 (2) 连续二次抛掷一枚硬币,二次正面朝上的概率是( ) (A)1 (B)1 2 (C)1 4 (D)3 4 〖预习练习〗 1. 指出下列事件是必然事件,还是随机事件,还是不可能事件? (1) 5 张卡片上各写 2,4,6,8,10 中的一个数,从中任取一张是偶数; (2) 从(1)题的 5 张中任取一张是奇数; (3) 从(1)题的 5 张卡片中任取一张是 3 的倍数. 2. 下列事件中哪些是等可能性事件,哪些不是? (1) 某运动员射击一次中靶心与不中靶心; (2) 随意抛掷一枚硬币背面向上与正面向上; (3) 随意抛掷一只纸可乐杯杯口朝上,或杯底朝上,或横卧; (4) 从分别写有 1,3,5,7,9 中的一个数的五张卡片中任抽 1 张结果是 1,或 3, 或 5,或 7,或 9. 3. 从装有 5 个红球和 3 个白球的袋中任取 4 个,那么取道的“至少有 1 个 是红球”与“没有红球”的概率分别为 与 4. 某产品出现次品的概率 0.05,任意抽取这种产品 800 件,那么大约有 件是次 品 5. 设有甲、乙两把不相同的锁,甲锁配有 2 把钥匙,乙锁配有 1 把钥匙,设事件 A 为“从这 3 把钥匙中任选 2 把,打开甲、乙两把锁”,则 P(A)= 6.甲、乙、丙三人随意排成一列拍照,甲恰好排在中间的概率( ) (A)2 9 (B)1 3 (C)4 9 (D)以上都不对 7.从 1,2,3,4,5 的 5 个数中任取 2 个,它们的和是偶数的概率是( ) (A) 1 10 (B)1 5 (C)2 5 (D)以上都不对 考点训练: 1、 下列事件是随机事件的是( ) (A)两个奇数之和为偶数, (B)某学生的体重超过 200 千克, (C)宁波市在六月份下了雪, (D)三条线段围成一个三角形。 2、下列事件中是等可能性事件有( )件 1 某运动员射击一次中靶心与不中靶心, 2 随意抛一枚硬币背面向上与正面向上, 3 随意投掷一只纸可乐杯杯口朝上或杯底朝上或横卧, 4 从分别写有 1,3,5,7,9 中的一个数的五张卡片中任抽 1 张结果是 1 或 3 或 5 或 7 或 9 (A)1 件 (B)2 件 (C)3 件 (D)4 件 3、设有编号为 1 到 50 的 50 张考签,一学生任意抽取一张进行面授,那么该学生抽到前 20 号考签的概率是 ; 4、袋中装有 3 个白球,2 个红球,1 个黑球,从中任取 1 个,那么取到的不是红球的概率 是 ; 5、某射手在同一条件下进行射击,结果如下表所示: 射击次数(n) 10 20 50 100 200 500 … 击中靶心次数(m) 8 19 44 92 178 455 … 击中靶心频率(m n ) … 请填好最后一行的各个频率,由此表推断这个射手射击 1 次,击中靶心的概率的 是 ; 6、人寿保险公司的一张关于某地区的生命表的部分摘录如下: 年龄 活到该年龄的人数 在该年龄的死亡人数 40 80500 892 50 78009 951 60 69891 1200 70 45502 2119 80 16078 2001 … … … 根据上表解下列各题: (1) 某人今年 50 岁,他当年去世的概率是多少?他活到 80 岁的概率是多少? (保留三个有效数字) (2) 如果有 20000 个 50 岁的人参加人寿保险,当年死亡的人均赔偿金为 10 万元,预计保 险公司需付赔偿的总额为多少? 解题指导: 1、 一次有奖销售活动中,共发行浆券 1000 张,凡购满 100 元商品者得奖券一张,这次有 奖销售设一等奖 1 名,奖金 500 元,二等奖 2 名,奖金各 200 元,三等奖 10 名,奖金 各 50 元,四等奖 100 名,奖金各 10 元; (1) 求出奖金总额,并与 95 折销售相比,说明哪一种销售方法向消费者让利较多; (2) 某人购买 100 元的商品,他中一等奖的概率是多少?中二等奖的概率是多少?中三等 奖的概率是多少?中四等奖的概率是多少? (3) 某人购买 1000 元的商品,他中奖的概率是多少? 2、 一项新产品试制实验结果如下表: 试制次数 5 10 20 40 60 成功次数 3 7 15 31 48 用 500 万元投资生产该种新产品,如果成功,则可获利 2000 万元;如果失败,将亏损投资 数的 80%,求投资该项目的期望值。 3、 有左、中、右三个抽屉,左边的抽屉里放 2 个白球,中间和右边的抽屉里各放一个红球 和一个白球,从三个抽屉里任选一个球是红球的概率是多少?是白球的概率是多少? 独立训练: 1、对某厂的 200 件产品任意抽取 200 件进行检查,结果有 4 件是次品,其余都是合格品, 那 么 从 中 任 意 取 1 件 产 品 , 取 道 的 是 “ 次 品 ” 与 “ 合 格 品 ” 的 概 率 分 别 是 与 ; 2、小明书包中有语文、社会、数学、自然、外语 5 本书,从中任意取 1 本,设事件 A 为“取 出的书是数学或外语”,那么 P(A)= ; 3、某产品出现次品的概率为 0.05,任意抽取这种产品 600 件,那么大约有 件是次品; 4、从装有 5 个红球和 3 个白球的袋中任意取 4 个,那么取道的“至少有 1 个是红球”与“没 有红球”的概率分别为 和 ; 5、 对某名牌衬衫抽检结果如下表: 抽检件数 10 20 100 150 200 300 不合格件数 0 1 3 4 6 9 如果销售 1000 件该名牌衬衫,至少要准备 件合格品,供顾客更换; 6、 在某种条件下,只有事件 A,B,C,三种可能,且它们彼此互斥,已知 P(A)=1 7 ,P(B)=1 4 ,P(C)= ; 7、 某地区道路如图,其中 H 区域是布雷区, 工兵沿箭头方向前进,进入布雷区的概率是 ; 8、随意地抛掷一只纸可乐杯,杯口朝上的概率约是 0.22,杯底朝下的概率约是 0.38,则横卧的概率是 ; 9.布袋里有 2 个白球和 3 个红球,从布袋里取两次球,每次取一个,取出后放回, 则两次 取出都是红球的概率是 。 10.某篮球运动员投 3 分球的命中率为 0.5,投 2 分球的命中率为 0.8,一场比赛中据说他投 了 20 次 2 分球, 投了 6 次 3 分球,估计他在这场比赛中得了 分; 11.某零存整取有奖储蓄 5000 张奖券中,有一等奖 1 张,二等奖 10 张,三等奖 50 张,不 设其奖,则买 1 张奖券,得三等奖以上的概率是 ,买 2 张奖券,都不中奖以上的概率 是 ; 12.由 1 到 9 的 9 个数字中任意组成一个二位数(个位与十位上的数字可以重复),计算: 1 个位数字与十位数字之积为奇数的概率 ; ②个位数字与十位数字之和为偶数的概率 ; ③个位数字与十位数字之积为偶数的概率 ; 第 18 课 线段与角、相交线与平行线 〖知识点〗 两点确定一条直线、相交线、线段、射线、线段的大小比较、线段的和与差、线段的 中点、角、角的度量、角的平分线、锐角、直角、钝角、平角、周角、对顶角、邻角、余角、 补角、点到直线的距离、同位角、内错角、同旁内角、平行线、平行线的性质及判定、命题、 定义、公理、定理 〖大纲要求〗 1. 了解直线、线段和射线等概概念的区别,两条相交直线确定一个交点, 解线段和与差及线段的中点、两点间的距离、角、周角、平角、直角、锐角、钝角等概念, 掌握两点确定一条直线的性质,角平分线的概念,度、分、秒的换算,几何图形的符号表示 法,会根据几何语句准确、整洁地画出相应的图形; 2. 了解斜线、斜线段、命题、定义、公理、定理及平行线等概念,了解垂线 段最短的性质,平行线的基本性质,理解对顶角、补角、邻补角的概念,理解对顶角的性质, 同角或等角的补角相等的性质,掌握垂线、垂线段、点到直线的距离等概念,会识辨别同位 角、内错角和同旁内角,会用一直线截两平行线所得的同位角相等、内错角相等、同旁内角 互补等性质进行推理和计算,会用同位角相等、内错角相等、或同旁内角互补判定两条直线 平行 〖考查重点与常见题型〗 1. 求线段的长、角的度数等,多以选择题、填空题出现,如: 已知∠а=112°,则∠а的补角的度数是 2. 利用平行线的判定与性质证明或计算,常作为主要定理或公理使用,如: 如图,AB∥CD,∠CFE=112°,ED 平分∠BEF, A E B 交 CD 于 D,则∠EDF= 〖预习练习〗 C F D 1.下列语句正确的是( ) (A)正方形是轴对称图形,它共有两条对称轴 (B)两条直线被第三条直线所截,同位角相等 (C)两点确定一条直线 (D)从直线外一点到这条直线的垂线段,叫作点到直线的距离 2.命题“等腰三角形的两个底角相等”的逆命题是 3. 若一个角的余角是这个角的 4 倍,则这个角的度数是 4. 把 63.5°用度分秒表示 ,把 18°18′18″用度表示 5. 计算(1)(36°15′24″+13°21′54″)×3 (2)(180°-91°32′24″)÷2 考点训练: 1.在平面上画出四条直线,交点的个数最多应该是( ) (A) 4 个 (B) 5 个 (C) 6 个 (D) 8 个 2.如果∠α与∠β是邻补角,且∠α> ∠β,那么∠β的余角是( ) (A) 1 2 (∠α±∠β) (B) 1 2 ∠α (C) 1 2 (∠α-∠β) (D)不能确定 3.已知三条直线 a,b,c,下列命题中错误的是( ) (A)如果 a∥b,b∥c,那么 a∥c (B) (B)如果 a⊥b,b⊥c,那么 a⊥c (C)如果 a⊥b,b⊥c,那么 a∥c (D) (D)如果 a⊥b,a∥c,那么 b⊥c 4.如图,AB∥CD,AC∥BD,下面推理不正确的是( ) (A)∵AB∥CD(已知) ∴∠A=∠5(两直线平行,同位角相等); (B)∵AC∥BD(已知) ∴∠3=∠4(两直线平行,內錯角相等); (C)∵AB∥CD(已知) ∴∠1=∠2(两直线平行,內錯角相等); (D)∵AB∥CD(已知) ∴∠3=∠4 (两直线平行,內錯角相等)。 5. B 是线段 AC 上一点,若 M 为 AB 中点,N 为 AC 中点,则 MN:BC 。 6. 如果两个角的两边分别平行且一个角比 另一个角的 3 倍少 30°,则这两个角的度数分别为 7. 如图,已知 DE∥BC,BD 是∠ABC 的分别平分线∠EDC=109°, ∠ABC=50°则∠A 度,∠BDC= 度。 8. 如图,AB∥CD,BE,CE 分别平分∠ABC,∠BCD,则∠AEB+∠CED= 。 9.两个相等的钝角,它们有公共顶点和一条公共边, 另两条边所成的角是直角,求这两个钝角的度数。 10.已知如图,AB∥CD ∠DAB=∠DCB,AE 平分∠DAB 且交 DC 于 E,CF 平分∠DCB 且交 AB 于 F.求证: AE∥FC。 解题指导: 1.判断题: (1).延长射线 OM;( ) (2).平角是一条射线;( ) (3).线段、射线都是直线的一部分; ( ) (4).锐角一定小于它的余角;( ) (5).大于直角的角是钝角;( ) (6).一个锐 角的补角与这个锐角的余角的差是 90°;( ) (7).相等的两个角是对顶角;( ) (8). 若∠A+∠B+∠C=180°,则这三个角互补;(9). 互为邻补角的两个角的平分线互相垂直。 ( ) 2.如图,直线MN,PQ相交于O,OR平分∠MON,OK⊥PQ. 图中锐角有 个,钝角有 个, ∠ROK 的余角是 ; ∠ROK 的补角是 . 3.(1) 16.38°化为度分秒是 ; 53°30´45´´ 化为度是 (精确到 0.1 度). (2).若∠α=38°5´46´´,∠β=72°18´8´´ 则 3α-1 2 β= . 4.下列命题中(1)过一点有且只有一条直线垂直于已知直线;(2)经过一点有且只有一条直线 C D A B 5 1 3 2 4 A D C E B A B E D C K R Q M P O N A F D B E C 和已知直线平行;(3)过线段 AB 外一点 P 作线段 AB 的中垂线;(4)如果直线 l1 与 l2 相交, 直线 l3 与 l4 相交,那么 l1∥l3;(5)如果两条直线都与同一条直线垂直,那么这两条直线平 行;(6)两条直线没有公共点,那么这两条直线一定平行;(7)两条直线与第三条直线相交, 如果内错角相等,则同旁内角互补;其中正确命题的个数为( ) (A) 2 个 (B) 3 个 (C) 4 个 (D)5 个 5.已知∠α<60°,∠AOB=3∠α,如果 OC 平分∠AOB,求∠α的值. 6.已知如图:AC⊥BC,HF⊥AB,CD⊥AB, ∠EDC 与∠CHF 互补,求证:DE⊥AC. 7.如图,AB∥CD,求∠BAE+∠AEF+∠EFC+∠FCD 的度数. 独立训练: 1.在同一平面内,有 l1,l2,l3,l4,l5 五条直线,若 l1⊥l2, l2⊥l3, l3⊥l4, l4⊥l5,那么 l1 与 l5 的位置关系是( ) (A)平行 (B)垂直 (C)平行或垂直 (D)即不平行,也不垂直; 2.下列叙述中正确的是( ) (A)平角是一条直线 (B)平角就是两个直角 (C)两边成一条直线的角就是直角 (D)互补的角就是平角 3.如图,直线 a∥b∥c,则图中与∠1 相等的角有( )个 (A) 1 (B) 2 (C) 3 (D) 5 4.用一副三角板可以作出大于 0°而小于 180°的角的个数( ) (A) 5 个 (B) 10 个 (C) 11 个 (D)以上都不对 5.Rt△ABC 中,CD 是斜边上的高,则图中互为余角的角有( ) (A)6 对 (B)5 对 (C)4 对 (D)3 对 6.如图,AB∥CD, ∠A=75°,∠C=30°, 则∠E 的度数为 . 7.如图,∠B=43°26´,DE∥BC,DF⊥AB 于, 则∠D= . 8.如图,三条直线两两相交 图中共有 对对顶角,共有 对同位角, 共有 对内错角,共有 对同旁内角。 9.如图,∠DAB=∠BCD=110°,∠ADC=70°,哪些直线互相平行,为什么? 10.如图,已知∠1 与它的余角相等,∠2 是它的补角的 3 倍, 那么直线 l1 与 l2 平行吗?为什么? 第 19 课 三角形与全等三角形 知识点: 三角形,三角形的角平分线,中线,高线,三角形三边间的不等关系,三角形的内角和, 三角形的分类,全等形,全等三角形及其性质,三角形全等判定 大纲要求 1.了解全等形,全等三角形的概念和性质,逆命题和逆定理的概念,理解三角形,三角形 的顶点,边,内角,外角,角平分线,中线和高线,线段中垂线等概念。 A F B E C D H A F B E C D 1 A B E DC a bc A B D C l2 l2 l3 1 2 A B ED C 2.理解三角形的任意两边之和大于第三边的性质,掌握三角形的内角和定理,三角形的外 角等于不相邻的两内角的和;三角形的外角大于任何一个和它不相邻的内角的性质; 3.理解全等三角形的概念和性质。掌握全等三角形的判定公理及其推论,并能应用他们进 行简单的证明和计算。 4.学会演绎推理的方法,提高逻辑推理能力和逻辑表达能力,掌握寓丁几何证明中的分析, 综合,转化等数学思想。 考查重点与常见题型 1.三角形三边关系,三角形内外角性质,多为选择题,填空题; 2.论证三角形全等,线段的倍分,常见的多为解答题 预习练习 1.若ΔABC 的三边长分别为整数,周长为 11,且有一边为 4,则这个三角形的最大边长为 ( ) (A)7 (B)6 (C)5 (D)4 2.与三角形三个顶点距离相等的点是这个三角形的( ) (A)二条中线的交点 (B) 二条高线的交点 (C)三条角平分线交点 (D)三条中垂线交点 3.已知如图,∠A=32°,∠B=45°,∠C=38°则ΔDEF 等于( ) (A) 120°(B)115°(C)110°(D)105° 4.在ΔABC 中,如果∠A-∠B=90°,那么ΔABC 是( ) (A)直角三角形 (B) 钝角三角形 (C)锐角三角形 (D)锐角三角形或钝角三角形 5.已知 a,b,c 为ΔABC 的三条边,化简 (a-b-c)2 +|b-a-c|得 6.已知如图,BA=BD,BC=BE,∠ABD=∠CBE: 求证:AC=DE 考点训练: 1.三角形的三边分别为 3,1-2a,8,则 a 的取值范围是( ) (A)-6-2 2.ΔABC 的周长是 36,a+b=2c,a∶b=1∶2,则 a=------,b=------,c=--------, 3.下列命题(1)等边三角形也是等腰三角形;(2)三角形的外角等于两个内角的和;(3) 三角形中最大的内角不能小于 60°;(4)锐角三角形中,任意两内角之和必大于 90°,其 中错误的个数是( ) (A)0 个 (B)1 个 (C)2 个 (D)3 个 4.一个三角形的内心在它的一条高线上,则这个三角形一定是( ) (A)直角三角形 (B)等腰三角形 (C)等腰直角三角形 (D)等边三角形 5.如图ΔABC 中,D,E 分别为 BC,AB,AC 上的点 BD=BE,CD=CF,设∠A=α ∠EDF=β则下 列关系中正确的是( ) (A)2α+β=180°(B)α+2β=180°(C)α+β=90°(D)α+β=180° 6.满足下列用 P 种条件时,能够判定ΔABC≌ΔDEF( ) (A)AB=DE,BC=EF, ∠A=∠E (B)AB=DE,BC=EF ∠A=∠D (C) ∠A=∠E,AB=DF, ∠B=∠D (D) ∠A=∠D,AB=DE, ∠B=∠E 7.如图,平行四边形 ABCD 对角线 AC,BD 交于 O,过 O 画直线 EF 交 AD 于 E,交 BC 于 F,,则 图中全等三角形共有( ) (A)7 对 (B)6 对 (C)5 对 (D)4 对 8.两个三角形有以下三对元素对应相等, 则不能判定全等的是( ) (A)一边和任意两个角 (B)两边和他们的夹角 (C)两个角和他们一角的对边 (D)三边对值相等 9.如图,ΔABC 中,过 A 分别作∠ABC, ∠ ACB 的外角的平分线的垂线 AD,AE,D,E 为垂足; 求证(1)ED||BC(2)ED=1 2 (AB+AC+BC); (3)若过 A 分别作∠ABC,∠ACB 的平分线的垂线 AD,AE,垂足分别为 D,E,结论有无变 化?请加以说明。 10.如图,平行四边形 ABCD 中,E 是 CA 延长线上的点, F 是 AC 延长线上的点,且 AE=CF,求证:∠E=∠F 解题指导 1.如图,已知ΔABC 中,∠A=58°,如果(1)O 为外心,(2)O 为内心,(3)O 为垂心, 分别求∠BOC 的度数。 2.如图,在 ABC 中,D 在 AB 上,且ΔCAD 和ΔCBE 都是等边三角形, 求证:(1)DE=AB,(2)∠EDB=60° 3.求证:两个角及第三个角的角平分线对应相等的两个三角形全等。 4.如图,已知在ΔABC 中,∠B=2∠C,AD 平分∠BAC,求证:AC=AB+BD 5.如图,已知ΔABC 中,AB=AC,E 是 AB 的中点,延长 AB 到 D, 使 BD=BA,求证 :ED=2CE 独立训练 1.三角形的三个内角中至少有-----个锐角,三个外角中最多有-----个锐角。 2.三角形的一边是 8,另一边是 1,第三边如果是偶数,则第三边是----- ,这个三角形是--------- 三角形, 3.a,b,c 是 ΔABC 的三边,则 (a+b-c)2 -|b-a-c|=----------- 4.三角形的三边长度一定,这个三角形形状大小就完全确定。这个性质叫------------- 5.如果三角形的一个内角等于其他两个内角的和,这个三角形是( ) (A)锐角三角形 (B)钝角三角形 (C)直角三角形 (D)不能确定 6.如果一个三角形的内心,外心都在三角形内,则这个三角形是( ) (A)锐角三角形 (B)只有两边相等的锐角三角形 (C)直角三角形 (D)锐角三角形或直角三角形 7.如果一个三角形的外心,内心重合,那么这个三角形一定是( ) (A) 等边三角形 (B)只有两边相等的锐角三角形 (C) 直角三角形 (D)钝角三角形 8.如图, ΔABC 是钝角三角形 (1) 作出 AC 边上的高 BE (2) 作出角平分线 BF,中线 CG 9.下列所叙述的图形中,是全等三角形的只有( ) (A)两边相等的两个直角三角形(B)一边和一角对应相等的两个直角三角形 (C)长为厘米的 两个等边三角形 (D)一个钝角相等的两个等腰三角形 10.如图,ΔABC 中,AB=AC,D 为 BC 上一点,BF=CD,CE=BD 那么∠EDF 等于( ) (A)90°-∠A (B)90°-1 2 ∠A (C)180°-∠A (D)45°-∠A 11.求证:三角形一边的两个端点到这边上的中线的距离相等 12.如图,在ΔABC 中,AD 平分∠BAC,DE||AC,EF⊥AD 交 BC 延长线于 F。求证: ∠FAC=∠B 第 20 课 等腰三角形 〖知识点〗 等腰三角形、等腰三角形的性质和判定、等边三角形、等边三角形的性质 和判定、轴对称、轴对称图形 〖大纲要求〗 1. 理解等腰三角形的概念,掌握等腰三角形的两底角相等、等腰三角形三线合一等性 质,掌握两个角相等的三角形是等腰三角形等判定定理,并能运用它们进行简单的 证明和计算; 2. 理解等边三角形的概念,掌握等边三角形的各角都是 60°等性质,掌握三个角都 相等的三角形或一个角是 60°的等腰三角形都是等边三角形等判定,能运用它们 进行简单的证明和计算; 3. 了解轴对称及轴对称图形的概念,会判断轴对称图形。 〖考查重点与常见题型〗 等腰三角形和等边三角形的性质和判定的应用,证明线段、角相等,求线 段的长度、角的度数,中考题中多以选择题、填空题为主,有时也考中档 解答题,如: (1)如果,等腰三角形的一个外角是 125°,则底角为 度; (2)等腰三角形一腰上的高与底边的夹角为 45°,则这个三角形是( ) A.锐角三角形 B.钝角三角形 C.等边三角形 D.等腰直角三角形 〖预习练习〗 1.一个正三角形的边长为 a,它的高是( ) (A) 3 (B) 3 2 (C)1 2 (D) 3 4 2.如果等腰三角形一腰长为 8,底边长为 10,那么连结这个三角形各边的 中点所成的三角形各边的中点形成的三角形的周长为( ) (A)26 (B)14 (C)13 (D)9 3.等腰直角三角形的一条直角边为 1cm,则斜边上的高为 4. 若等腰三角形的底角为 15°,腰长为 2,则腰上的高为 5. 已知等腰三角形的一边等于 4cm,一边等于 9cm,那么它的周长等于 cm 6. 等腰三角形的底边长为 3,周长为 11,则一腰长为 7. 等腰三角形的周长为 2+ 3 ,腰长为 1,底角等于 度 8. 已知如图,在△ABC 中,∠B=90°,AB=BC, BD=CE, M 是 AC 的中点,求证:△DEM 是等腰三角形 考点训练 1.等腰三角形周长是 29,其中一边是 7,则等腰三角形的底边长是( ) (A)15 (B)15 或 7 (C)7 (D)11 2.在△ABC 中,AB=AC,BD 平分∠ABC,若∠BDC=75°,则∠A 的度数为( ) (A)30° (B)40° (C)45 ° (D)60° 3.等腰△ABC 的顶角∠A=15°,P 是△ABC 内部的一点,且∠PBC=∠PCA,则∠BPC 的度 数为( ) (A)100° (B)130° (C)115 ° (D)140° 4.等腰三角形的对称轴有( ) (A)1 条 (B)2 条 (C)3 条 (D)1 条或 3 条 5.在△ABC 中,AB=AC,用∠A 表示∠B,则∠B= 6.如图,CD、BD 平分∠BCA 及∠ABC,EF 过 D 点且 EF∥BC, 则图中的等腰三角形有 个,它们是 7.如图△ABC 中,AB=AC,∠A=36°,BD 平分∠ABC, DE⊥AB 于 E,则∠C= ,∠BDE= , AE= ;若△BDC 周长为 24,CD=4,则 BC= , △ABD 的周长为 ,△ABC 的周长为 8. 等腰三角形一腰上的中线把这个三角形的周长分为 15 厘米和 11 厘米两部分,则此三角形的底边长为 9. 如图,等腰三角形 ABC 中,AB=AC,∠A=90°,BD 平分∠ABC,DE⊥BC 且 BC=10, 求△DCE 的周长。 10.等边三角形 ABC 中,D 是 AC 中点,E 为 BC 延长线一点,且 DB=DE,求证: △DCE 是等腰三角形。 解题指导 1. 如图,在△ABC 中,∠ACB=90°,CD⊥AB 于 D,AF 平分∠BAC 交 CD 于 E,交 BC 于 F, EG∥AB 交 BC 于 G,求证:BG=CF。 2.已知如图△ABC 是边长为 a 的等边三角形,△BCD 的顶角∠BDC=120°,DB=DC 以 D 为 顶点作一个 60°的角,角的两边 DM、DN 分别交 AB 于 M,交 AC 于 N, 连结 MN,求△ABD 的周长。 3.如图在△ABC 中,AE 平分∠BAC,∠DCB=∠B-∠ACB, 求证:△DCE 是等腰三角形。 4.如图在△ABC 中,CD⊥AB 于 D,且 E、F、G 分别是 AC、BC、AB 的中点, 求证:∠DEF=∠BGF 独立训练 1. 在△ABC 中,∠B=36°,D、E 在 BC 边上,且 AD 和 AE 把∠BAC 三等分,则图 中等腰三角形的个数( ) (A)3 (B)4 (C)5 (D)6 2.如图,在△ABC 中,AB=AC,BD=BC,AD=DE=EB, 则∠A 等于( ) (A)30° (B)36° (C)45 ° (D)54° 3.等腰三角形的一个内角为 70°,它的一腰上的高与底边所夹的 角的度数是( ) (A)35° (B)20° (C)35 °或 20°(D)无法确定 4.等腰三角形的顶角等于一个底角的 3 倍,则顶角的度数为 ,底角的度数为 2. 等腰三角形三个内角与顶角的外角之和等于 260°,则它的底角度数为 3. 等腰△ABC 中,AB=AC,BC=6cm,则△ABC 的周长的取值范围是 7.如图,等边△ABC 中,O 点是∠ABC 及∠ACB 的角平分线的交点,OM∥AB 交 BC 于 M,ON∥AC 交 BC 于 N,求证:M、N 是 BC 的三等分点。 8.已知△ABC 中,AB=AC,D、M 分别为 AC、BC 的中点,E 为 BC 延长线上一点,且 CE=1 2 BC, 求证:(1)∠DMC=∠DCM;(2)DB=DE 9.如图,在△ABC 中,∠A=90°,且 AB=AC,BE 平分∠ABC 交 AC 于 F,过 C 作 BE 的垂线 交 BE 于 E,求证:BF=2CE 10.如图,△ABC 为等边三角形,延长 BC 到 D,延长 BA 到 E,AE=BD, 连结 EC、ED,求证:CE=DE 第 21 课 直角三角形 〖知识点〗 直角三角形的性质和判定、逆命题和逆定理、勾股定理及逆定理、角平分线的性质、线 段的中垂线及其性质 〖大纲要求〗 了解逆命题和逆定理的概念;掌握直角三角形中两锐角互余、斜边上的中线等于斜边的 一半及 30°角所对的直角边等于斜边的一半等性质,掌握勾股定理及其逆定理,并能运用 它们进行简单的论证和计算;掌握角平分线的性质定理及其逆定理,线段中垂线性质定理及 其逆定理。 〖考查重点与常见题型〗 直角三角形性质及其判定的应用,角平分线性质定理及其逆定理,线段中垂线的性质定 理及其逆定理的应用,逆命题的概念,中考题中多为选择题或填空题,有时也考查中档的解 答题,如: (1) 在直角三角形中,已知一条直角边的长为 6,斜边上的中线长为 5,则另一条直 角边的长为 (2) 命题“平行四边形的对角线互相平分”的逆命题是 (3) 在△ABC 中,如果∠A-∠B=90°,那么△ABC 是( ) (A)直角三角形(B)锐角三角形(C)钝角三角形(D)锐角三角形或钝角三角形 〖预习练习〗 1.直角三角形的两个锐角的平分线所交成的角的度数是( ) (A) 45° (B)135° (C)45°或 135° (D)以上答案都不对 2.如图 Rt△ABC,∠C=90°,CD⊥AB,CE 是 AB 上的中线, ∠ACD:∠BCD=3:1,若 CD=4cm,则 ED 是( ) C (A) 2cm (B)4cm (C)3cm (D)5cm 3.等腰直角三角形中,若斜边和斜边上的高的和是 6cm, A B 则斜边长是 cm E D 4.三角形三个角的度数之比为 1:2:3,它的最大边长等于 16cm,则最小边长是 cm A 5.如图,△ABC 中,AB=AC,∠BAC=120 度,AD⊥AC, DC=5,则 BD= 6.AD 是 Rt△ABC 斜边上的高,已知 AB=5cm,BD=3cm , B D C 那么 BC= cm 7.如图,△ABC 中,AB=AC,DE 是 AB 的中垂线, A △BCE 的周长为 14cm, BC=5cm,求 AB 的长。 D E B C 考点训练: 1.如果三角形中有一条边是另一条边是 2 倍,并且有一个角是 30°,那么这个三角形是( ) (A) 直角三角形 (B)锐角三角形 (C)钝角三角形 (D) 图形不能确定 2.如图,RtΔABC 中,∠BCA=90°, ∠A=30°CD⊥AB 于 D,DE⊥BC 于 E, 则 AB:BE 的值为( ) (A) 8 (B) 4 (C) 5 2 (D) 3.5 3.等腰三角形一腰上的高与底边所成的角等于( ) (A) 顶角的 2 倍 (B) 顶角的一半 (C) 顶角 (D) 底角的一半 4.在直角三角形中,两锐角的平分线相交成钝角的度数是 . 5.直角三角形中,一条直角边比斜边上的中线长 1 厘米,如果斜 边长是 10 厘米, 则两直角边长是 . 6.已知:如图,在ΔABC 中,AB>AC, D 点在 AB 上, AD=AC,AM⊥CD 于 M,E 为 BC 的中点,若 AB=16,AC=10, 则 EM 的长为 7.有一个角为 30°的等腰三角形,若腰长为 4,则腰上的高是 , 面积是 . 8.如图,在 RTΔABC 中, 斜边 AB 的中垂线 DE 交 BC 于 D,连结 AD, 若∠1:∠2=2:5,求∠B、∠BAC 的度数. 9.如图,在ΔABC 中, ∠BCA=90°,且 AC=BC, 直线 L 过 C 点,AE⊥L 于 E, BF⊥L 于 F. 求证:EF=AE+BF 解题指导 1. 如图,在ΔABC 中, ∠ABC=2∠C,AD⊥BC 于 D,E 是 AC 中点,ED 的 延长线与 AB 的延长线交于点 F,求证:BF=BD 2.如图,在ΔABC 中,∠B=40°, ∠C=20°, AD⊥CA 于 A, 交 BC 于 D, 求证:CD=2AB 3.如图,AB⊥a 于 B,DC⊥a 于 C,∠BMA=75°, ∠DMC=45°,AM=DM, 求证:AB=CB 4.如图,在四边形 ABCD 中,BC=DC,AC 平分∠BAD,CE⊥AB, CF⊥AD,E、F 为垂足,若 AB=21,AD=9,BC=DC=10,求 AC 的长. 独立训练: 1.如图,在ΔABC 中,AD 是∠BAC 的平分线,DM⊥AB 于 M,DN⊥AC 于 N, 连接 MN,则图中等腰三角形有 个,直角三角形有 个 2.如图,在 RTΔABC 中, ∠B=90°, AD 为 BC 边中线,DE⊥AC 于 E, 则:AB2+EC2 AE2 3.已知:如图,AD∥BC,F 是 AB 中点,DF 交 CB 延长线于点 E,CE=CD,则图中与∠ADE 相等的角 有 , 与∠ADE 互余的角有 . 4.已知:如图,在四边形ABCD 中,M、N 分别是 CB、CD 中点,且 AM⊥BC 于M,AN⊥CD于 N, ∠MAN=80°, 则∠B+∠D 的度数是 (3) (4) (5) 5.如图,在ΔABC 中 , ∠B=2∠C,AD⊥BC 于 D,M 为 AB 边中点,ME∥AC 交 BC 于 E,则 AB 是 DE 的 倍. 6.如图,AB∥CD,E 是 AD 中点,CF⊥AB 于 F。求证:CE=EF 7.如图,A、C 是∠MON 的 OM 边上两点,AB⊥ON 于 B,CD⊥ON 于 D, 若 OA=1 2 ,OB=CD,且 OD+AB=1.求∠MON 的度数 第 22 课 平行四边形及特殊平行四边形 〖知识点〗四边形、四边形的内角和与外角和、多边形、多边形的内角和与外角和、平行四 边形、平行四边形的性质和判定、两条平行线间的距离、矩形、菱形、正方形的性质和判定。 〖大纲要求〗 1. 理解多边形,多边形的顶点、边、内角、外角及对角线等概念,理解多边形的理解 和定理,掌握四边形的理解和和外角和都是 360°的性质; 2. 了解两点间的距离。点到直线的距离与两条平行线之间的距离及三者之间的联系, 了解平行四边形不稳定性的应用,理解两条平行线间的距离概念; 3. 掌握平行四边形、矩形、菱形、正方形等概念,掌握平行四边形、矩形、菱形、正 方形的性质和判定,通过定理的证明和应用的教学,使学生逐步学会分别从题设和结论出发, 寻找论证思路分析法和综合法,进一步提高分析问题,解决问题的能力。 〖考查重点与常见题型〗 1. 考查特殊四边形的判定、性质及从属关系,此类问题在中考中常以填空题或选择题 出现,也常以证明题的形式出现。如: 下列命题正确的是( ) (A) 一组对边相等,另一组对边平行的四边形一定是平行四边形 (B) 对角线相等的四边形一定是矩形 (C) 两条对角线互相垂直的四边形一定是菱形 (D) 两条对角线相等且互相垂直平分的四边形一定是正方形 2. 求菱形、矩形等的面积,线段的长,线段的比及面积的比等,此类问题以不同种题 型常以如选择题,填空题出现,也常以论证题型和求解题型出现。如: 若菱形的周长为 16cm,两相邻角的度数之比是 1:2,则菱形的面积是( ) (A) 4 3 cm (B)8 3 cm (C)16 3 cm (D)20 3 cm 3. 三角形和四边形与代数中的函数综合在一起 4. 求多边形的边数、内角和、外角和及正多边形的角、边长及半径、边心距,以正五 边形、正六边形为常见,多见于填空题和选择题,如: (1)正五边形的每一个内角都等于 度 (2)若正多边形的边心距与边长的比是 1:2,则这个正多边形的边数是 (3)已知正六边形的边长是 2 3 ,那么它的边心距是 〖预习练习〗 在线段、角、等腰三角形、等边三角形、平行四边形、矩形、菱形、正方 形、梯形、直角梯形、等腰梯形、圆、正五边形、正六边形中,既是中心 对称图形又是轴对称图形的是 考点训练 1. 已知:平行四边形 ABCD 的周长是 30cm,对角线 AC,BD 相交于点 O,⊿AOB 的周长比 ⊿BOC 的周长在 5cm ,则这个平行四边形的各边长为_____。 2. 已知:平行四边形 ABCD 中,AC=2cm,BD=6cm,CA⊥AB,则平行四边形的周长是_ ____,面积______。 3. 已知:平行四边形 ABCD 中, AE⊥BC 交 CB 的延长线于点 E,AF⊥CD 交 CD 的延长线 于点 F,AB+BC+CD+DA=32cm,BC=3 5 AB,∠EAF=2∠C,则 BE 长为____,则 ∠C____。 4. 已知:如图,矩形 ABCD 中,AC,BD 交于点 O,AE⊥BD 于 E, AB=2cm,BD=4cm,则 AC 长为____BE 长为____, ∠ADB 度数为____∠BAD 度数_____。 5. 如图:平行四边形 ABCD 中 AB>AD, AE,BF,CG,DH 是各内角的角平分线, 分别交于 CD,AB 于 E,F,G,H,DH 与 AE, CG 交于 P,M,BF 与 AE,CG 交于 N,G, 求证:AB=AD+PQ 6. 已知:如图,⊿ABC 中,∠BAC=90°,AD 是高,BE 平分 ∠ABC 交 AD 于 M,AN 平分∠DAC,求证:平行四边形 AMNE 是菱形。 解题指导: 1. 已知:平行四边形 ABCD 是,E,F 分别是 AB,CD 的中点,AF,DE 交于 G,BF,CE 交 于点 H,求证:平行四边形 EHFG 是平形四边形。 2. 已知:⊿ABC 中,∠ACB=90°,∠CBA=30°,⊿ABD,⊿BCE 均是在⊿ABC 外的等边 三角形,DE 交 AB 于点 F,求证:DF=EF。 3. 已知:⊿ABC 中,AB=BC,∠ABC=90°,D 是 AC 上一点,DE⊥AB 于 E,DF⊥BC 于 G, A D CB E O D F E C P N Q M G HA BB D N CA M E A D N CMB P 是 AC 的中点,求证:PE=PF。 4. 已知:如图,在正方形 ABCD 中,M,N 分别是 BC,CD 上的点。 (1) 若∠MAN=45°,求证:MB+ND=MN 。 (2) 若 MB+ND=MN,求证:∠MAN=45°。 独立训练(一) 1. 一个多边形内角和等于它的外角和的二倍,遇这个多边形的边数为___。 2. 若多边形的边数增加 2,则该多边形的内角和增加____。 3. 若一个多边形的每个内角都为钝角,则边数最少是____。 4. 四边形四个内角之比 1:2:3:4,则这四个角中最小的一个为____度。 5. 在平形四边形 ABCD 中,BC=2AB,点 E 为 BC 的中点,则∠AED 的度数为___。 6. 若平形四边形两邻边长为 6,8,夹角为 30°,则这外平形四边形面积是_ 7. 若正方形的对角线长为 2 2 cm,则正方形的面积为___。 8. 若菱形的边长是它的高的 2 倍,则它的一个较小内角的度数是___。 9. 矩形两条对角线的交角是 60°,一条对角线与较短边的和是 15,则对角线长___。 10. 若矩形一个内角的平分线,把另一边分为 4cm,5cm 两部分,遇这个矩形周长是___ 11. 已知:正方形 ABCD 的边长的 12,点 P 在 BC 上,BP=5,PE⊥AP,交 CD 于点 E,则 DE 的长为____。 12. 如图:在平形四边形 ABCD 中,BM 平分∠ABC,且 M 为 AD 的中点, 13. 求证:CM 平分∠BCD。 14. 如图,ABCD 是正方形,CE∥BD,BE=BD,BE 交 DC 于点 F, 求证:(1)∠BEC=30° (2)DE=DF 独立训练(二) 1.两个全等的三角形(不等边)可拼成不同的平形四边形的个数是( ) (A)1 (B)2 (C)3 (D)4 2.延长平形四边形 ABCD 的一边 AB 到 E,使 BE=BD,连结 DE 交 BC 于 F,若∠DAB=120°, ∠CFE=135°,AB=1,则 AC 的长为( ) (A)1 (B)1.2 (C) 3 2 (D)1.5 3.若菱形 ABCD 中,AE 垂直平分 BC 于 E,AE=1cm,则 BC 的长是( ) (A)1cm (B)2cm (C)3cm (D)4cm 4.若顺次连结一个四边形各边中点所得的图形是正方形,那么这个四边形的对角线( ) (A) 互相垂直 (B)相等 (C)互相平分 (D)互相垂直且相等 5.正方形 ABCD 的边长为 1,M 是 AB 的中点,N 是 BC 中点,AN 和 CM 相交于点 O,则四边形 AOCD 的面积是( ) (A)1 6 (B)3 4 (C)2 3 (D) 3 4 6.下列结论中错误的是( ) (A) 五边形最少有两个钝角。 (D)立边形共有九条对角线。 (B) 任意四边形一组对边中点的边线长不大于另一组对边长度和的一半。 (C) 平行四边形即是轴对称图形又是中心对称图形。 7.如图,已知⊿DAB,⊿EAC, ⊿FBC 都是等边三角形, 求证:四边形 DECF 为平等四边形。 8.如图,E 是矩形 ABCD 边 CB 延长线上一点,CE=CA,F 是 AE 的中点。 A M D CB A D CB F E E D C F A B E D C F A B 求证:BF⊥FD 独立训练(三) 1.如图,平形四边形 ABCD 周长这 32cm,AB:BC=5:3,AE⊥CD 于 F 且∠EAF=2∠C 求 AE 和 AF 的长 2.如图,菱形 ABCD,E,F 分别是 BC,CD 上的点,∠B=∠EAF=60°, ∠BAE=18°求∠CEF 的度数。 3.如图,正方形 ABCD 中,E,F 分别为 AD,DC 的中点,BF,CG 相交于点 M,求证:AM=AB 4.如图,BF,BE 分别是∠ABC 及它的邻补角的平分线,AE⊥BE 于 E,AF⊥BF 于 F,EF 分别交 AB,AC 于 M,N 求证:(1)AEBF 为矩形 (2)MN=1 2 BC 第 23 课时 梯形 知识点:梯形、等腰梯形、直角梯形、等腰梯形的性质和判定、四边形的分类 大纲要求: 1. 掌握梯形、等腰梯形、直角梯形的概念,等腰梯形的性质和判定; 2. 四边形的分类和从属关系。 考查重点与常见梯形 1. 考查梯形的判定、性质及从属关系,在中考题中常以选择题或填空题出现,也常以证明 题的形式出现。如: (A) 圆内接平行四边形是矩形; (B) 一组对边平行另一组对边不平行的四边形一定是梯形; (C) 顺次连结等腰梯形各边中点构成的四边形是菱形; (D) 两条对角线互相垂直且相等的四边形是正方形。 2. 求梯形的面积、线段的长,线段的比及面积的比等,在中考题中常以选择题或填空题出 现,也常以证明题的形式出现。 如:如图梯形 ABCD 中,AD∥BC,AC、BD 交于 O 点,S ⊿AOD:S⊿COB=1:9,则 S⊿DOC:S⊿BOC= 3. 梯形与代数中的方程、函数综合在一起, 如在直角梯形 ABCD 中,AD∥BC,AB⊥AD,AB =10 3 ,AD、BC 的长是 x2-20x+75=0 方程的两根,那么以点 D 为圆心、AD 长为半径的 圆与以 C 圆心,BC 为半径的圆的位置关系是 。 预习练习 1. 梯形两底的差是 4,中位线长是 8,则上底是 ,下底长是 。 2. 等腰梯形有一个角是 60°,上下底长分别是 2cm 和 6cm,则腰长为 。 3. 若梯形的中位线被它的两条对角线三等分,则梯形的上底 a 与下底 b(an>0),求梯形中位线 MN 的长 2. 如图,梯形 ABCD 中,AD∥BC,∠ B+∠ C=90°,E、F 分别是 AD、BC 的中点,求证:EF=1 2 (BC-A 3. 在梯形 ABCD 中,AD∥BC,AB=AD+BC,E 为 CD 中点, 求证:AE 平分∠ DAB。 4. 如图 ABCD 是等腰梯形,AB∥DC,AD=BC。P 是 CD 上任意一点, 过点 P 作 AD,BC 的平行线,分别交对角线 AC,BD 于点 E、F, 求证:PE+PF=AD。 5. 如图,过⊿ABC 的顶点 A,任作一条直线 AD,作 BE⊥AD,CF⊥AD,E、F 为垂足,M 是 BC 的中点,求证:ME=MF。 独立训练: 1.等腰梯形的下底是上底的 3 倍,上底与高相等,则下底角的度数为( ) (A)30 ° (B)45° (C)60° (D)75° 2.若梯形 ABCD 中,AB∥DC,AB=5,BC=3 2 ,∠BCD=45°,∠CDA=60°,则 DC 等于 ( ) (A)7+2 3 (B)8 (C)8+ 3 (D)8+3 3 3.若梯形的两条对角线分中位线为三等分,则梯形的上、下底之比为( ) (A)1:3 (B)2:3 (C)3:5 (D)1:2 4. 已 知 直 角 梯 形 的 高 为 h , 中 位 线 长 为 m 。 一 个 底 角 为 150 ° , 则 梯 形 的 周 长 为 . 5. 等腰梯形的两底长为 4cm 和 10cm,一底角为 45°, 则它的面积为 6. 如图直角梯形 ABCD 中,AD∥BC,∠ABC=90°, AD:BC=1:4,则 BD:AC= 7. 如图,梯形 ABCD 中,AB∥DC,对角线 BD⊥AB,已知两底 与高的和为 16cm,梯形面积为 32cm2,求 AC 的长。 8. 如图,点 E 在正方形 ABCD 的对角线 AC 上,CF⊥BE 交 BD 于点 G,F 是垂足,求证:四边 形 ABGE 是等腰梯形。 9. 如图,梯形 ABCD 中,AD∥BC,BD 为对角线,S⊿ADB:S⊿DBC=3:7,求中位线 EF 将梯形分 成的两部分面积之比。 第 24 课 中位线与面积 〖知识点〗 平行线等分线段、三角形、梯形的中位线、三角形、平行四边形、矩形、矩形、正方 形、梯形的面积、等积变形、几何变换(平移、旋转、翻折) 〖考查要求〗 1. 掌握平行线等分线段定理,三角形、梯形中位线定理,三角形一边中点 且平行另 一边的直线平分第三边,过梯形一腰的中点且平行底的直线平分另一腰的定理; 2. 使学生了解面积的概念,掌握三角形、平行四边形、矩形、菱形、正方形、梯形的 面积公式,等底等高的三角形面积相等的性质,会用面积公式解决一些几何中的简 单问题; 3. 使学生掌握几何证题中的平移、旋转、翻折三种变换。 〖考查重点与常见题型〗 1. 考查中位线、等分线段的性质,常见的以选择题或填空题形式,也作为基础知识应 用,如: 一个等腰梯形的周长是 100cm,已知它的中位线与腰长相等,则这个题型的中位线是 2. 考查几何图形面积的计算能力,多种题型出现,如: 三角形三条中位线的长分别为 5 厘米,12 厘米,13 厘米,则原三角形的面积是 厘米 2 3. 考查形式几何变换能力,多以 中档解答题形式出现 〖预习练习〗 1.顺次连结对角线相等的四边形各边中点所得的四边形是( ) (A) 矩形 (B)等腰梯形 (C)菱形 (D)正方形 2.在四边形 ABCD 中,AC=BD,厘米顺次连结四边形 ABCD 各边中点所得的四边形一定 是( ) (A)平行四边形 (B)矩形 (C)正方形 (D)菱形 3.正方形的对角线的长为 6cm,则正方形的面积是 cm2 4.菱形的两条对角线之比是2:3,面积是15厘米 2,则两条对角线的长分别是 厘 米和 厘米 5.一个三角形和一个梯形的面积相等,它们的高也相等,已知三角形德国底边为 18 厘米,厘米梯形的中位线的长等于 厘米 6.△ABC 中,若 D 是 BC 边的中点,则 S△ACD= =1 2 ;若 BD:DC=3:2, 则 S△ABD:S△ACD= 考点训练: 1.等腰三角形腰长为 2,面积为 1,则顶角大小是( ) (A) 90° (B) 30° (C) 60° (D) 45° 2.如图,G 是△ABC 的重心(三角形中线的交点), 若 S△ABC=6,则的面积是( ) (A) 4 3 (B) 1 (C) 2 (D) 3 4 3.如图,AB∥DC,ED∥BC,AE∥BD,则图中和△ABD 面积相等的三角形个数(不包括△ABD) 为( ) (A) 1 (B) 2 (C) 3 (D) 4 4. 矩形两邻边的长是 4cm,6cm,顺次连结它的四边中点所得的四边形面积是 ______cm2 . 5.若等边三角形的边长为 a,则它的面积为____________. 6.菱形的边长为 5cm,一条对角线长为 8cm,则它的面积是__________. 7.等腰梯形的中位线长为 m,且对角线互相垂直,则此梯形的面积为____. 8.四边形 ABCD 为平行四边形,P,Q 分别是 AD,AB 上的任意点,则 S△PBC 与 S△QCD 有什么 关系?它们与原平行四边形的面积之间有什么关系? 9.在△ABC 中,AB=10,BC=5 5 ,AC=5,求∠A 的平分线的长。 10.如图,在△ABC 中,AD 为角平分线,CE⊥AD,F 为 BC 中点, 求证:EF=1 2 (AB – AC). 解题指导: 1.已知:如图,△ABC 中,AD 是 BC 上的中线,E 是 AD 中点,BE 的延长线交 AC 于 F。 求证:EF=1 3 BE. A B CD G A B CD E A B CD E F A B CD FE 2.已知:如图,△ABC 中,BD,CE 分别平分∠B 和∠C,P 是 DE 中点,过点 P 作 BC,CA,AB 的垂线,垂足分别为 L,M,N,求证:PL=PM+PN. 3.证明以梯形一腰的中点及另一腰的两个端点为顶点的三角形面积等于原梯形面积的 一半。 4. 如图,在△ABC 中,D 是 BC 中点,N 是 AD 中点,M 是 BN 中点,P 是 MC 的中点。求 证:S△MNP=1 8 S△ABC. 独立训练: 1. 如图,△ABC 中,DE∥BC,且 S△ADE∶S△ABC=1∶2, 则 AD∶DB 等于( )。 (A) 1 2 (B) 1 2 (C) 2 – 1 (D) 2 + 1 2.已知三角形的一边长为 2,这边上的中线长为 1,另外两边和为 1+ 3 , 则此三角形面积为( )。 (A) 3 (B) 2 2 (C) 3 2 (D) 2 3.矩形 ABCD 中,AD=5,AB=12,O 为对角线 AC,BD 的交点,E 为 BC 延长线上一点, 且 CE=AC,则 S△OCE=____________. 4. 已知∠POQ 内有一点 A,求作△ABC,使△ABC 的周长最小,且顶点 B,C 分别在 OP,OQ 上。 5.如图,AB=DE,直线 AE,BD 相交于点 O,∠B 与∠D 相等, 求证:AO=EO. 6.如图,ABCD 为正方形,E 为 CD 的中点,过 E 作 EF,使∠AEF=∠BAE,EF 交 BC 于, 求证:CF=2BF. 7.如图,在平行四边形 ABCD 中,E 是 BC 的中点,DE,AB 的延长线交于点 F,求证:S△ ABE=S△EFC. A B C DE L P M N A B CD P M N A B C ED Q O A P A B D E O A B CD E F A B C D E F 第 25 课 比例线段 〖知识点〗 比与比例、比例的基本性质、合比性质、等比性质、两线段的比、成比例线段、平行 线分线段成比例、截三角形两边或其延长线的直线平行于第三边的判定、黄金分割 〖大纲要求〗 1.理解比与比例及比例中项等概念,掌握比例的基本性质、合比定理和更比定理,会 用它们进行简单的比例变形; 2.理解比例线段及黄金分割的概念,理解平行线分线段成比例定理,会作第四比例项 〖考查重点与常见题型〗 1.考查比例的性质,常以选择题或填空题出现,如: (1) 已知 a=4,b=9,则 a、b 的比例中项是 (2) 已知线段 a=4cm,b=9cm,线段 c 是 a、b 的比例中项,则线段 c 的长为 2.求线段的比、面积的比,在中考题中常以选择题、填空题或求解题型出现,如图, 已知 DE∥BC,CD 和 BE 相交于 O, S△DOE:S△COB =4:9,则 AE:EC 为( ) 〖预习练习〗 1.若互不相等的四条线段的长 a,b,c,d 满足a b =c d ,m 为任意实数,则下列各式中,相 等关系一定成立的是( ) (A)a+m b+m =c+m d+m (B)a+b b =c+d c (C)a c =d b (D)a-b a+b =c-d c+d 2.如图,已知△ABC 中,DE∥BC,则下列等式中不成立的是( ) (A)AD:AB=AE:AC (B)AD:DB=AE:EC (C)AD:DB=DE:BC (D)AD:AB=DE:BC 3.如图,△ABC 中,DE∥FG∥BC,AD:DF:FB=3:2:1, 则△ADE,四边形 DFGE,四边形 FBCG 的面积比是( ) (A)3:2:1(B)9:4:1(C)9:16:11(D)9:25:36 4.已知(-3):5=(-2):(x-1),则 x= 5.若 x 是 3、4、9 的第四比例项,则 x= , 又 x 是 6 和 y 的比例中项,则 y= 6.已知a b =c d =e f =3 5 ,b+d+f=50,那么 a+c+e= 7.如果x y =7 3 ,那么x-y y = ,x+y y = , x+y x+y = 考点训练: 1、若3 x =x 4 ,则 x 等于( ) (A)12 (B)2 3 (C)- 2 3 (D)±2 3 2、已知 y 是 3,6,8 的第四比例项,则 y 等于( ) (A)4 3 (B)16 (C)12 (D)4 3、若(m+n):n=5:2,则 m:n 的值是( ) (A)5:2 (B)2:3 (C)3:2 (D)2:5 4、如图,DF∥AC,DE∥BC,下列各式中正确的是( ) (A) AD BD =BF CF (B) AE DE =CE BC (C) AE CE =BD CD (D) AD DE =AB BC (4) (8) 5、把 m=ab c 写成比例式,且使 m 为第四比例项 ; 6、若线段 a=5cm,b=10cm,c=4dm,d=2cm,它们是否成比例线段 ; 7、已知x y =5 3 ,则(x+y):(x-y)= ; 8、如图,已知ΔABC 中,DE∥BC,AC=7cm,CE=3cm,AB=6cm,则 AD= ; 9、如图,已知梯形 ABCD 中,AD∥BC,AC,BD 交于 O,过 O 作 AD 的平行线交 AB 于 M,交 CD 于 N,若 AD=3cm,BC=5cm,求 ON. 10、如图,已知平行四边形 ABCD 中,G 是 DC 延长线上一点,AG 交 BD 和 BC 于 E,F,求证: AE EF =EG AE 解题指导 1、(1)已知 a:b:c=2:3:7,且 a-b+c=12,求 2a+b-3c 的值; (2)已知b+c a =c+a b =a+b c ,求a+b c 的值。 2、如图,已知ΔABC 中,DE∥BC,AD2=AB•AF,求证∠1=∠2 3、已知ΔABC 中,AD 为∠BAC 的外角∠EAC 的平分线,D 为平分线与 BC 延长线交点,求证:AB AC = BD DC 4、已知,如图,ΔABC 中,直线 DEF 分别交 BC,AD 于 D,E,交 BA 的延长线于点 F,且BD CD = BF CE , 求证 AF=AE 5、已知,在梯形 ABCD 中,AD∥BC,点 E,F 分别在 AB,AC 上,EF∥BC, A B C D E F A B C D E A B C D M N O A B C D E F 1 2 A B C D E A B CD E F A B C D E F G EF 交 AC 于 G,若 EB=DF,AE=9,CF=4,求 BE,CD, GF AD 的值。 独立训练 1、若a b =c d ,下列各式中正确的个数有( ) a d =c d , d:c=b:a, a b =a2 b2 , a b =c+5 d+5 , a b =a+c a+d , c d =ma mb (m≠0) (A)1 (B)2 (C)3 (D)4 2、已知线段 a,m,n,且 ax=mn,求作 x,图中作法正确的是( ) (A) (B) (C) (D) 3、如果 D,E 分别在ΔABC 的两边 AB,AC 上,由下列哪一组条件可以推出 DE∥BC (A) AD BD = 2 3 ,CE AE = 2 3 (B)AD AB = 2 3 ,DE BC = 2 3 (B) AB AD = 3 2 ,EC AE = 1 2 (D) AB AD = 3 4 ,AE EC = 4 3 4、已知 S 正方形=S 矩形,矩形的长和宽分别为 10cm 和 6cm,则正方形的边长为 5、在 RtΔABC 中,∠C=90°, ∠A=30°则 a:b:c= 6、已知 x:y=2:3,则(3x+2y):(2x-3y)= 7、已知 5x-8y=0,则x+y x = 8、已知x 5 =y 3 =z 4 ,则2x+y-z x+3y+z = 9、已知 5x+y 3x-2y =1 2 ,则x y = , x+y x-y = ; 10、已知线段 AB 长为 1cm,P 是 AB 的黄金分割点,则较长线段 PA= ; PB= ; 11、设点 F 在平行四边形 ABCD 的边 CB 的延长线上,DF 交 AB 于点 E,求证, AE:AD=AB:CF 12、在梯形 ABCD 中,AD∥BC,点 E 在 BD 的延长线上,且 CE∥AB,AC 与 BD 相交于点 O, 求证:OB2=OD•OE 第 26 课 相似三角形 〖知识点〗 相似三角形、相似三角形的判定、直角三角形相似的判定 〖大纲要求〗 1. 了解相似三角形的概念,掌握相似三角形的判定及直角三角形相似的判定; 2. 会用相似三角形证明角相等或线段成比例,或进行角的度数和线段长度的计算等 〖考查重点与常见题型〗 1.论证三角形相似,线段的倍分以及等积式,等比式,常以论证题型 或计算题型出现; 3. 寻找构成三角形相似的条件,在中考题中常以 选择题或填空题形式出现,如:下 a m x n a m x n a m x n a m x n 列所述的四组图形中,是相似三角形的个数是( ) 1 有一个角是 45°的两个等腰三角形;②两个全等三角形;③有一个角是 100°的两 个等腰三角形;④两个等边三角形。 (A)1 个 (B)2 个 (C)3 个 (D)4 个 〖预习练习〗 1. 点 P 为△ABC 的 AB 边上一点(AB>AC),下列条件中不一定能保证△ACP∽△ABC 的是( ) (A)∠ACP=∠B(B)∠APC=∠ACB(C)AC AB =AP AC (D)PC BC =AC AB 2.下列各组的两个图形,一定相似的是( ) (A) 两条对角线分别对应成比例的两个平行四边形 (B) 等腰梯形的中位线把它分成的两个等腰梯形 (C) 有一个角对应相等的两个菱形 (D) 对应边成比例的两个多边形 3. 如图,在△ABC 中,∠BAC=90°,AD⊥BC,垂足 为 D,DE⊥BC,垂足为 E,则图中与△ADE 相似的三角 A 形个数为( ) (A)1 (B)2 (C)3 (D)4 E 4. M 在 AB 上,且 MB=4,AB=12,AC=16, B D C 在 AC 上有一定 N,使△AMN 与原三角形相似,则 AN 的长为 5. 如图,△ABC 中,DE∥AC,BD=10,DA=15, A BE=12,则 EC= ,DE:AC= , D S△BDE:S 梯形 ADEC= B E C 考点训练 1.以下条件为依据,能判定△ABC 和△A1B2C3 相似的一组是( ) (A) ∠A=45°,AB=12cm,AC=15cm, ∠A´=45°,A´B´=16cm,A´C´=25cm (B) AB=12cm,BC=15cm,AC=24cm, A´B´=20cm,B´C´=25cm,A´C´=32cm (C)AB=2cm,BC=15cm, ∠B=36°, A´B´=4cm,B´C´=5cm, ∠A´=36° (D) ∠A=68°,∠B=40°∠A´=68°,∠B´=40° 2.如图,△ABC 中 DE,DF,EG 分别平行于 BC,AC,AB, 图中与△ADG 相似的三角形共有( )个 (A) 3 (B) 4 (C) 5 (D) 6 3.如图,已知 D,E 分别在△ABC 的 AB,AC 边上,△ABC 与△ADE 则下列各式成立的是( ) (A) AD BD = AE CE (B) AD AB = DE BC (C) AD·DE=AE·EC (D) AB·AD=AE·AC 4.如图,已知△ABC 与△ADE 中,则∠C=∠E, ∠DAB=∠CAE ,则下列各式成立的个数是( ) ∠D=∠B ,AF AC = AD AB , DE BC = AE AC , AD AE = AB AC (A) 1 个 (B) 2 个 (C)3 个 (D)4 个 A GD CFEB A D C E B A D C F E B A D CB 5.如图,梯形 ABCD 中,AD∥BC,AB⊥AD, 对角线 BD⊥DC,则△ABD∽ , BD2= . 6.如图,∠1=∠2,AB·AC=AD·AE,则∠C= . 7.如图△ABC 中,DE∥BC,AD∶DB=3∶2, 则△ADE 与△ABC 的面积比为 . 8.如图,△ABC 内接正方形 DEFG,AM⊥BC 于 M, 交 DG 于 H,若 AH 长 4cm,正方 形边长 6cm,则 BC= . 9.如图,已知△ABC 中 CE⊥AB 于 E,BF⊥AC 于 F, 求证:△AFE∽△ABC 10.如图,平行四边形 ABCD 中,E 是 CB 延长线上一点, DE 交 AB 于 F, 求证:AD·AB=AF·CE 解题指导 1.M 在 AB 上,且 MB=4,AB=12,AC=16.在 AC 上求作一点 N, 使△AMN 与原三角形相似,并求 AN 的长. 2.在△ABC 中,AB=AC, ∠A=36°,∠ABC 的平分线 BD 与 AC 交于 D,求证: (1) BC=AD (2) △ABC∽△BDC (3)BC=1 2 ( 5 –1)AB 3.如图,已知 BD 和 CE 是△ABC 的高,∠BAC 的平分线交 BC 于 F ,交 DE 于 G, 求证:BF·EG=CF·DG. 4.如图,在△ABC 中, ∠C=90°,AE 平分∠A 交 BC 于 E,CD⊥AB 于 D,交 AE 于 F, FM∥AB 交 BC 于 M,求证(1) AE AF = AB AC (2) EB MB = AE AF (3)CE=BM 5.如图,△ABC 的∠A 的内角平分线交 BC 于 P, ∠BAC 的外角平分线交 BC 的延长线于 Q,M 为 PQ 的中点,求证:(1)MA2=MB·MC (2) MB MC = AB2 AC2 独立练习 1,如图,梯形 ABCD 中,AB∥CD,AC,BD 交于 O 点, BE∥AD 交延长线于 E,相似三角形的对数是( ) (A) 1 (B) 2 (C) 3 (D) 4 2.如图,已知∠1=∠2=∠3,则下列关系正确的是( ) (A) AB AD = BC AE (B) AC AE = BC AD (C) BE DE = AC AE (D) AC AE = AB AD 3.两个直角三角形一定相似; 两个等腰三角形一定相似; A D C E B 1 2 A CFEB D GH M A C F E B A CB M A CB D A CF E B D G A C F E B D M A CB MP Q A C E B D O A C E B D 1 2 3 A C F EB D A C F E B D 两个等腰直角三角形一定相似;两个顶角相等的等腰 三角形一定相似。以上说法正确的共有( )个 (A) 2 (B) 3 (C) 4 (D) 5 4.如图,已知,平行四边形 ABCD,CE=1 2 BC,S△AFD=16cm2 , 则 S△CEF= ,平行四边形 ABCD 的面积___ 5.两个相似三角形对应中线之比是 3:7,周长之和为 30cm, 则它们的周长分别是 6.如图,已知∠ACB=∠E,AC=6,AD=4,则 AE= 7.如图,已知 AB AD = BC DE = AC AE ,求证:△ABD∽△ACE 8.如图,已知梯形 ABCD 中, AB∥BC,AC,BD 交于 E, 民过 E 作 FG∥BC,求证:EF=EG. 9.如图,已知矩形 ABCD 的对角线 AC,BD 相交于 O,OF⊥AC 于 O, 交 AB 于 E,交 CB 的延长线于 F,求证:OB 是 OE 与 OF 的比例中项. 10.如图,直线交△ABC 的 BC,AB 两边于 D,E,与 CA 延长线交于 F,若BD DC = FE ED =2,求 BE:EA 的比值. 第 27 课 相似三角形性质及其应用 知识点 相似三角形性质,直角三角形中成比例线段 大纲要求 1.掌握相似三角形对应高线的比,对应中线的比和对应角平分线的比都等于相似比,相似三 角形面积的比等于相似比的平方等性质,能应用他们进行简单的证明和计算。 2.掌握直角三角形中成比例的线段:斜边上的高线是两条直角边在斜边上的射影的比例中项; 每一条直角边是则条直角边在斜边上的射影和斜边的比例中项,会用他们解决线段成比例的 简单问题。 考查重点与常见题型 1.相似三角形性质的应用能力,常以选择题或填空形式出现,如: 若两个相似三角形的对应角的平分线之比是 1∶2,则这两个三角形的对应高线之比是---------, 对应中线之比是------------,周长之比是---------,面积之比是-------------,若两个相似三角形的面积 之比是 1∶2,则这两个三角形的对应的角平分线之比是----------,对应边上的高线之比是-------- 对应边上的中线之比是----------,周长之比是--------------, 2.考查直角三角形的性质,常以选择题或填空题形式出现,如: 如图,在 RtΔABC 中,∠ACB=90°, CD⊥AB 与 D,AC=6,BC=8, 则 AB=--------,CD=---------, A C E B D A C E B D A C F E B D G A CF E B D O A C F E B D AD=---------- ,BD=-----------。, 3.综合考查三角形中有关论证或计算能力,常以中档解答题形式出现。 预习练习 1.已知两个相似三角形的周长分别为 8 和 6,则他们面积的比是( ) 2.有一张比例尺为 1 4000 的地图上,一块多边形地区的周长是 60cm,面积是 250cm2,则 这个地区的实际周长-------- m,面积是----------m2 3.有一个三角形的边长为 3,4,5,另一个和它相似的三角形的最小边长为 7,则另一个 三角形的周长为----------,面积是------------- 4.两个相似三角形的对应角平分线的长分别为 10cm 和 20cm,若它们的周长的差是 60cm, 则较大的三角形的周长是----------,若它们的面积之和为 260cm2,则较小的三角形的面积 为---------- cm2 5.如图,矩形 ABCD 中,AE⊥BD 于 E,若 BE=4,DE=9,则矩形的面积是----------- 6.已知直角三角形的两直角边之比为 12,则这两直角边在 斜边上的射影之比------------- 考点训练 1.两个三角形周长之比为 95,则面积比为( ) (A)9∶5 (B)81∶25 (C)3∶ 5 (D)不能确定 2.RtΔABC 中,∠ACB=90°,CD⊥AB 于 D,DE⊥AC 于 E,那么和ΔABC 相似但不全等的三角 形共有( ) (A)1 个 (B)2 个 (C)3 个 (D)4 个 3.在 RtΔABC 中,∠C=90°,CD⊥AB 于 D,下列等式中错误的是( ) (A)AD• BD=CD2 (B)AC•BD=CB•AD (C)AC2=AD•AB (D)AB2=AC2+BC2 4.在平行四边形 ABCD 中,E 为 AB 中点,EF 交 AC 于 G,交 AD 于 F,AF FD =1 3 则CG GA 的比值是 ( )(A)2 (B)3 (C)4 (D)5 5.在 RtΔABC 中,AD 是斜边上的高,BC=3AC 则ΔABD 与ΔACD 的面积的比值是( )(A) 2 (B)3 (C)4 ( D)8 6.在 RtΔABC 中,∠ACB=90°,CD⊥AB 于 D,则 BD∶AD 等于( ) (A)a∶b (B)a2∶b2 (C) a ∶ b (D)不能确定 7.若梯形上底为 4CM,下底为 6CM,面积为 5CM2,则两腰延长线与上底围成的三角形的面积 是---------- 8.已知直角三角形的斜边的长为 13CM,两条直角边的和为 17CM,则斜边上的高的长度为 ------------- 9..RtΔABC 中,CD 是斜边上的高线,,AB=29。AD=25,则 DC=--------- 10.平行四边形 ABCD 中,E 为 BA 延长线上的一点,CE 交 AD 于 F 点,若 AE∶AB=1∶3 则 SABCF∶ SCDF=--------- 11.如图,在ΔABC 中,D 为 AC 上一点,E 为延长线上一点, 且 BE=AD,ED 和 AB 交于 F 求证:EF∶FD=AC∶BC 12.如图,在ΔABC 中,∠ABC=90°,CD⊥AB 于 D,DE⊥AC 于 E, 求证:CE AE =BC2 AC2 A B C D E A B D E C 解题指导 1.如图,在 RtΔABC 中,∠ADB=90°,CD⊥AB 于 C,AC=20CM,BC=9CM,求 AB 及 BD 的长 2.如图,已知ΔABC 中,AD 为 BC 边中线,E 为 AD 上一点,并且 CE=CD, ∠EAC=∠B,求证:ΔAEC∽ΔBDA,DC2=AD•AE 3.如图,已知 P 为ΔABC 的 BC 边上的一点,PQ∥AC 交 AB 于 Q ,PR∥AB 交 AC 于 R,求证: ΔAQR 面积为ΔBPQ 面积和ΔCPQ 面积的比例中项。 4.如图,已知 PΔABC 中,AD,BF 分别为 BC,AC 边上的高,过 D 作 AB 的垂线交 AB 于 E, 交 BF 于 G,交 AC 延长线于 H,求证:DE2=EG•EH 5.如图,已知正方形 ABCD,E 是 AB 的中点,F 是 AD 上的一点,EG⊥CF 且 AF=1 4 AD,于,(1)求证:CE 平分∠BCF,(2) 1 4 AB2=CG•FG 独立训练 1.用一个 2 倍的放大镜照一个ΔABC,下列命题中正确的是( ) (A)ΔABC 放大后是原来的 2 倍(B)ΔABC 放大后周长是原来的 2 倍; (C)ΔABC 放大后面积是原来的 2 倍 (D)以上的命题都不对 2.边长为 a 的等边三角形被平行于一边的直线分成等积的两部分,则截得的梯形一底的长 为( ) (A)1 2 a (B) 2 a (C) 2 2 a (D)2 3 a 3 如图,PLMN 为矩形,AD⊥BC 于 D,PL∶LM=5∶9, 且 BC=36CM,AD=12CM,则矩形 PLMN 的周长为( ) 4 在 RtΔABC 中,CD 是斜边上的高线,AC∶BC=3∶1 则 SΔABC∶SΔACD 为( )(A)4∶3 (B) 9∶1 (C)10∶1 (D)10∶9 5 如图,RtΔBAC 中,∠BAC=90°,AD⊥BC 于 D, DE⊥AB 于 E,DF⊥AC 于 F,下列中正确的个数是( ) AB2=BD•BC,DE2=AE•BD,AC2=DC•BC,AB3 AC3 =BD CF , AD2=BD•DC,BD2=BE•AB(A)6 (B)5 (C)4 (D)3 6 如图,若 DC∥EF∥AB,且 DE∶EA=m∶n,BC=a, 则 CF=---------,FB=-------------- 6.CD 是 RtΔABC 斜边上的高线, A B C D E M NP L A B CD E F A B CD E F A BC D A B CD E B A CP Q R A B C D E F G H A B C D E F G 7.BC=10,BD=6,则 AD=---------AC=--------- 8 如图,M 为 AB 中点,AB∥CD,延长 NC 交 BD 延长线于 E,延长 MD 交 AC 延长线于 F,求证: EF∥AB 9 如图,在正方形 ABCD 中,M 为 AB 上一点,N 为 BC 上一点,并且 BM=BN,BP⊥MC 于 P 求 证:DP⊥NP 10 如图,在ΔABC 中,BC= a ,P 是 BC 上一点,PE∥AC,PF∥AB,分别交 AB,AC 于 E,F, 求使平行四边形 AEPF 面积最大时点 P 的位置。 第二十八课 锐角三角函数 〖知识点〗 锐角三角函数、锐角三角函数值的符号、锐角三角函数值的变化规律、特殊角三角函 数值、互为余角的三角函数间的关系、同角三角函数间的关系(平方关系、商数关系、倒数 关系) 〖大纲要求〗 1. 理解正弦、余弦、正切、余切的概念,并能运用; 2. 掌握正弦和余弦表、正切和余切表的查法,掌握特殊角三角函数值,并能运用特殊 角的三角函数值进行计算和化简; 3. 掌握互为余角和同角三角函数间关系,并能运用它们进行计算或化简。 〖考查重点与常见题型〗 1. 求三角函数值,常以填空题或选择题形式出现,如: 在 Rt△ABC 中,∠C=90°,3a= 3 b,则∠A= ,sinA= 2. 考查互余或同角三角函数间关系,常以填空题或选择题形式出现,如: (1) sin53°cos37°+cos53° sin37°= (2) 在 Rt△ABC 中,∠C=90°,下列各式中正确的是( ) (A)sinA=sinB (B)sinA=cosB (C)tanA=tanB (D)c0tA=cotB 3. 求特殊角三角函数值的混合运算,常以中档解答题或填空题出现,如: 1-2sin30°cos30°= 〖预习练习〗 1.Rt△ABC 中,若 sinA=4 5 ,AB=10,那么 BC= ,tanB= 2.若 tanα·tan16°=1,且α为锐角,则α= 3.写出适合条件的锐角α cosα= 3 2 ,α= , 3 tan2α-4tanα+ 3 =0,则α= 4. 查表求 cot68°19ˊ时,先查得 cot68°18ˊ=0.3979,又查得 1ˊ的修正值是 0.0003,则 cot68°19ˊ= A B C E F P DA M NB C P BA C D E F M 5. 设α、β互为余角,则 tanα·tanβ-cotα+β 2 = 6. 直角三角形中,∠C=90°,a,b 分别是 A,B 的对边,则a b 是角 A 的( ) (A)正弦 (B)余弦 (C)正切 (D)余切 7. △ABC 中,∠C=90°,则 cosA·cotB 的值是( ) (A)a c (B)c a (C)a b (D)b a 考点训练 1.Rt△ABC 中,∠C=90°,AB=6,AC=2,则 sinA=( ) (A) 1 3 (B)2 3 (C)2 3 2 (D) 2 3 2.在△ABC 中,∠C=90°,sinA=3 5 ,则 tanA·cosA 的值是( ) (A) 3 5 (B)4 5 (C) 9 25 (D)16 25 3.已知∠A+∠B=90°,则下列各式中正确的是( ) (A)sinA=sinB (B)cosA=cosB (C)tanA=cogB (D)tanA=tanB 4.若 0°cosa (B)cosa>sina (C)cota<1 (D)tana>cota 5.Rt△ABC 中,∠C=90°,AC∶BC=1∶ 3 ,则 cosA= ,cotA= 6.设 a 为锐角,若 sina= 3 2 ,则 a= ,若 tana= 3 3 ,则 a= 7.查表得 cot56°42ˊ=1.5224,2ˊ的修正值为 0.0019,则 cot56°44ˊ= 8.已知 a 为锐角,若 cosa=1 2 ,则 sina= ,tan(90°-a)= 9. 已知 sina=12 13 , a 为锐角,则 cosa= ,tana= ,cota= 10.用“>”或“<”连结: cos18° cos18°3ˊ; tan31° tan32°; tan29°30ˊ cot60°29ˊ sin39° cos51°;cot30° sin89°;sina+cosa 1(a 为锐角) 11.计算:(1)1 2 sin60°+ 2 2 cos45°+sin30°·cos30° (2)3 tan30°- 1-2 tan60°+tan260° +cos0°·cos45° 12.△ABC 中,∠BAC=90°,AD 是高,BD=9,tanB=4 3 ,求 AD、AC、BC 13.已知方程 x2-5x·sina+1=0 的一个根为 2+ 3 ,且 a 为锐角,求 tana 的值。 解题指导 1. 计算:(1)sin45°·cos45°+sin60°·cot45° cos0°·tan60° +3cot260°+cos90° cos30° (2)-tan260°+2 cos45°-cot230°+tan10°·tan80° sin223°+sin267° 2. 若 a 为锐角,tga=3,求cosa-sina cosa+sina 的值。 3. 在 Rt△ABC 中,∠C=90°,求证:a3cosA+b3cosB=abc 4. 方程 x2- 2 x +m=0 的两根是一个直角三角形中两锐角的余弦 cosA 和 cosB,求 A、B 的度数和 m 的值。 5. 若方程 2x2-2x·cosa+1 2 cosa(cosa+4)=0 的两个根 x1、x2 满足(x1-1)(x2-1) =109 100 ,求 sina 的值。 6.△ABC 中,AB=AC,∠BAC=36°,AD 是 BC 边上的高,BE 是∠ABC 的平分线,BC=1,试 利用这个三角形求出 sin18°的值。 7.已知 sinθ和 cosθ是方程 a2x2+a3x+1=0 的两根,求 a 的值。 独立练习 1.在 Rt△ABC 中,∠C=90°,sinA∶sinB=3∶4,则 ctgA 的值( ) (A) 3 4 (B)4 3 (C)3 5 (D)4 5 2.若 2cosa- 3 =0,则锐角 a=( ) (A) 30°(B)15° (C)45°(D)60° 3. 已知 a=sin25°,b=tan46°,c=cot17°,m=cos20°,则 a、b、c、m 的大小关系( ) (A) a∠B,点 P 在斜边 AB 上移 动,连结 PC,(1)求∠A 的度数(2)设 AP 为 x,CP2 为 y,求 y 关于 x 的函数表达式及自变量 x 的 取值范围,(3)求证:AP=1 时,CP⊥AB 四.解题指导(2) 1.(1)已知锥体轴截面(如图),斜角α,tanα=1 8 ,求锥度 K= (2)一锥形零件锥度为1 8 ,小头直径为 20mm,长为 64mm,求这个零件侧面积; (3)如图,渠道横截面为等腰梯形,内坡比为 2:1,测得距深为 2m,上口宽为 3.5m,求 渠道底宽。 2.如图,某海埂的横断面是梯形,坎上底 AD 为 4 米,近水面(斜坡 AB)的坡度 i=1: 3 , 斜坡 AB 的长度为 12 米,背水面(斜坡 CD)的坡度为 i=1:1,求(1)斜坡 AB 的坡角(2) 坎底宽 BC 和斜坡 CD 的长。 3.要测得底部不能到达的烟囱的高 AB,从与烟囱底部在同一水平线的 C、D 两处测得烟囱 的仰角为α、β,CD 间的距离是 a 米,已知测角仪的高 b 米,求烟囱的高 AB 4.某海轮以每小时 30 海里的速度航行,在 A 处测得海面上油井 P 在南偏东 60°,一直向北 航行 40 分钟后到达 B 点,测得油井 P 在南偏东 30°。海轮改为北偏东 60°的航向再航行 80 分钟到达 C 点(1)画出海轮航行的示意图(2)试求 P、C 间的距离(结果可保留根号) 5..如图,A 城气象台测得台风中心从 A 城正西方向 300 千米 B 处以每小时 10 7 千米的速 度向北偏东 60°的 BF 方向移动,距台风中心 200 千米的范围内为受台风影响的区域(1) 问 A 城是否会受这次台风的影响?并说明理由(2)若 A 城受到这次台风的影响,那么 A 城遭受这次影响的时间有多少长? 独立练习: 1.在 Rt△ABC 中,∠C=90°,cosB=2 3 ,则 a:b:c=( ) (A) 2: 5 :3 (B) 1:2:3 (C) 1: 2 :3 (D) 2: 5 : 3 2. 在 Rt△ABC 中,∠C=90°,斜边中线是 3cm,sinA=1 3 ,则 S△ABC=( ) (A) 2 cm2 (B) 2 2 cm ( C ) 3 2 cm2 ( D) 4 2 cm2 2. 在 Rt△ABC 中,∠C=90°,AB=50 2 ,AC=50,则 BC= ,∠B= ,S△ABC= 4. 在 Rt△ABC 中,两条直角边之比为 2:3,斜边长为 3 13 ,则最小角的余弦值是 5.已知,如图△ABC 中,∠ C=90°,AD 平分∠BAC,CD= 3 ,BD=2 3 ,求平分线 AD 的长, AB,AC 的长,外接圆的面积,内切圆的面积。 6.已知△ABC 中,AD⊥BC 于 D,点 E 在 AC 上,且∠B=∠DEC,CE CB =1 2 (1)求∠C 的度数(2) 若 CD=2,S△ABC=6 3 ,求 AB 的长 7.一船从西向东航行,航行到灯塔 C 处,测得海岛 B 在北偏东 60°方向,该船继续向东航 行到达灯塔 D 处时,测得海岛 B 在北偏东 45°方向,若灯塔 C、D 间的距离是 10 海里,海 岛 B 周围 12 海里有暗礁,问该船继续航行(沿原方向)有无触礁的危险? 8.如图,二次函数 y=x2+bx+c 的图像与 x 轴相交于 A,B,点 A 在原点左边,点 B 在原点右边, 点 P(1,m)(m>0)在抛物线上,AB=2, tan∠PAB=2 5 ,(1)求 m 的值;(2)求二次函数解析式 第 30 课 圆的有关性质 〖知识点〗圆、圆的对称性、点和圆的位置关系、不在同一直线上的三点确定一个圆、三角 形的外接圆、垂径定理逆定理、圆心角、弧、弦、弦心距之间的关系、圆周角定理、圆内接 四边形的性质 〖大纲要求〗 1. 正确理解和应用圆的点集定义,掌握点和圆的位置关系; 2. 熟练地掌握确定一个圆的条件,即圆心、半径;直径;不在同一直线上三点。一个 圆的圆心只确定圆的位置,而半径也只能确定圆的大小,两个条件确定一条直线,三个条件 确定一个圆,过三角形的三个顶点的圆存在并且唯一; 3. 熟练地掌握和灵活应用圆的有关性质:同(等)圆中半径相等、直径相等直径是半 径的 2 倍;直径是最大的弦;圆是轴对称图形,经过圆心的任一条直线都是对称轴;圆是中 心对称图形,圆心是对称中心;圆具有旋转不变性;垂径定理及其推论;圆心角、圆周角、 弧、弦、弦心距之间的关系; 4. 掌握和圆有关的角:圆心角、圆周角的定义及其度量;圆心角等于同(等)弧上的 圆周角的 2 倍;同(等)弧上的圆周角相等;直径(半圆)上的圆周角是直角;90°的圆周 角所对的弦是直径; 5. 掌握圆内接四边形的性质定理:它沟通了圆内外图形的关系,并能应用它解决有关 问题; 6. 注意:(1)垂径定理及其推论是指:一条弦①在“过圆心”②“垂直于另一条弦” ③“平分这另一条弦”④“平分这另一条弦所对的劣弧”⑤“ 平分这另一条弦所对的优弧” 的五个条件中任意具有两个条件,则必具有另外三个结论(当①③为条件时要对另一条弦增 加它不是直径的限制),条理性的记忆,不但简化了对它实际代表的 10 条定理的记忆且便于 解题时的灵活应用,垂径定理提供了证明线段相等、角相等、垂直关系等的重要依据;(2) 有弦可作弦心距组成垂径定理图形;见到直径要想到它所对的圆周角是直角,想垂径定理; 想到过它的端点若有切线,则与它垂直,反之,若有垂线则是切线,想到它被圆心所平分; (3)见到四个点在圆上想到有 4 组相等的同弧所对的圆周角,要想到应用圆内接四边形的 性质。 〖考查重点与常见题型〗 1. 判断基本概念、基本定理等的正误,在中考题中常以选择题、填空题的形式考查学 生对基本概念和基本定理的正确理解,如:下列语句中,正确的有( ) (A)相等的圆心角所对的弧相等 (B)平分弦的直径垂直于弦 (C)长度相等的两条弧是等弧 (D)弦过圆心的每一条直线都是圆的对称轴 2. 论证线段相等、三角形相似、角相等、弧相等及线段的倍分等。此种结论的证明重 点考查了全等三角形和相似三角形判定,垂径定理及其推论、圆周角、圆心角的性质及切线 的性质,弦切角等有关圆的基础知识,常以解答题形式出现。 考点训练: 1.在⊿ABC 中,∠C=90°,AB=3cm,BC=2cm,以点 A 为圆心,以 2.5cm 为半径作圆,则点 C 和⊙A 的位置关系是( ) (A)C 在⊙A 上 (B)C 在⊙A 外 (C)C 在⊙A 内 (D)C 在⊙A 位置不能确定。 2.一个点到圆的最大距离为 11cm,最小距离为 5cm,则圆的半径为( ) (A)16cm 或 6cm, (B)3cm 或 8cm (C)3cm (D)8cm 3.如图,弦 AC,BD 相交于 E,且 AB,BC,CD 的弧长相等, ∠AED=30°,则∠AED 的度数是( ) (A)150° (B) 105° (C) 120° (D) 140° 4.在⊿ABC 中,∠C=90°,O 是 BC 上的一点,以 OB 为半径作 ⊙O 交于 AB 于 D,交 BC 于 E,∠A=30°BD=6,则⊙O 的直径是( ) (A)12 (B) 9 (C) 6 (D)3 5.AB 是⊙O 直径,AB=4,F 是 OB 中点,弦 CD⊥AB 于 F,则 CD=_________ 6.⊿ABC 内接于⊙O,OD⊥BC,∠BOD=36°,则∠A=____ 7.圆内接⊿ABC 中,AB=AC,圆心到 BC 的距离为 3cm,圆的半径为 7cm,则腰长 AB=__ _ 8.四边形 ABCD 内接于圆,AB,BC,CD,DA 的弧长之比为 5:8:3:2 则∠ABC=____ _ 9.如图,⊙O 中两条不平行弦 AB 和 CD 的中点 M,N.且 AB=CD, 求证:∠AMN=∠CNM 10.如图,四边形 ABCD 内接于⊙O,∠ADC=90°,B 是弧 AC 的中点,AD=20,CD=15,求 BD 的长。 解题指导。 1.如图,⊙O1 的圆心在⊙O 的圆周上,⊙O 和⊙O1 交于 A,B,AC 切⊙O1 于 A,连结 CB,BD 是⊙O 的直径,∠D=40°求:∠A O1B、∠ACB 和∠CAD 的度数。 2.如图,AB 是⊙O 直径,ED⊥AB 于 D,交⊙O 于 G,EA 交⊙O 于 C,CB 交 ED 于 F,求证: DG2=DE•DF 3.如图,⊙O 是⊿ABC 外接圆,AD⊥BC 于 D,交⊙O 于 N,AE 平分∠BAC 交⊙O 于 E,求证: AE 平分∠OAD 4.已知,如图 O 为圆心,∠AOB=120°,弓形高 ND=2cm,矩形 EFGH 的两顶点 E,F 在弦 AB 上,H,G 在弦 AB 上,且 EF=4HE,求 HE 的长。 独立训练: 1.三角形的外心一定在该三角形上的三角形是( ) (A) 锐角三角形 (B)钝角三角形 (C)直角三角形 (D)等腰三角形 2.边长为 2 的等边三角形的外接圆的半径是( ) (A) 3 3 (B) 3 (C)2 3 (D)2 3 3 3,圆内接四边形 ABCD 中,四个角的度数比可顺次为( ) (A)4:3:2:1 (B)4:3:1:2 (C)4:2:3:1(D)4:1: 3:2 4.AB 是⊙O 的弦,∠AOB=80°则弦 AB 所对的圆周角是( ) (A)40° (B) 140°或 40° (C) 20° (D)20°或 160° 5.AB 是⊙O 的弦,C 为⊙O 上的一点,弧 AC,CB 的长比是 1:2,弦 BC=12cm,则⊙O 半径 为______cm 6.⊙O 直径为 8,弦 AB=4 2 ,则∠AOB=_____。 7.圆的半径为 2cm,圆内一条弦长为 2 3 cm,则弦的中点与弦所对弧的中点间的距离为_ _____,这条的弦心距为_______ 8.已知⊙O 中,半径 OD⊥直径 AB,F 是 OD 中点,弦 BC 过 F 点, 若⊙O 半径为 R 则弦 BC 长_____ 9.如图,⊿ABC 内接于⊙O,且 BC 是⊙O 的直径,AD⊥BC 于 D,F 是弧 BC 中点,且 AF 交 BC 于 E,AB=6,AC=8,求 CD,DE,及 EF 的长。 10.如图,弦 EF⊥直径 MN 于 H,弦 MC 延长线交 EF 的反向延长线于 A,求证:MA•MC=MB• MD 第 31 课 直线和圆的位置关系 知识点: 直线和圆的位置关系、切线的判定和性质、三角形的内切圆、切线长定理、弦切角的定 理、相交弦、切割线定理 大纲要求: 1.掌握直线和圆的位置关系的性质和判定; 2.掌握判定直线和圆相切的三种方法并能应用它们解决有关问题:(1)直线和圆有唯一公 共点;(2)d=R;(3)切线的判定定理 (应用判定定理是满足一是过半径外端,二是与这半径 垂直的二个条件才可判定是圆的切线) 3.掌握圆的切线性质并能综合运用切线判定定理和性质定理解决有关问题:(1)切线与圆 D CB A O E F D C B A O M N E H 只有一个公共点;(2)圆心到切线距离等于半径;(3)圆的切线垂直于过切点的半径;(4) 经 过圆心且垂直于切线的直线必过切点;(5)经过切点且垂直于切线的直线必过圆心;(6)切线 长定理;(7) 弦切角定理及其推论。 4,掌握三角形外切圆及圆外切四边形的性质及应用; 5.注意:(1)当已知圆的切线时,切点的位置一般是确定的,在写条件时应说明直线和圆相 切于哪一点,辅助线是作出过确定的半径;当证明直线是圆的切线时,如果已知直线过圆上 某一点则可作出这一点的半径证明直线垂直于该半径;即为“连半径证垂直得切线”;若已 知条件中未明确给出直线和圆有公共点时,则应过圆心作直线的垂线,证明圆心到直线的距 离等于半径,即为:“作垂直证半径得切线”。(2) 见到切线要想到它垂直于过切点的半径; 若过切点有垂线则必过圆心;过切点有弦,则想到弦切角定理,想到圆心角、圆周角性质, 可再联想同圆或等圆弧弦弦心距等的性质应用。(3)任意三角形有且只有一个内切圆,圆心 为这个三角形内角平分线的交点。 考查重点与常用题型: 1.判断基求概念,基本定理等的证误。在中考题中常以选择填空的形式考查形式对基本 概念基求定理的正确理解,如:已知命题:(1)三点确定一个圆;(2)垂直于半径的直线是圆 的切线;(3)对角线垂直且相等的四边形是正万形;(4)正多边形都是中心对称图形;(5)对 角线相等的梯形是等腰梯形,其中错误的命题有 ( ) (A)2 个 (B)3 个 (C)4 个 (D)5 个 2.证明直线是圆的切线。证明直线是圆的切线在各省市中考题中多见,重点考查切线的 判断定理及其它圆的一些知识。证明直线是圆的切线可通过两种途径证明。 3.论证线段相等、三角形相似、角相等、弧相等及线段的倍分等。此种结论的证明重点 考查了金等三角形和相似三角形判定,垂径定理及其推论、圆周角、圆心角的性质及切线的 性质,弦切角等有关圆的基础知识。 考点训练: 1.如图⊙O 切 AC 于 B,AB=OB=3,BC= 3 ,则∠AOC 的度数为( ) (A)90 ° (B)105° (C)75° (D)60° 2.O 是⊿ABC 的内心,∠BOC 为 130°,则∠A 的度数为( ) (A)130° (B)60° (C)70° (D)80° 3.下列图形中一定有内切圆的四边形是( ) (A)梯形 (B)菱形 (C)矩形 (D)平行四边形 4.PA、PB 分别切⊙O 于 A、B,∠APB=60°,PA=10,则⊙O 半径长为( ) (A)10 3 3 (B)5 (C)10 3 (D)5 3 5.圆外切等腰梯形的腰长为 a,则梯形的中位线长为 6.如图⊿ABC 中,∠C=90°,⊙O 分别切 AB、BC、AC 于 D、E、F,AD=5cm,BD=3cm,则⊿ ABC 的面积为 7.如图,MF 切⊙O 于 D,弦 AB∥CD,弦 AD∥BF,BF 交⊙O 于 E,  40 m CD  ,  80 m AB  ,则∠ ADM = °,∠AGB= °,∠BAE= °。 8.PA、PB 分别切⊙O 于 A、B,AB=12,PA=3 13 ,则四边形 OAPB 的面积为 9.如图,AB 是⊙O 直径,EF 切⊙O 于 C,AD⊥EF 于 D,求证:AC2=AD·AB。 10.如图,AB 是⊙O 的弦,AB=12,PA 切⊙O 于 A,PO⊥AB 于 C,PO=13,求 PA 的长。 解题指导: 1.如图⊿ABC 中∠A=90°,以 AB 为直径的⊙O 交 BC 于 D,E 为 AC 边中点,求证:DE 是⊙ O 的切线。 2.如图,AB 是⊙O 直径,DE 切⊙O 于 C,AD⊥DE,BE⊥DE,求证:以 C 为圆心,CD 为半径 的圆 C 和 AB 相切。 3.如图,梯形 ABCD 中,AD∥BC,AB=CD,⊙O 分另与 AB、BC、CD、AD 相切于 E、F、G、H, 求证:⊙O 直径是 AD,BC 的比例中项。 4.已知:AB 是⊙O 的直径,AC 和 BD 都是⊙O 切线,CD 切⊙O 于 E,EF⊥AB,分别交 AB, AD 于 E、G,求证:EG=FG。 独立训练: 1.已知点 M 到直线 L 的距离是 3cm,若⊙M 与 L 相切。则⊙M 的直径是 ;若 ⊙M 的半径是 3.5cm,则⊙M 与 L 的位置关系是 ;若⊙M 的直径是 5cm,则⊙ M 与 L 的位置是 。 2.RtΔABC 中,∠C=90°,AC=6,BC=8,则斜边上的高线等于 ;若以 C 为圆心 作与 AB 相切的圆,则该圆的半径为 r= ;若以 C 为圆心,以 5 为半径作圆, 则该圆与 AB 的位置关系是 。 3.设⊙O 的半径为 r,点⊙O 到直线 L 的距离是 d,若⊙O 与 L 至少有一个公共点,则 r 与 d 之间关系是 。 4.已知⊙O 的直径是 15 cm,若直线 L 与圆心的距离分别是①15 cm;②③7.5 cm;③5 cm 那么直线与圆的位置关系分别是 ; ; 。 5.已知:等腰梯形 ABCD 外切于为⊙O,AD∥BC,若 AD=4,BC=6,AB=5,则⊙O 的半径 的长为 。 6.已知:PA、PB 切⊙O 于 A、B,C 是弧 AB 上一点,过点 C 的切线 DE 交 PA 于 D,交 PB 于 E,ΔPDE 周长为 。 7.已知:PB 是⊙O 的切线,B 为切点,OP 交⊙O 于点 A,BC⊥OP,垂足为 C ,OA=6 cm, OP=8 cm,则 AC 的长为 cm。 8.已知:ΔABC 内接于⊙O,P、B、C 在一直线上,且 PA2=PB•PC,求证:PA 是⊙O 的切线。 9.已知:PC 切⊙O 于 C,割线 PAB 过圆心 O,且∠P =40°,求∠ ACP 度数。 10 已知:过⊙O 一点 P,作⊙O 切线 PC,切点 C,PO 交⊙O 于 B,PO 延长线交⊙O 于 A,CD ⊥AB,垂足为 D,求证:(1)∠DCB=∠PCB (2)CD:BD=PA:CP 第 32 课 与圆有关的比例线段 〖知识点〗 相交弦定理、切割线定理及其推论 〖大纲要求〗 1. 正误相交弦定理、切割线定理及其推论; 2. 了解圆幂定理的内在联系; 3. 熟练地应用定理解决有关问题; 4. 注意(1)相交弦定理、切割线定理及其推论统称为圆幂定理,圆幂定理是圆和相 似 三角形结合的产物。这几个定理可统一记忆成一个定理:过圆内或圆外一点作圆的两条割线, 则这两条割线被圆截出的两弦被定点分(内分或外分)成两线段长的积相等(至于切线可看 作是两条交点重合的割线)。使用时注意每条线段的两个端点一个是公共点,另一个是与圆 的交点; (2)见圆中有两条相交想到相交弦定理;见到切线与一条割线相交则想到切割线定理; 若有两条切线相交则想到切线长定理,并熟悉此时图形中存在着一个以交点和圆心连线为对 称轴的对称图形。 〖考查重点与常见题型〗 证明等积式、等比式及混合等式等。此种结论的证明重点考查了相似三角形,切割线 定 理及其推论,相交弦定理及圆的一些知识。常见题型以中档解答题为主,也有一些出现在选 择题或填空题中。 〖预习练习〗 1.圆内两弦相交,其中一条弦长为 8cm,且被交点平分,另一条被交点分为 1:4 两部分,则 这条弦长为( ) (A)2cm (B)8cm (C)10cm (D)16cm 2.自圆外一点所作过圆心的割线长是 12cm,圆的半径为 4cm,则过此点所引的切线长为 ( ) (A) 16cm (B)4 3 cm (C)4 2 cm (D)以上答案都不对 3.如图,圆内接四边形 ABCD 的 BA、CD 的延长线交于 P,AC、BD 交 于 E,则图中相似三角形有( ) (A)2 对 (B)3 对 (C)4 对 (D)5 对 4.圆内两条弦 AB 与 CD 相交于 E,如果 AE=BE,CE=9,DE=4,那么 AB= 5.从圆外一点 P 向圆引两条割线 PAB、PCD,分别与圆相交于 A、B、C、D,如果 PA=4,PC =3,CD=5,那么 AB= 6.Rt△ABC 中两条直角边分别为 6cm,8cm,则外接圆半径为 ,内切圆半径为 7.PA、PB 分别是⊙O 的切线,切点分别为 A、B,∠AOB=144°,则∠P= 考点训练: 1.⊙O 中直径 CD⊥弦 AB 于 E,AB=6,DE∶CE=1∶3,则 DE 的长为( ) (A) 3 (B) 3 (C) 2 3 (D) 6 2.由圆外一点作圆的切线长为 6,过这点作过圆心的割线长为 12,则此圆半径长为 ( ) (A) 19cm (B) 6cm (C) 4.5cm (D)以上答案都不对 3. 如图 1,⊙O 的半径为 6,PQ=6,AR=8 则 QR 的长为( ) (A) 9 (B) 10 (C) 11 (D) 12 4. 如图 2,CD 为⊙O 直径,弦 AB 垂直 CD 于 P,AP=4,PD=2,则 PO=___. 5. 如图 3,PAB 为⊙O 的割线,PC 切⊙O 于 C,PC=10,AB=15,则 PA 长为___________. 6.如图 4,弦 AB⊥弦 CD 于 E,若 AE=2,BE=6,DE=3,则⊙O 的直径长=________. 7.如图,PAB 为⊙O 的割线,PO 交⊙O 于 C,OP=13,PA=9,AB=7,求⊙O 直径的 长. 8.如图,P 是⊙O 外一点,PA 切⊙O 于 A,PBC 为⊙O 的割线,求证:AB2 AC2 =PB PC R Q A O P 1 A B OC DP 2 P A B O C 3 B A C D E O 4 r A B CO P A P B C O 9.如图,在两圆公共弦 AB 上,任取一点 G,过 G 作直线交一圆于 C,D,交另一圆于 E,F. 求证:CG·ED=EG·CF. 解题指导 1. 如图,ABCD 是⊙O 的内接四边形,DP∥AC,交 BA 的延长线于 P,求证:AD·DC=PA·BC. 2.如图,锐角△ABC,以 BC 为直径作圆,在 AB 上截取 AE=切线长 AD,过 E 作 AB 的垂 线交 AC 延长线于 F,求证:AE AB = AC AF . 3. 如图,若△ABC 的∠A 平分线交 BC 于 D,交其外接圆于 E,求证:AD2=AB·AC-BD·CD. 4.如图,△ABC 内接于⊙O,CP 切⊙O 于 C,交 AB 延长线于 P,割线 PD 交 AC 于 F,CB 于 E,且 CE=CF, 求证:(1)PD 是∠APC 的平分线,(2)CF2=AF·BE. A B C D E F O1 OG P A B C D O A B C D F E O A B C E D O P B A C EFD O 独立训练: 1.AB 是⊙O 直径,C 是 AB 延长线上一点,CD 切⊙O 于 D,AB=6,CD=4,则 CB 的长为 ( ) (A) 2 (B) 8 3 (C) 2 3 (D) 3 2.如图 1,P 在半圆 O 的直径 AB 延长线上,且 PB=OB=2, PC 切⊙O 于 C,CD⊥AB 于 D,则 CD 的长为( ) (A) 2 3 (B) 3 (C) 3 2 (D) 4 3 3.如图 2,△ABC 中∠A=90°,AC=3,AB=4,半圆圆心在 BC 上,与 AB,AC 切于 D,E,则 ⊙O 半径为( ) (A) 12 7 (B) 7 12 (C) 7 2 (D) 2 3 4.⊙O 中直径 CD 垂直弦 AB 于 E,AB=8,DE∶CE=3∶1, 则 DE 的长为( ) (A)2 (B)4 (C)2 3 (D)4 3 5.如图 3,AB 为⊙O 直径,弦 CD⊥AB 于 P,若 CD=a,AP=b, 则半径 R=____. 6.如图 4,AB 为⊙O 直径,CD 切⊙O 于 B,且 BC=BD,AD 交⊙O 于 E,AB=8,CD=12,则 S△CDE=___________. 7.如图 5,BE 为半圆 O 直径,AD 切⊙O 于 B,BC 切 ⊙O 于 B,BE=BC=6,则 AD 长为___________. 8.如图 6,以直角坐标系的原点 O 为圆心作圆,A 是 x 轴上一点, AB 切⊙O 于 B,若 AB=12,AD=8,则点 B 坐标为____________. 9.如图,AB 是⊙O 直径,BC 是弦,CD 切⊙O 于 C,AD⊥CD 交 BC 延长线于 E,AE=8cm, 求 AB 的长。 10.已知:如图,AD 切⊙O 于点 D,ACB 为⊙O 的割线,AP=AD,BP,CP 分别交⊙O 于 M,N, 求证:(1)△PCA∽△ABP (2)MN∥AP. A C B D PO 3 A DE B C O 4 A BO C D E C PBDA O 图1 C B D A O E 图2 C B D A OE 图5 B D AOE 图6 y x A D P N M C O B 第 33 课 圆与圆的位置关系 知识点: 圆和圆的位置关系、两圆的连心线的性质、两圆的公切线 大纲要求: 1.了解两圆公切线的求法,掌握圆和圆的位置关系; 2.了解两圆位置关系与公共点个数、外公切线条数、内公切线条数以及 d、R、r 之间的关 系; 3.掌握相交两圆的性质和相切两圆的性质; 4.注意 (1)圆与圆的五种位置关系相交和相切是重点;(2)在解题中把两个圆中有关问题 利用圆的性质和直线圆的位置关系的定理和性质转化为一般圆的问题;(3)涉及相交两圆的 问题常可作出公共弦,利用圆周角定理及其推论或连心线垂直乎分公共弦。公共弦可沟通两 个圆的角之间关系,有了连心线,公共弦不仅可取应用相交两圆的性质定理且还能沟通两圆 半径、公切线等之间的关系;(4)涉及相切两圆问题主要可从以下几个方面考虑;①过切点 作两圆的公切线,利用弦切角定理或切线长定理;②作出连心线,利用连心线过切点的性质; ③利用两圆的圆心距等于两圆半径之和或之差;④当两圆外切时,利用连心线、外公切线及 过公切线切点的两条毕径组成的直角梯形,将有关圆的间题转化为直线形间题,把梯形问题 转化为直角三角形问题,通过解直角三角形来解决有关两圆公切线等问题。 考查重点与常甩题型: 1.判断基本概念、基本定理等的正误。在中考题申常以选择题或填空题的 形式考查学生对基本概念和基本定理的正确理解,如:已知两圆的半径分别为 2、5,且圆 心距等于 3,则两圆位置关系是 ( ) (A)外离 (B)外切 (C)相交 (D) 内切 2.考查两圆位置关系中的相交及相切的性质,可以以各种题型形式出现, 多见于选择题 或填空题,有时在证明、计算及综合题申也常有出现。 预习练习: 1.已知两圆的半径分别是 2 和 4,圆心距是 3,那么这两圆的位置是( ) (A)内含 (B)内切 (C)相交 (D) 外切 2.已知半径为 R 和 r 的两个圆相外切。则它的外公切线长为( ) (A)R+r (B) R2+r2 (C) R+r (D) 2 Rr 3.已知⊙O1 半径为 3cm,⊙O2 半径为 4cm,并且⊙O1 与⊙O2 相切,则这两个圆的圆心距为( ) (A)1cm (B)7cm (C) 10cm (D) 1cm 或 7cm 4.两圆半径为 5 和 r,圆心距为 8,当两圆相交时,r 取值范围是 5.两圆直径分别为 6、8,圆心距为 10,则这两圆的最多公切线条数是 考点训练: 1.已知半径为 R 和 r 的两个圆外切,R=2+ 3 ,r=2- 3 ,两圆的一条公切线与连心线 的夹角为α,则角α的度数为( ) (A)30 ° (B)45 ° (C) 60 ° (D) 无法确定 2.如图,两个同心圆,点 A 在大圆上,ABC 为小圆的割线,若 AB·AC=8,则圆环的面积为 ( ) (A)8π (B)12π (C) 4π (D) 16π。 3.如果两圆有两条外公切线,那么两圆的位置关系共有( )种 (A)2 (B)3 (C) 4 (D) 5 4.两圆半径分别为方程 x2-5x+6=0 的两根,圆心距为 5 cm,则它们公切线的条数为( ) (A)4 (B)3 (C) 2 (D) 1 5.两圆半径分别为 12 和 4,外公切线长为 5,则两圆位置关系为 6.两圆内切,其圆心距为 3 cm,一个圆半径为 5 cm,则另一个圆的半径为 7.两圆相交,半径分别为 3 cm 和 4 cm,圆心距为 5 cm, 则两圆的公共弦长为 8.如图,⊙O1 和⊙O2 外切于 P,外公切线与连心线夹角为 30 °, ⊙O1 半径为 3 cm,⊙O2 半径为 1 cm,则 AC 的长为 。 9.如图⊙O 和⊙OA 交于 M、N,且 A 在⊙O 上,弦 MC 交⊙O 于点 D,连结 AD,NC,求证:DA ⊥NC 10.⊙O 和⊙O1 外切于 C,AB 是外公切线, 延长⊙O 交 AB 的延长线于 P 点,若∠P=300, AB=2,求两圆的半径。 解题指导: 1.如果两个圆和一条直线相切于同一点,那么这两个圆相切。 2.如图,ΔABC 的∠C=Rt∠,BC=4,AC=3,两个外切的等圆⊙O1,⊙O2 各与 AB,AC, BC 相切于 F,H,E,G,求两圆的半径。 3.如图,⊙O1 和⊙O2 相切于点 P,AB 切两圆于 A,B,ΔPAB 的周长为 40,面积为 60,求 P 点到 AB 的距离。 4.如图,⊙O 与⊙O1 外离,AB,CD 是内公切线,OO!是圆心距,⊙O 半径为 4,⊙O1 半径 为 6,OO1=20,求两圆内公切线所夹的锐角及内公切线长。 独立训练 1.两圆外切时圆心距为 10cm,且这两圆半径之比为 2:3,如果内含,那么这两圆的圆心距 为( ) (A) 小于 10cm,(B)小于 2cm(C)小于 5cm(D)小于 3cm 2.如图两个同心圆,大圆的弦 AB 交小圆于 C,D,AB=2CD,AB 的弦心 距等于 CD 的一半,则大圆的半径与小圆的半径之比( ) (A) 3:2(B) 5 :2 (C) 5 : 2 (D)5:4 3.已知两圆的半径为 3 cm,和 1 cm,一条外公切线长为 4 cm,那么这两圆的位置半径为 ( ) (A)内切 (B)相交 (C)外切 (D)外离 4.两圆外切,半径为 4cm 和 9cm,则两圆的一条外公切线的长等于 。 5.已知直角三角形的一条直角边为 6,斜边长为 10,那么这个直角三角形的内切圆与外接 圆的圆心距为 。 6.两圆半径分别为 4 和 2,如果它们有两条外公切线互相垂直,则这两圆的连心线长 为 。 7.两圆内切,圆心距为 3,一个圆的半径为 5,另一个圆的半径为 . 8.如图,两圆外切于 P,直线交两圆于 A,B,C,D,求证:∠APD+∠BPC=180° 9.如图,⊙O 和⊙O1 内切于 E,大圆弦 AD 经过⊙O1 且交⊙O1 于 B,C,AB:BC:CD=2:4:3, 求⊙O1 与⊙O 半径之比。 10. 如图,已知⊙O1 和⊙O2 相交于 A,B,过 A 作直线分别交⊙O1,⊙O2 于 C,D,过 B 作 直线分别交⊙O1,⊙O2 于 E,F,求证:CE∥DF 第 34 课 和圆有关的计算 知识点:正多边形和圆、正多边形的有关计算、等分圆周、圆周长、弧长、圆的面积、扇 形的面积、弓形的面积、面积变换 大纲要求: 1.了解用量角器等分圆周的方法,会用直尺和圆规画圆内接正方形和正多边形; 2. 掌握正多边形的定义和有关概念、判定和性质; 3. 熟练地将正多边形的边长、半径、边心距和中心角有关计算转变为解直角三角形问题 来解诀; 4.熟练地运用圆周长、弧长公式、圆的扇形弓形面积公式进行有关计算; 5.明确图形构成,灵活运用、转化思想,提高解决综合图形面积的计算能力; 6.注意(1)任何一个正多边形都有一个外接圆和一个内切圆,这两个圆是同心圆,反之 也成立;(2) 证多边形是轴对称图形,且正 n 边形有 n 条对称轴;(3)正多边形不一起是中 心对称图形,有奇数条边的正多边形没有对称中心,有偶数条边的正多边形有对称中心就是 它的中心;(4)解诀正多边形问题经常需要作出它的外接圆,可转化成解直角三角形问题。 考查重点与常见题型 求解线段的长及线段的比,角的大小,三角函数的值及阴影部分的面积等。此类问题问 题在近三年的中考题中也是多见,求线段的长及比,角的大小等多数是利用恰当地设未知数、 列方程的思想方法来加以解决。求阴影部分的面积除考查了扇形等图形面积的求法,还重点 考查学生灵活应用知识的能力,求阴影部分的面积多半用两种方法解决:一种是将所求阴影 部分的面积转化为所学过的易求图形的面积的和或差;一种是恰当地引辅助线,将所求阴影 部分的面积转化为所学过的易求图形的面积。 预习练习 1.填写下表: 边数 内角 中心角 半径 边长 边心距 周长 面积 n a an Rn an rn Pn Sn 3 3 4 16 6 2 3 2.扇形的圆心角度数 60°,面积 6π,则扇形的周长为 ; 3.已知扇形的圆心角为 140°,弧长为 20πcm,则扇形的面积为 ; 4.圆的半径为 4cm,弓形弧的度数为 60°,则弓形的面积为 ; 5 . 两 个 同 心 圆 , 小 圆 的 切 线 被 大 圆 截 得 的 部 分 为 6 , 则 两 圆 围 成 的 环 形 面 积 为 。 考点训练: 1.已知扇形的半径为 2 3 ,它的面积等于一个半径为 2 的圆的面积,则扇形的圆心角为 ( ) (A)90° (B)120° (C)60° (D)100° 2.两圆的之比为 1:3,则小圆的外切正三角形与大圆的内接正三角形的面积之比为( ) (A)1:9 (B)1:3 (C)2:3 (D)4:9 3.如图,⊿ABC 中,∠C=90°,BC=4,AC=3,⊙O 内切于⊿ABC ,则阴影部分面积为( ) (A)12-π (B)12-2π (C)14-4π (D)6-π 4.同圆的外切正六边形与内接正六边形的面积之比为 。 5.正三角形边长为 a,高为 h ,圆的半径为 R,内切圆半径为 r,则 h:R:r= . 6.边长为 a 的正六边形对角线的长为 。 7.圆外切正方形半径为 2cm,该圆内接正六边形的面积为 . 8.如图:O 内切于弓形 ADB 的最大的圆,且弧 ADB 的度数 为 120°,则⊙O 的周长:L 弧 AB= 。 9.如图,C、D 是以 AB 为直径的圆周三等分点,⊙O 的半 径为 R,则图中阴影部分面积为 。 10.如图,在矩形 ABCD 中,AB=8 cm,将矩形绕点 A 转 90°, 到达 A‵B‵C‵D‵的位置,则在转过程 中,边 CD 扫过的 (阴影部分)面积 S= 。 11.如图,正方形 ABCD 边长为 2 cm,以 B 圆心作弧 AC,P 是弧 AC 上一点,PE⊥CD 于 E, 弧 PA 的长。 12.如图,扇形 OAB 的中心角∠AOB=90°,以 AB 为直径向形外作半圆弧 ANB,以 O 为圆心, AO 为半径作弧 AMB,求证:弧 AMB 与弧 ANB 所围成的月牙形面积和⊿AOB 的面积相等 解题指导: 1.如图,已知扇形 OACB 中,∠⊙ AOB=120°,弧 AB 长为 L=4,⊙O 和弧 AB、OA、OB 分 别相切于点 C、D、E,求⊙O 的周长。 2.如图,半径为的正三角形 ABC 的中心为 O,过 O 与两个顶点画弧,求这三条弧所围成的 阴影部分的面积。 3.如图,割线 PCD 过圆心 O,且 PD=3PC,PA、PB 切⊙O 于 A、B,∠APB=60°,PA=4, AB 与 PD 相交于 E,求弓形 ACB 的面积。 4.如图,同心圆 O,大圆的面积被小圆所平分,若大圆的弦 AB,CD 分别切小圆于 E、F 点, 当大圆半径为 R 时,且 AB∥CD,求阴影部分面积。 独立训练: 1.在半径为 2cm 的圆内,30°、45°、60°、90°、120°的圆心角所对的弧长分别 为 。 2.弧长为 15cm,它所对的圆心角为 60°,圆的直径为 。 3.边长为 6 的正三角形的外接圆和内切圆的周长分别为 。 4.矩形 ABCD 中,对角线 AC=4,∠ACB=30°,以直线 AB 为轴旋转一周得到圆柱的表面积 是 。 5.如图,矩形 ABCD 中,AD=2AB=2。以 D 为圆心 AD 为半径的 弧交 BC 于 F,交 DC 的延长线于 E,则图中阴影部分面积 为 。 6.如图,矩形 ABCD 中,以 AB 为直径的半圆 O 切 CD 于 E,AB=a, 求夹在 BD,DE 及弧 BE 间阴影部分面积 7.如图,PA、PB 切⊙O 于 A、B,若∠APB=60°,⊙O 半径为 3,求阴影部分面积。 8.如图,AB 是⊙O 直径,CD 切⊙O 于 E,BC⊥CD,AD⊥CD 交⊙O 于 F,∠A=60°,AB=4, 求阴影部分面积。 第 35 讲 轨迹与作图 一.考纲要求 1.了解轨迹概念及五种基本轨迹。 2.能利用轨迹进行简单的作图,计算动点所经过的路程的长。 本节内容的知识点:五种基本轨迹和基本作图。 二.基础回顾 1.到点 O 的距离等于 3cm 的点的轨迹是 。 2.和线段 AB 两个端点距离相等的点的轨迹是 。 3.到已知角的两边距离相等的点的轨迹是 。 4.半径为 2cm,且与已知直线 l 相切的圆的圆心的轨迹是 。 5.和两条已知直线 l1 和 l2 相切的圆的圆心轨迹是 。 三.典型例题 例 1.如图,在直角坐标系平面内,线段 AB 的两端点 A、B 分别在 x 轴、y 轴的正半轴上滑 动,AB=8cm,求线段 AB 中点 M 的轨迹。 例 2.如图,A、B、C 三点表示三个村庄,要建一个电视转播站,使它到三个村庄的距离相 等,求作电视转播站的位置(要求尺规作图,保留作图痕迹,不写作法和证明) 例 3.如图,已知:线段 r 和∠ACB 求作一圆 O,使它与∠ACB 的两边相切,且圆的半径等 于 r。要求用直尺和圆规作图) 例 4.如图,已知线段 a、b、∠α,求作:平行四边形 ABCD,使 BD=a,AC= b,BD、AC 的 夹角为α。(要求用直尺和圆规作图,保留作图痕迹) 例 5.如图,一辆汽车在直线形的公路 AB 上由 A 向 B 行驶,M、N 分别是位于公路 AB 两 侧的村庄。(1)设汽车行驶到公路 AB 上点 P 位置时,距离村庄 M 最近;行驶到点 Q 位置 时,距离村庄 N 最近。请在图中的公路 AB 上分别画出点 P,Q 的位置。(保留作图痕迹)。 (2)当汽车从 A 出发向 B 行驶时,在公路 AB 的哪一段路上距离 M,N 两村庄都越来越近? 在哪一段路上距离村庄 N 越来越近,而离村庄 M 越来越远?(分别用文字表述你的结论, 不必证明)。(3)在公路 AB 上是否存在这样一点 H,使汽车行驶到该点时,与村庄 M,N 的距离相等?如果存在,请在图中的 AB 上画出这一点(保留作图痕迹,不必证明);如果 不存在,请简要说明理由。 四.反馈练习 1.斜边为 AB 的直角三角形 ABC 的顶点 C 的轨迹是 。 2.AB 是半径为 R 的⊙O 中的一条弦,若 AB 沿点 A 旋转 30°角,那么,AB 中点 P 随之运动 所经过路程为( ) A 1 12 πR B 1 2 R C 1 6 πR D 1 3 πR 3.如图,已知△ABC,求作 △ABC 的外接圆. 4.如图,已知∠AOB 和边 OB 上一点 E,求作:一点 P,使 P 到∠AOB 两边的距离相等.且 OP=EP 5.如图,已知:线段 m 和角α.求作:等腰三角形 ABC,使底角∠B=α,腰 AB=m. 五.作业 1)底边为已知线段 BC 的等腰三角形 ABC 的顶点 A 的轨迹是 2)以⊙O 上一点 A 为端点的弦的中点的轨迹是 3)设⊙O1、⊙O、2 的半径都是 r,且 O1 O、2>2r,则与⊙O1、⊙O、2 都外切的圆的圆心的轨迹是 4)如图,扇形 AOB,OA⊥OB,点 P 是弧 AB 上任一点,过 B 作 OP 的垂线,垂足为 Q,则点 Q 的轨迹是 5)已知线段 AO(如图),(1)以定点 O 为圆心,定长 OA 为半径作⊙O;(2)作⊙O 的圆内接 六边形 ABCDEF;(3)作正六边形 ABCDEF 的内切圆。 6)已知△ABC(如图),作△ABC 的内切圆。 7)已知△ABC,BC=a,高线 AD=h(如图),求作正方形,使其面积等于△ABC 面积的 2 倍。 8)用直尺和圆规作一个∠AOB,使∠AOB=30°。 9)已知直线 L 上一点 P 以及直线外一点 Q(如图),求作:经过 点 Q 且与直线 L 相切于点 P 的⊙O。 10)已知:线段 a、b 和∠α(如图),求作: ABCD,使 AB=a, AD=b,∠A=∠α。 11)已知一直角边及与它不相邻的锐角(如图),限用直尺和圆规 作 Rt△。(不写作法,但须保留作图痕迹) 12)如图,两个相同的正方形 ABCD 和 A1B1C1D1 ,A1 与 ABCD 的中心重合,且 A1B1C1D1 绕 A1 转 动,试说出它们重叠部分的面积与正方形面积的比。 13)已知等腰三角形的底角和底边(如图),用直尺和圆规作此三角形(不写作法,但须保 留作图痕迹) 14)如图,在一块矩形的铁皮上有一点 P,现要在这块铁皮上剪去一个等腰直角三角形,把 它加工成零件,请你在已知矩形 ABCD 上求作这个等腰直角三角形,使它的直角顶点为 P, 斜边落在 AD 上。 第 36 讲 空间图形的基本知识 (分两课时) 一.考纲要求 1.了解平面的概念、画法及表示法,平面的基本性质,直线 和平面、平面和平面的垂 直及其应用. 2.会画长方形的直观图;会画立方体、长方体的直观图. 3.了解圆柱、圆锥、圆台的底面、高线、母线、轴截面等概念. 通过画长方体等的直观图,以此为基本模型,来研究直线与平面,平面与平面的垂直与 否,逐步培养学生空间想象能力。圆柱、圆锥、圆台的轴截面及其在生产生活中的实际应用 不可忽视。 二.基础回顾 1.下面说法中,正确的是( ) (A)一点能确定的一个平面 (B)两点能确定的一个平面 (C)任意三点能确定一个平面 (D)任意三点不一定能确定一个平面 2.如图,长方体中,和平面 AD1 垂直的棱是_______,和棱的 BB1 垂直的平面是________. 3.如图,长方体中,过点 A1 和平面 A1C1 垂直的平面有( ) (A)1 个 (B)2 个 (C)3 个 (D)4 个 4.画一个水平放置的边长为 3cm 的正方形的直观图. (要求正确画出图形,画图工具不限) 5.等腰三角形以底边上的高线为轴旋转,其余各边旋转所围成的几何体是( ) (A)一个圆锥 (B)二个圆锥 (C)三个圆锥 (D)四个圆锥 三.典型例题 例 1.要画立方体(即正方体)的直观图,甲、乙两位同学分别画出了以下两个表示 立方体上底面 A1B1C1D1 的直观图,请你选择其中画得正确的一个,将它画成立方体的直观图, 并标上顶点字母.(画图工具不限,不要求写画法) 例 2.在半径为 30m 的圆形广场的中心上空,设置一个照明光源,射向地面的光束呈圆 锥形,它的轴截面顶角为 120°,要使光源照到整个广场,求光源的高度至少要多少 m.(精 确到 0.1m) 例 3.如图,圆锥的底面半径为 R,用一个平行于底面的平面去截这个圆锥,把圆锥分 成一个小圆锥和一个圆台,设小圆锥的底面半径为 r,母线长为 x,圆台的母线长为 l. (1)求证;x l = r R-r (2)若x l =1 3 ,R=8,l=13,求圆台的高线长 h. 例 4.如图,平面 ABC 与平面 BCD 是空间两个相交平面,AB 是⊙O 的直径,C 是⊙O 上 一点,D 是平面 ABC 外的一点,CD⊥AC,试判断平面 ABC 与平面 BCD 是否垂直,并说明理由 例 5.某纸晶加工厂为了制作甲、乙两种无盖的长方体小盒(如图),利用边角废料裁出 正方形和长方形两种硬纸片,长方形的宽与正方形的边长相等(如图),现将 150 张正方形硬 纸片和 300 张长方形硬纸片全都用于制作两种小盒,可以各做多少个? 四.反馈练习 1.画出长、宽、高分别为 4cm,3cm,2cm 的长方体的直观图. 2.巳知圆锥的轴截面周长 32cm,底面积为 36πcm2,求轴截面的面积. 3.在长方体 ABCD--A1B1C1Dl 中,如果 AA1=1,AB=BC=2,求 A1C 的长. 五.作业 1.若圆台的上、下底面面积分别为 16π,36π经过高线的中点画平行于底面的截面, 求这个截面的面积。 2.圆锥的母线长是 3cm,轴截面的顶角是 45°,用于平行于圆锥底面的截面截圆锥, 截面过高线的三等分点,求截面圆的面积. 3.下列各图是由全等的正方形组成的图形,能围成一个立方体的图形是( ) 4..一个正方体的六个面上分别标有 2、3、4、5、6、7 中的一个数字;如图所示,表 示这个正方体的三种不同的放置方法,则这三种放置方法中,三个正方体下底面上所标数字 之和是( ) 5.观察图中的正方体,AC 为上底的对角线,A'C'、B'D',为下底的对角线.AC 与 A'C' 相互______;且 C 与 B'D'相互_________.(填人下面的标即可) (1) 平行;(2)相交但不垂直; (3)垂直但不相交;(4)垂直相交. 第 37 讲 圆柱圆锥圆台侧面积计算 一.考纲要求 会计算圆柱、圆锥、圆台的侧面积和表面积. 二.基础回顾 1.用一张边长为 3лcm 和 4лcm 的矩形卷成一个圆柱,则这个圆柱的母线长是________. 2.若圆柱的母线长为 10cm,侧面积为 60cm2,则圆柱的底面半径为( ). (A)3cm (B)6cm (C)9cm (D)12cm 3.圆锥的母线与底面直径都等于 8cm,则圆锥的侧面积是_______. 4.已知圆锥底面半径为 r,若它的侧面积是底面积的 1,5 倍,则母线长_______.,展开后 扇形的圆心角=_______. 5.巳知圆台的轴截面梯形的腰与下底的夹角为 60°,高线长为 4 3 ,中位线长为 5,则圆 台的侧面积是_______ 三.典型例题 例 1.若矩形 ABCD 的邻边不等,分别以直线 AB、BC 为轴旋转一周得两个圆柱,观察 这两个圆柱的底面和侧面,则有 ( ). (A)S 底 S 侧都相等. (B)S 底不等,S 侧相等. (C) S 底相等,S 侧不等.(n) S 底 S 侧都不等. 例 2.如果圆台的上底面半径为 5,下底面半径为 R,中截面把圆台分为上、下两个圆 台,它们的侧面积的比为 1:2,那么 R=( ) (A)10 (B)15 (C)20 (D)25 例 3.用一块圆心角为 150°,面积为 240лcm2 的扇形硬纸片围成一个圆锥模型(相交 粘贴部分忽略不计),求圆锥模型的底面半径. 例 4.巳知圆锥的轴截面周长为 10cm,设腰长为 x,圆锥的表面积为 S, (1) 求 S 关于 X 的函数表达式和自变量 X 的取值范围; (2)画出这个函数图象,确定 S 的取值范围. 例 5.如图,已知直角梯形 ABCD,BC∥AD,∠B=90°,AB=5 cm ,BC=16cm,AD=4cm。 (1)求以 AB 所在直线为轴旋转一周所得几何体的表面积; (2)求以直线 BC 为轴旋转一周所得几何体的表面积. 四.反馈练习 1.用一张边长为 20cm 的正方形纸片围成一个圆柱的侧面,则这个圆柱的底面直径是( ). (A) 20 л cm (B) 10 л cm (C)2 5 л cm (D) л 20 cm 2.一个圆锥的侧面积是底面积的 2 倍,则这个圆锥的侧面展开扇形的圆心角为_______. 3.圆台的侧面展开图扇环圆心角为 180,则圆台下底半径与上底半径之差与母线的比为 ( ). (A)1 2 (B) 1 3 (c)1 4 (D)不能确定 4.以 AB 为斜边的直角三角形 ABC 中,AC=5,BC=12,分别以 AC、CB、BA 所在直线为轴 旋转而得几何体的表面积分别记作 SAC、SBC、SAB,则下列不等式成立的是( ) (A) SAB> SBC> SAC (B) SBC > SAC> SAB (C) SAC> SBC > SAB (D) SAB >SAC> SBC 5.如图,矩形的边 AB=5cm,AD=8cm,分别以直线 AB、AD 为轴旋转一周得两个不同的 圆柱,问哪个圆柱的表面积大? 6.一车间要用铁皮加工一批元件.元件由两部分组成,一个圆柱形的铁管,上面有一个 圆锥形帽子,尺寸如图所示(单位:rnm),问总共需要多少千方厘米的铁皮(精确到个位). 五.巩固提高 1.圆柱的底面半径为 2crn,高为 3crn,则它的侧面积是 crn2 2.巳知圆柱的母线长是 5cm,侧面展开图的面积为 20лcm2,则这个圆柱的底面半径 为 cm. 3.底面半径为 3cm,母线长为 5cm 的圆锥侧面展开图面积为 cm2 4.巳知圆锥的底面直径为 80crn,母线长为 90crn,则它的侧面展开图的圆心角 是 . 5.若一圆锥形烟囱帽的侧面积是 2000лcm2,母线长为 50cm,则这个烟囱帽的底面直 径为( ). (A)80cm (B)lOOcrn(C)40crn (D)60crn 6.圆柱铁桶的侧面展开图是边长为 12лcm 的正方形,则该铁桶的底面直径是( ). (A) 12лcrn(B)6лcrn (C)12cm (D)6cm 7.两个圆锥的母线长相等.侧面积之比为 1:2,底面积之比为 ( ) (A)2:1 (B)1:2 (C)1:3 (D)1:4 8.将一块半径为 Rcm,圆心角为θ°的扇形铁皮做成一个圆锥形的烟囱帽,则这个圆锥 的底面半径是 cm. 9.巳知圆锥的高线和底面直径相等,求底面积和侧面积之比. 10.巳知圆台形铅桶口直径为 28cm,桶底直径 20cm,高线长 36cm;若做这样无盖铅桶 100 个,共需铅皮多少 m2(接头损耗不计,x 取 3.14, 82 = 9.06,结果保留两个有效数 字) 11.若干毫升水倒人底面半径为 2cm 的圆柱形器皿中,量得水面的高度为 6cm,若将这 些水倒人轴截面是正三角形的倒圆锥形器皿中,求水面的高度.(圆锥形器皿容积 V=1 3 πr2h) 12.已知一个圆锥的底面半径为 R,高为 h。在其中有一个高为 x 的内接圆柱。 (1)求圆柱的侧面积; (2)x 为何值时,圆柱的侧面积最大?
查看更多